Download as pdf or txt
Download as pdf or txt
You are on page 1of 146

SMLE Questions and Answers

September 2019 - August 2021


‫ﻏﯿﺮ ﻣﻜﺘﻤﻠﺔ‬
https://t.me/WafaOBGYN
‫اﻟﺨﻄﺄ وارد‬
‫أﻛﺮﻣﻮﻧﻲ ﺑﺼﺪق اﻟﺪﻋﻮات ﻓﻲ ظﮭﺮ اﻟﻐﯿﺐ‬

References:
- American College of Obstetrics and Gynecology (ACOG)
- Uptodate
- Williams
- Amboss (rarely)
Contents
1. Puberty Disorders, Pediatrics, Adolescent and Young Gynecology……………………………………
2. Menstrual Cycle Abnormalities and Abnormal Uterine Bleeding (AUB)
3. Vaginal Infections………………………………………………………………………………………………………………
4. Pelvic Inflammatory Disease…………………………………………………………………………………
5. Family Planning, Contraception and HRT.…………………………………………………………………………
6. Polycystic Ovarian Syndrome (PCOS) …………………………………………………………………………
7. Preconception and Infertility ……………………………………………………………………………………………
8. Contraindications to Pregnancy (Medications and Vaccines) ………………………………
9. Abortion, Pregnancy Loss, and Intrauterine Fetal Demise (IUFD) ………………………………………………
10. Ectopic Pregnancy ………………………………………………………………………………………………………………
11. Gestational Trophoblastic Disease (Molar pregnancy and Choriocarcinoma) ………………
12. Cervical incompetence ………………………………………………………………………………………………………………
13. Hypertension in Pregnancy and Preeclampsia…………………………………………………………………………
14. Diabetes Mellitus and Gestational Diabetes Mellitus……………………………………………………….
15. Fetal Conditions and Abnormalities
16. Pregnancy Related Medical and Surgical Conditions ……………………………………………………
17. Investigations and Screening Tests During Pregnancy ……………………………………………….
18. Infections in pregnancy ………………………………………………………………………………………………………………
19. UTI in Pregnancy……………………………………………………………………………………………………………………………
20. Pregnancy Related Hematological Problems …………………………………………………………………………
21. Abnormal Placenta Implantation………………………………………………………………………………………………
22. Antepartum hemorrhage …………………………………………………………………………………………………………………………
23. Labor and CTG monitoring………………………………………………………………………………………………………………
24. Preterm labor, Preterm Rupture of Membrane (PROM) and Premature Preterm Rupture of Membrane
(PPROM) …………………………………………………………………………………………………………………
25. Postpartum Hemorrhage (PPH) ……………………………………………………………………………………..
26. Postpartum……………………………………………………………………………………………………………………….
27. OBGYN Related Breast diseases and Breastfeeding………………………………………………………………………………….
28. Adnexal masses ………………………………………………………………………………………………………………
29. Leiomyoma (Uterine Fibroids) and Leiomyosarcoma ………………………………………………………
30. Uterine Polyps ………………………………………………………………………………………………………………
31. Endometriosis ……………………………………………………………………………………………………………………………
32. Adenomyosis ………………………………………………………………………………………………………………………………
33. Asherman’s Syndrome…………………………………………………………………………………………………………………
34. Menopause ………………………………………………………………………………………………………………………………………
35. Vulvar and Vaginal Disorders
36. Cervical cancer and Screening ……………………………………………………………………………………………………
37. Endometrial Hyperplasia and Carcinoma …………………………………………………………………………
38. Urogynecology

By: Wafa AlSalem 2


Puberty Disorders, Pediatrics, Adolescent and Young Gynecology
Topic Overview:
Vaginal agenesis (Mayer-Rokitansky-Kuster-Hauser syndrome)( Müllerian agenesis )
Clinical presentation:
⁃ NORMAL female karyotype with normal ovaries and ovarian function, thus they
develop normal secondary sexual characteristics (eg, breast development,
axillary hair, and public hair).
⁃ PRESENTS ONLY with primary amenorrhea
Investigations:
Testosterone level: NORMAL “is one way to differentiate between Mayer-Rokitansky-
Küster-Hauser (MRKH) syndrome and androgen insensitivity.”

Complete androgen insensitivity (CAIS)


Clinical presentation:
⁃ Primary amenorrhea
⁃ Little or no axillary and pubic hair (coarse pubic hair).
Investigations:
Testosterone level: within or above the normal range for boys/men. (High testosterone levels for a women)

17 Years old female, no menstruation, high testosterone, Normal breast development, coarse pubic hair?
A. Mayer Rokitansky Kuster Hauser syndrome (or Mullerian agenesis)
B. Complete androgen insensitivity
C. Congenital hypothyroidism

17 Years old female, no menstruation, normal testosterone, Normal breast development, normal pubic hair?
A. Mayer Rokitansky Kuster Hauser syndrome (or Mullerian agenesis)
B. Complete androgen insensitivity
C. Congenital hypothyroidism

17year old female, medically free, (athlete) gymnast in her class, breasts later and never menstruated, on
developed examination she is tanner stage 5, but no menstruation, diagnosis?

A-Hypothalamic hypogonadism
B-Transverse vaginal septum
C-Gonadal agenesis
D-Testicular feminization

Note to Remember (ACOG)


**Menstrual cycle disturbances (e.g. amenorrhea) in young women ESPECIALLY ATHLETES!! Can be due to
underlying eating disorders and dietary restrictions à resulting into hypogonadotropic hypogonadism!
Please read the explanation below by ACOG:

By: Wafa AlSalem 3


**Transverse vaginal septum will present with other symptoms(in addition to amenorrhea), e.g., periodic lower
abdominal pain, urinary retention and bulging bluish membrane!

‫وﻋﺸﺎن اﺧﺘﺼﺮھﺎ ﻋﻠﯿﻚ اول ﻣﺎﺗﺸﻮف ﺑﺎﻟﺴﺆال‬female athlete


‫ ﻋﻠﻰ طﻮل وﻗﻒ ﻗﺮاﯾﮫ اﻟﺴﺆال ودور ﺑﺎﻻﺟﺎﺑﺎت ﻋﻠﻰ‬hypothalamic hypogonadism
‫وﻓﺮ وﻗﺘﻚ ﻟﺴﺆال اﺻﻌﺐ‬

By: Wafa AlSalem 4


Menstrual Cycle Abnormalities and Abnormal Uterine Bleeding (AUB)
Topic Overview:
Abnormal Uterine Bleeding (AUB) or Heavy Menstrual Bleeding (HMB)
Defined as bleeding from the uterine corpus that is abnormal in regularity, volume, frequency, or duration and
occurs in the absence of pregnancy.

Acute AUB
Refers to an episode of heavy bleeding that, in the opinion of the clinician, is of sufficient quantity to require
immediate intervention to prevent further blood loss. Acute AUB may occur spontaneously or within the
context of chronic AUB

Management of Acute AUB in reproductive aged women (ACOG+ Williams Gynecology)


**At times, women with AUB may have brisk bleeding that requires acute intervention (Hemodynamically unstable)-
> Start with fluid resuscitation and blood transfusion as indicated first!! -> Then Medical treatment is
simultaneously administered to slow bleeding
**Medical Management should be the initial treatment for most patients with acute AUB
Treatment options include:
• IV conjugated equine estrogen: Primary choice for heavy bleeding (Once bleeding has slowed, patients can
be transitioned to an oral taper using Prernarin pills or more commonly COCs.)
• Combined oral contraceptives (OCs): Primarily for less severe bleeding
• Oral progestins: Primarily for less severe bleeding and when estrogen is contraindicated
• Tranexamic acid (TXA): is also an option for acute heavy menstrual bleeding (HMB).

**Therapeutic D&C should be reserved as a last resort for the rare patient who continues to have life-threatening
bleeding despite high-dose of estrogen administration

Chronic AUB
Refers to abnormal uterine bleeding present for most of the previous 6 months

Management of Chronic AUB in reproductive aged women (Williams Gynecology)


⁃ Levonorgestrel-Releasing IUD (Mirena): First line treatment for heavy menstrual bleeding (HMB).
- Combined oral contraceptives (OCs)
- Depot medroxyprogesterone acetate
- Norethindrone acetate (progestins)
- Tranexamic acid (TXA)
- NSAID
• Mefenamic acid
• Naproxen
• Ibuprofen
• Flurbiprofen
- GnRH agonists

By: Wafa AlSalem 5


Patient came with heavy bleeding after doing evacuation the physician said it’s dysfunctional vaginal bleeding what
to do next?
A. D&C
B. OCPs
C. embolization
D. hysterectomy

Management of Acute AUB in reproductive aged women (ACOG+ Williams Gynecology)


**At times, women with AUB may have brisk bleeding that requires acute intervention (Hemodynamically unstable)-
> Start with fluid resuscitation and blood transfusion as indicated first!! -> Then Medical treatment is
simultaneously administered to slow bleeding
**Medical Management should be the initial treatment for most patients with acute AUB
Treatment options include:
⁃ IV conjugated equine estrogen: Primary choice for heavy bleeding (Once bleeding has slowed, patients can
be transitioned to an oral taper using Prernarin pills or more commonly COCs.)
⁃ Combined oral contraceptives (OCs): Primarily for less severe bleeding
⁃ Oral progestins: Primarily for less severe bleeding and when estrogen is contraindicated
⁃ Tranexamic acid (TXA): is also an option for acute heavy menstrual bleeding (HMB).

**Therapeutic D&C should be reserved as a last resort for the rare patient who continues to have life-threatening
bleeding despite high-dose of estrogen administration

Management of Chronic AUB in reproductive aged women (Williams Gynecology)


⁃ Levonorgestrel-Releasing IUD (Mirena): First line treatment for heavy menstrual bleeding (HMB).
- Combined oral contraceptives (OCs)
- Depot medroxyprogesterone acetate
- Norethindrone acetate (progestins)
- Tranexamic acid (TXA)
- NSAID
• Mefenamic acid
• Naproxen
• Ibuprofen
• Flurbiprofen
- GnRH agonists

Williams Gynecology:

About 28 years old, came to ER with heavy vaginal bleeding, she is nulliparous, pregnancy test negative, no pain,
regular cycle, but always with menorrhagia, how to stop the bleeding now?
A- Give conjugated estrogen
By: Wafa AlSalem 6
B- Give progesterone
C- Give GnRH
D- Insert levenogestrel IUD

Same note as the previous question.


40s old female, with heavy bleeding came to ER, what to do?
A- IUD
B- D/C
C- Hysterectomy
D- Mefenamic acid

Same note as the previous question.

D&C ‫ﻣﺎﻟﻘﯿﺖ ادوﯾﮫ اﻻﻛﯿﻮت ﻣﺎﻧﺠﻤﻨﺖ ﺑﺨﺘﺎر‬acute management! ‫ھﻮ ﻗﺎﻟﻲ ﺟﺎﯾﺘﻨﻲ اﻟﻄﻮارئ ﻓﺮاح اﺳﻮي‬
IUD and Mefenamic acid for chronic HMB not acute! ‫ﻷن‬
Let’s Exclude!!
- IUD-> first line for chronic HMB
- D&C -> in case of acute bleeding
- Hysterectomy-> why?
- Mefanamic acid -> also for chronic HMB but not the first line!

A female patient present with heavy PV bleeding. Her bleeding is associated with pain and of large volume. Upon
vaginal examination, you noticed pooling of blood. Pregnancy test is negative, her BP low, Labs: RBC low, Hb low
What is your next step in management?
A. Progesterone
B. Conjugated estrogen
C. Blood transfusion

In hemodynamically unstable patients -> Start with fluid resuscitation and blood transfusion as indicated first!! ->
Then Medical treatment is simultaneously administered to slow bleeding (IV conjugated equine estrogen)

Patient with uterine bleeding. What is the most appropriate investigation?


A) CBC
B) TFT
C) B-HCG
D) US

According to (ACOG)
To Exclude pregnancy
Pregnancy should be excluded in all reproductive-age patients with AUB

Patient with intermenstrual bleeding. What is the most appropriate investigation?


A) CBC
B) TFT
C) B-HCG
D) US

By: Wafa AlSalem 7


According to (ACOG)
Intermenstrual Bleeding is mainly caused my uterine and cervical etiologies.

Lets Exclude!!
-CBC would be initial
-TFH would be initial (Thyroid dysfuction causes oligo- or amenorrhea or menorrhagia and NOT intermenstrual
bleeding)
-B-HCG its intermenstrual bleeding (between menstruations!!), I would go for it if its only bleeding or AUB, but the
type of bleeding is specified in the question
-US is the primary imaging is the primary imaging test of the uterus for the evaluation of AUB (ACOG)

Female patient complaining of pain before menses and resolved in the third day of menses, how do you diagnose it?
A-Hysteroscopy
B-Abdominal US
C-Clinical symptoms

According to (ACOG)

Primary Dysmenorrhea:
Is the cramping pain that comes before or during a period pain tends to lessen after the first few days of a period.
In the absence of pelvic pathology.
- Evaluation include:
• History to determine whether the patient has primary dysmenorrhea or symptoms suggestive of
secondary dysmenorrhea.
When a patient presents with symptoms only of primary dysmenorrhea, a pelvic examination is not
necessary.

Secondary dysmenorrhea:
Is caused by a disorder in the reproductive organs. The pain tends to get worse over time and it often lasts longer
than normal menstrual cramps.
- Causes include:
• Endometriosis diagnosed by histologic evaluation of a lesion biopsied during surgery (typically
laparoscopy)
• Adenomyosis diagnosed by histology after hysterectomy

primary ‫ﺑﮭﺬا اﻟﺴﺆال ﺑﺲ ﻋﻄﺎﻧﻲ اﺧﻒ ﻋﺮض واﻟﻠﻲ ھﻮ ﺑﺲ ﺑﻄﻨﮭﺎ ﯾﻌﻮرھﺎ وﯾﺮوح ﻣﺎﻗﺎل ﯾﺰﯾﺪ ﻣﻊ اﻟﻮﻗﺖ وﻻ ﻗﺎل ﻣﻌﮫ أﺷﯿﺎء ﺛﺎﻧﯿﮫ ﺗﺨﻮﻓﻨﻲ ﻓﯿﻌﻨﻲ اﻧﮫ‬
dysmenorrhea

By: Wafa AlSalem 8


Patient with severe abdominal pain with menstrual cycle (dysmenorrhea), affecting her work, what can you give?
A-Misoprostol
B-NSAID
C-Progesterone
D-OCP

Patient with severe abdominal pain with menstrual cycle (dysmenorrhea), affecting her work, what can you give?
A-Misoprostol
B-Paracetamol
C-Progesterone
D-OCP

According to (ACOG)
Management of Primary Dysmenorrhea
- NSAID (first line)
- OCP (second line, if a trial of NSAIDs does not provide adequate relief of dysmenorrhea symptoms)

17 yrs old female came C/O primary dysmenorrhea, How will you manage??
A/ COCPs
B/ NSAIDs
C/ Mefanimic acid

36-year-old lady with secondary amenorrhea (elevated FSH & LH) which of risk or complication she might probably
develop in the future?
A) risk of endometrial cancer
B) risk of ovarian cancer
C) risk of osteoporosis

High FSH levels with amenorrhea indicates spontaneous primary ovarian failure -> which is a risk for osteoporosis

Vaginal infections
Topic Overview:

Pelvic Inflammatory Disease


Topic Overview:

Intrauterine device implantation patient have brown discoloration and abdominal pain. Whats the most likely
diagnosis?
A. Uterine rupture
B. Pelvic inflammatory disease

By: Wafa AlSalem 9


Family Planning, Contraception and HRT.

Polycystic Ovarian Syndrome (PCOS)


Topic Overview:

Infertile women with irregular cycle and PCOS:


A. induction of ovulation with clomiphene
B. Induction of ovulation with gonadotropin
C. Use OCP to regulate her cycle

By: Wafa AlSalem 10


Preconception and Infertility
Topic Overview:

Folic acid quantity for a healthy lady wants to conceive and with no prior diseases or disorders
A) 1 mg
B) 5 mg
C) 10 mg
D) 15 mg

According to ACOG, Uptodate and Williams Obstetrics:


Female prepregnancy folic acid supplementation should be encouraged to reduce the risk of NTDs.
- All women of reproductive age (15–45 years) should take folic acid supplementation. For average-risk
women, supplementation with 400 micrograms per day is adequate.
- Women at increased risk of NTDs, including women with a prior pregnancy with an NTD or women with
seizure disorders, should be counseled to take 4 mg of folic acid daily

‫ھﺬي اﺻﺢ اﺟﺎﺑﮫ‬0.4 mg ‫ﻟﻮ ﻓﯿﮫ‬

Also, from ACOG:


Standard 1mg of folate in prenatal vitamins
‫ﯾﻌﻨﻲ اﻻﺟﺎﺑﮫ اﻟﻠﻲ اﺧﺘﺮﺗﮭﺎ ﺻﺢ‬

By: Wafa AlSalem 11


Contraindications to Pregnancy (Medications and Vaccines)
Topic Overview:
PRECONCEPTION IMMUNIZATION (Williams Obstetrics + UTD)

- Influenza
o Vaccinate all women who will be pregnant during flu season. Vaccinate high-risk women prior to flu
season.
o Vaccination against influenza throughout the influenza season, but optimally in October or
November, is recommended by the Centers for Disease Control and Prevention (CDC) and the
American College of Obstetricians and Gynecologists for all women who will be pregnant during the
influenza season.

- Measles, mumps, rubella (MMR)


o Screen for rubella immunity. If nonimmune, vaccinate and counsel on the need for effective
contraception during the subsequent month.
- Varicella
o Screen for varicella immunity. If nonimmune, vaccinate and counsel on the need for effective
contraception during the subsequent month.
o
- Tdap (tetanus, diphtheria, pertussis)
o Update vaccination in all reproductive-aged women.

**Before administering a live vaccine to a woman of childbearing age, reasonable practices should include asking
the woman if she is pregnant or could become pregnant in the next four weeks and counseling her about the
potential risks of vaccination during pregnancy or just before conception.
**Routine pregnancy testing before vaccination is not recommended if absence of pregnancy is reasonably certain
by history
**ACIP and American College of Obstetricians and Gynecologists (ACOG) recommendations to avoid pregnancy for
one month following each dose of a live vaccine. Nevertheless, adverse outcomes in women who became pregnant
soon after receiving these vaccines have not been established

ROUTINE PRENATAL IMMUNIZATIONS (UTD)


- Tetanus, diphtheria, and pertussis vaccination
o Routinely recommended during pregnancy
o Td immunization complete (previously immunized with a full three-dose series)
§ Pregnant women should receive a single dose of Tdap, ideally during the early part of the 27
to 36 week gestational age range
§ Tdap is indicated in each pregnancy, even if the woman has a previous history of pertussis
or vaccination, and even if consecutive pregnancies occur within 12 months.
§ For women for whom 10 years has elapsed since the last Td booster, this Tdap dose in
pregnancy will also serve as a booster vaccination against tetanus and diphtheria. If the
Tdap cannot be given, a Td booster during pregnancy is recommended if ≥10 years have
elapsed since a previous Td booster (administer during the second or third trimester) or if
booster protection is needed for wound management (administer when needed). Booster
protection for wound management is advised when the last booster was ≥5 years before a
wound that is not clean and minor.
o No, incomplete, or unknown immunization against tetanus and diphtheria (not received three doses
of a vaccine)

By: Wafa AlSalem 12


§ Pregnant women should undertake or complete the series of three vaccinations. The
preferred schedule in pregnant women is at time 0, 4 weeks later, and at 6 to 12 months
after the initial dose.
§ To also provide protection against pertussis, at least one of these doses should be with
Tdap; ideally, Tdap is given during the early part of the 27 to 36 week gestational age
range.
§ Tdap can be given earlier than 27 to 36 weeks and at any stage of pregnancy if a woman
lives in an area with a pertussis epidemic or requires a tetanus booster as part of wound
management. If it is given early in pregnancy, it should not be repeated later in pregnancy.
o If Tdap is not administered during pregnancy, it should be administered immediately postpartum to
women who have not previously received Tdap. This can provide protection to the infant indirectly
or, if breastfeeding, through transfer of maternal antibodies in breast milk; however, the maternal
immune response may not be sufficiently rapid to protect the infant until two weeks post-
immunization
- Inactivated Influenza Vaccine
o All women who are pregnant or might be pregnant during the influenza season should receive
the inactivated influenza vaccine as soon as it becomes available and before onset of influenza
activity in the community, regardless of their stage of pregnancy. Vaccination after onset of
influenza activity is still beneficial as long as influenza viruses are circulating.

LIVE-VIRUSES VACCINES ARE CONTRAINDICATED DURING PREGNANCY. EXAMPLES:


- Varicella-zoster
- Measles, mumps, rubella (MMR)
- Polio
- Chickenpox
- Yellow fever.
- Intranasal Influenza
**1 month or longer should ideally pass between vaccination and conception attempts
**Inadvertent administration of measles, mumps, rubella (MMR) or varicella vaccines during pregnancy should not
generally be considered indications for pregnancy termination.

POSTPARTUM IMMUNIZATION
Both inactivated and live vaccines (except smallpox and yellow fever vaccine) may be administered to nursing
mothers, and breastfeeding does not adversely affect success or safety of vaccination. Smallpox and yellow fever
vaccines are avoided in nonemergency situations because breastfed infants of vaccinated women are at risk of
developing vaccinia and meningoencephalitis, respectively.
- MMR and varicella – The following vaccines should be given before discharge to protect a nonimmune
mother and newborn:
o MMR – The measles, mumps, rubella (MMR) vaccine should be administered to women nonimmune to
rubella or measles
o Varicella vaccination is recommended for women without evidence of immunity.
The first dose is given while the patient is in the hospital and the second dose is given four to eight
weeks later, which typically coincides with the routine postpartum visit. Breastfeeding is not a
contraindication to administration.
For RhD-negative women who received anti-D postpartum, MMR and/or varicella vaccine is still administered
immediately postpartum, when indicated. Nevertheless, the CDC suggests that women who have received both anti-
D immune globulin and rubella vaccine be serologically tested after vaccination, if feasible, to ensure that
seroconversion has occurred. The ACIP recommends waiting ≥3 months before evaluating the immune response.

By: Wafa AlSalem 13


- HPV – Eligible patients who have not been previously vaccinated against human papillomavirus (HPV), or who
have not completed the vaccine series, may receive the vaccination postpartum; the vaccine can be safely
given to those patients who are breastfeeding.
- Tdap – If not given as recommended during pregnancy, Tdap should be given postpartum to women.
Breastfeeding is not a contraindication to administration.

28 years old female with history of recurrent pregnancy loss. She comes now want to improve her immunity before
trying to conceive. What you will give her?
A. Influenza vaccine
B. Rubella vaccine
C. Hepatitis
D. B immunoglobulin

According to UTD
Congenital rubella syndrome — Rubella infection can have catastrophic effects on the developing fetus, resulting in
spontaneous abortion, fetal infection, stillbirth, or intrauterine growth restriction

‫ ﯾﺴﺒﺒﮭﺎ‬rubella ‫ ﻷن‬recurrent pregnancy loss ‫ ﺑﺠﻤﻠﮫ ال‬not immunized to rubella ‫ ان اﻻم‬hint ‫ﺑﺎﻟﺴﺆال ﻣﻌﻄﯿﻨﻲ‬

Female her previous pregnancy is stellbirth and now she want to pregnant and ask the doctor
about all the vaccines that she is need before conception and reduce the stillbirth ?
A. Rubella
B. Varicella
C. Influenza

Same note as the previous question

Married women came in winter to OB /gyn clinic she want to conceive later what vaccine you should give her before
conceive
A. Rubella
B. Influenza
C. varicella
D. Tdap

According to Williams Obstetrics and UTD

PRECONCEPTION IMMUNIZATION (Williams Obstetrics + UTD)

- Influenza

By: Wafa AlSalem 14


o Vaccinate all women who will be pregnant during flu season. Vaccinate high-risk women prior to flu
season.
o Vaccination against influenza throughout the influenza season, but optimally in October or
November, is recommended by the Centers for Disease Control and Prevention (CDC) and the
American College of Obstetricians and Gynecologists for all women who will be pregnant during the
influenza season.

- Measles, mumps, rubella (MMR)


o Screen for rubella immunity. If nonimmune, vaccinate and counsel on the need for effective
contraception during the subsequent month.
- Varicella
o Screen for varicella immunity. If nonimmune, vaccinate and counsel on the need for effective
contraception during the subsequent month.
o
- Tdap (tetanus, diphtheria, pertussis)
o Update vaccination in all reproductive-aged women.

‫ ﺑﺲ ھﻨﺎ ﻣﺎﻗﺎﻟﻲ‬rubella ‫ ﺑﺨﺘﺎر‬recurrent pregnancy loss ‫وﻻ ﻗﺎﻟﻲ ﻋﻨﺪھﺎ‬not immunized ‫ﯾﻌﻨﻲ ﻟﻮ ﺑﺎﻟﻤﻌﻄﯿﺎت ﻗﺎﻟﻲ ھﻲ‬
‫ ﻓﻠﺬﻟﻚ ﻟﯿﺶ اﺧﺘﺎر روﺑﯿﻼ ؟ ﯾﻤﻜﻨﮭﺎ ﻣﺘﻌﻄﻤﮫ أﺻﻼ وھﻮ ﻣﺎﯾﻨﻌﻄﻰ اﻻ ﻟﻠﻲ ﻣﺐ ﻣﺘﻄﻌﻤﮫ‬،‫ﺷﻲء ﯾﺨﻠﯿﻨﻲ اﺷﻚ اﻧﮭﺎ ﻣﻮ ﻣﺘﻄﻌﻤﮫ زي ﺑﺎﻷﺳﺌﻠﺔ اﻟﻠﻲ ﻗﺒﻞ‬

Pregnant in 2nd tri which vaccine?


A. H. influenza
B. DTAP

According to UTD
- Pregnant women should receive Tdap, ideally during the early part of the 27 to 36 week gestational age
range (third trimester)
- Inactivated Influenza Vaccine
All women who are pregnant or might be pregnant during the influenza season should receive
the inactivated influenza vaccine as soon as it becomes available and before onset of influenza activity in
the community, regardless of their stage of pregnancy

Pregnant lady what vaccine should she get at first visit:


A- Influenza
B- Tdap

Same note as the previous question

Pregnant nullipara what vaccine should you give her


a-Influenza
b-DTaP
c-Rh immunoglobulin

Same note as the previous question


Depends on the gestational age
if 1st or second trimester-> I would go with influenza
if third trimester -> I would go with Tdap

By: Wafa AlSalem 15


Female have regular cycle every 30 days her last period before 36 ,,, she received rubella vaccine before 3 weeks.
She was asked not to get pregnant until 2 months of receiving rubella , But she got pregnant, what’s the most likely
pregnancy outcome?
A. Not affected
B. Associated with congenital malformations.

According to Williams Obstetrics and UTD


**Inadvertent administration of measles, mumps, rubella (MMR) or varicella vaccines during pregnancy should not
generally be considered indications for pregnancy termination.
**ACIP and American College of Obstetricians and Gynecologists (ACOG) recommendations to avoid pregnancy for
one month following each dose of a live vaccine. Nevertheless, adverse outcomes in women who became pregnant
soon after receiving these vaccines have not been established

Pregnant women doesnt have rubella vaccine what should do?


A. Take in second trimester
B. Postpartum

According to UTD

POSTPARTUM IMMUNIZATION
Both inactivated and live vaccines (except smallpox and yellow fever vaccine) may be administered to nursing
mothers, and breastfeeding does not adversely affect success or safety of vaccination. Smallpox and yellow fever
vaccines are avoided in nonemergency situations because breastfed infants of vaccinated women are at risk of
developing vaccinia and meningoencephalitis, respectively.
- MMR and varicella – The following vaccines should be given before discharge to protect a nonimmune
mother and newborn:
o MMR – The measles, mumps, rubella (MMR) vaccine should be administered to women nonimmune to
rubella or measles
o Varicella vaccination is recommended for women without evidence of immunity.
The first dose is given while the patient is in the hospital and the second dose is given four to eight
weeks later, which typically coincides with the routine postpartum visit. Breastfeeding is not a
contraindication to administration.

Pregnant her child school had an outbreak and she’s afraid to get to her child or something which vaccine she
should get ?
1/DtaP
2/Influenza
3/ Rubella
4/ Varicella

According to UTD
- Inactivated Influenza Vaccine
All women who are pregnant or might be pregnant during the influenza season should receive
the inactivated influenza vaccine as soon as it becomes available and before onset of influenza activity in
the community, regardless of their stage of pregnancy

A woman was taking highly androgenic progesterone without knowing she is pregnant. What complication will her
daughter face?
By: Wafa AlSalem 16
A. Nothing will change
B. Hirsutism
C. Masculinization
D. Feminization

According to UTD:
Gestational hyperandrogenism — Virilization in an XX individual with normal female internal anatomy can result
from exposure to maternal androgen or synthetic progestational agents. Because the placenta produces the
aromatase enzyme, which converts androgens to estrogens, only very high levels of maternal androgens can
overcome placental aromatase to cause virilization of the fetus. Causes include maternal luteoma or theca lutein
cysts. These disorders are suggested by a history of maternal virilization during pregnancy and/or exogenous
progestin or androgen exposure

According to Williams Obstetrics:

By: Wafa AlSalem 17


Abortion, Pregnancy Loss, and Intrauterine Fetal Demise IUFD
Spontaneous Abortion
• Loss of pregnancy before 20 weeks' gestation
Type Clinical Presentation Ultrasound Finding Management
Complete -Cervix closed -No products of conception Follow up in OPD
-Bleeding -No fetal activity
-Passage of products of
conception (completely)
Incomplete -Cervix dilated -Some products of conception - Expectant management (if ≤13 weeks GA)
-Bleeding -No fetal activity Up tp 8 weeks, if the pregnancy has not passed in a
-Passage of some products of reasonable time or if bleeding, infection, or other
conception and the rest is within complication develop -> Managed by medical or
the cervical or uterus surgical.
Inventible -Cervix dilated -Products of conception - Medical (up to 20 weeks GA)
-Bleeding present By two different medications mifepristone followed
-No passage of products of -Fetal activity may be present by misoprostol
conception. POC are visible within **considered in women without infection, hemorrhage,
the cervical canal or in uterus severe anemia, or bleeding disorders
Missed -Cervix closed -Fetus is dead in the uterus
-No bleeding -No fetal activity - D&C (up to 20 weeks GA)
** Women who present with hemorrhage,
hemodynamic instability, or signs of infection should
be treated urgently with surgical uterine evacuation.
Threatened -Cervix closed -Products of conception Expectant management.
-Bleeding present Resume physical activity.
-No passage of products of -Fetal activity present
conception
Fetal Demise or Stillbirth (IUFD)
• The delivery of a fetus showing no signs of life as indicated by the absence of breathing,
heartbeats, pulsation of the umbilical cord, or definite movements of voluntary muscles.
• Fetal deaths at 20 weeks or greater of gestation (if the gestational age is known), or a weight
greater than or equal to 350 grams if the gestational age is not known

Pregnant in first trimester develop vaginal bleeding and LL quadrant pain, she denied any passage of tissue
U/S shows : No sac either intrauterine or extrauterine , Dx ?
A-Ovarian ectopic
B-Complete abortion
C-missed abortion
D- Pregnancy of unknown location

Note to Remember
- Ovarian ectopic à adnexal mass on US
- Complete abortion à Passage or expulsion of tissue
- Missed abortion à a nonviable pregnancy in absence of symptoms! (OUR PT IS SYMPTOMATIC)
- Pregnancy of unknown location à if the gestational sac cannot be seen at all on US (When a pregnant patient
with pain and/or bleeding has an US that has no findings of pregnancy)

Pregnancy 7 weeks with vaginal bleeding. And no intrauterine or extrauterine sac


A. US after 2 days
B. Reassurance
C. Discharge home

Pregnancy of Unknown Location (UTD)


By: Wafa AlSalem 18
ABSENCE OF FINDINGS on US:
- When a pregnant patient with pain and/or bleeding has an US that has no findings of pregnancy (ie, no
intrauterine pregnancy [IUP], adnexal mass, or echogenic fluid in the pelvis),
- The differential diagnosis is normal early IUP, nonviable IUP, or ectopic pregnancy. Approximately 15 to 26
percent of patients with ectopic pregnancy will have a "normal" initial US

- In a hemodynamically stable patient with a desired pregnancy:

o an US with no findings should be repeated when the human chorionic gonadotropin (hCG) reaches the
discriminatory zone for endometrial findings
o or US with no findings should be repeated in three to four days since the gestational sac of an IUP
grows approximately 1 mm per day and is visible on US when it reaches 3 mm or greater.

Miscarriage in an old lady (~45yrs) she asked if her age had anything to do with her miscarriage:
A- risk of miscarriage is 3% at this age
B- from 10 to 50%
C- 80%
D- no risk

Pregnant suddenly at her mid second trimester came to ER complaining of gush of fluid + tissue passed out. What is
the diagnosis?
A. Complete abortion
B. cervical incompetence
C. Incomplete abortion
D. Threatened abortion

Note to Remember

‫ اﻛﺜﺮ ﻋﺸﺎن ﻗﺎل ﻧﺰﻟﺖ‬complete abortion ‫ ﺑﺲ ﻓﻲ ھﺬي اﻟﺤﺎﻟﮫ راح اروح ﻣﻊ‬،‫ ﻣﻔﺘﻮح وﻻ ﻻ وھﻞ ﻧﺰل ﻛﻠﮫ وﻻ ﺑﺲ ﺷﻮي‬cervix ‫اﻟﻤﻔﺮوض ﯾﻮﺿﺢ اﻛﺜﺮ ھﻞ ال‬
‫وﺳﻜﺖ‬
..‫ ﻛﺎن ﻋﻄﺎﻧﻲ ﺗﻔﺎﺻﯿﻞ اﻛﺜﺮ زي ﺑﺎﻻﺗﺮاﺳﻨﺎوﻧﺪ ﻟﻘﯿﻨﺎ ﺑﺎﻗﻲ وﻣﺎ اﻟﻰ ذﻟﻚ‬incomplete ‫وﻟﻮ ﯾﺒﻲ‬

By: Wafa AlSalem 19


Woman with hx of miscarriage, now in 7 weeks with vaginal spotting, no hx of passing tissue , on examination os is
closed and no bleeding

1. Threatened abortion
2. Normal pregnancy
3. Incomplete abortion
4. Inevitable Abortion

Note to Remember
Physiologic or implantation bleeding characterized by a small amount of spotting or bleeding approximately 10 to 14
days after fertilization (at the time of the missed menstrual period)
٧ ‫ﻟﻮ اﺳﺎﺑﯿﻌﮭﺎ ﻗﻠﯿﻞ ﻛﺎن ﻗﻠﺖ ﻧﻮرﻣﺎل ﺑﺮﯾﻘﻨﺎﻧﺴﻲ ﺑﺲ ھﻲ اﻻﺳﻮع‬

Picture of ultrasound (as shown above) and patient presenting with minimal bleeding and she denied passage of
anything & cervical os is closed, she’s pregnant at 8 weeks
A. Complete abortion
B. Threatened abortion
C. Missed abortion
D. Anembryonic sac

Note to Remember

Ultrasound picture is showing an empty gestational sac. ‫ﺻﻮره ﺗﻘﺮﯾﺒﯿﮫ وﻟﯿﺴﺖ اﻟﺼﻮره اﻟﺘﻲ ﺑﺎﻻﺧﺘﺒﺎر‬

Anembryonic pregnancy refers to a nonviable pregnancy with a gestational sac that does not contain a yolk sac or
embryo. Anembryonic pregnancy contrasts with "embryonic or fetal demise" in which an embryo or fetus is
visualized but cardiac activity is not present.

Pregnant woman at 15 weeks of gestation, came with severe bleeding and component seen in cervix. What is the
most appropriate next management?
A. IV fluid and misoprostol
B. IV fluid and expectant management
By: Wafa AlSalem 20
C. IV fluid and D&C
D. IV fluid and uterine message

Note to Remember
Inventible, incomplete, and missed abortions are managed by: (ACOG+UTD)
- Expectant management (if ≤13 weeks GA)
Up tp 8 weeks, if the pregnancy has not passed in a reasonable time or if bleeding, infection, or other complication
develop -> Managed by medical or surgical.
- Medical (up to 20 weeks GA)
By two different medications mifepristone followed by misoprostol
**considered in women without infection, hemorrhage, severe anemia, or bleeding disorders

- D&C (up to 20 weeks GA)


** considered in women who present with hemorrhage, hemodynamic instability, or signs of infection should be
treated urgently with surgical uterine evacuation.

A 19-year-old pregnant and did a home pregnancy test 8 weeks back.


She came to the clinic to do an examination and they found out that the baby was dead at 5 weeks old, and she had
black blood clot
1-Observation
2-Suction
3-Misoprostol
4-Expectant management

Same note as the previous question

Woman at 7 weeks came with abortion female denies passage of part of product after doctor
examination show tissue product in cervix, clear scenario what is management?
A. Expectant management
B. D&C
C. Progesterone
D. For surgery

Same note as the previous question


A 19-year-old pregnant and did a home pregnancy test 8 weeks back.
She came to the clinic to do an examination and they found out that the baby was dead at 5 weeks old, and she had
black blood clot
1-Observation
2-Suction
3-Misoprostol
4-Expectant management

Same note as the previous question

Woman at 14 weeks came with abortion female denies passage of part of product after doctor
examination show tissue product in cervix, clear scenario what is management?
A. Expectant management
B. D&C
C. Progesterone
D. For surgery
By: Wafa AlSalem 21
‫ھﺬا اﻟﺴﯿﻨﺎرﯾﻮ ﺟﺎﻧﻲ ﻧﺎﻗﺺ ﺑﺪون اﺳﺎﺑﯿﻊ اﻧﺎ اﺿﻔﺖ اﻷﺳﺎﺑﯿﻊ‬
‫!!ﻻن اﻟﺤﻞ ﯾﺨﺘﻠﻒ ﻋﻠﻰ ﺣﺴﺐ اﻻﺳﺒﻮع‬
‫ ﻣﺘﺄﻛﺪه اﻧﮫ‬bad recall
‫واﻧﮭﻢ ﺣﺎطﯿﻦ اﻷﺳﺎﺑﯿﻊ‬

If she is 13 weeks or less > expectant


If she is 14-20 weeks> medical or D&C

Patient presenting with severe bleeding in 9th week of pregnancy, Os is open, doctor saw some tissue on the cervix.
What is your management?
A-Expectant management
B-D&C
C-Oxytocin
D-For Surgery

- D&C is indicated if the patient has bleeding!


- I would go for expectant if she’s not bleeding!

23 years old female pregnant in her 28th week, pregnancy test was done at home. She presented with mild
bleeding loss of fetal movement. She denied any passage of tissue or abdominal pain. Transvaginal US was done and
showed pregnancy of 18 weeks and no heart beat. (No other information was provided about the cervix). What is
your diagnosis?
A) Fetal Demise
B) Missed Abortion
C) Incomplete abortion

‫ ﺑﺲ ﺑﻘﻮﻟﻜﻢ اﻧﺎ ﻛﯿﻒ ﻓﺎھﻤﺘﮫ وﻣﻤﻜﻦ اﻛﻮن ﻏﻠﻄﺎﻧﮫ‬،‫!ﻋﺎد ھﺬا اﻟﺴﺆال ﻣﻦ اﻟﺴﻨﮫ اﻟﻠﻲ ﻓﺎﺗﺖ ﻣﺤﯿﺮ اﻟﻌﻠﻤﺎء وﺟﺎﻧﻲ ﺑﺎﻻﺧﺘﺒﺎر‬
Missed abortion
‫ھﻮ ان اﻟﻮﺣﺪه ﯾﺘﻮﻓﻰ اﻟﺒﯿﺒﻲ ﻓﻲ ﺑﻄﻨﮭﺎ وﯾﺠﻠﺲ ﺑﺎﻻﺳﺎﺑﯿﻊ وﻣﺎﺗﺤﺲ ﺑﺎي ﻋﺮض اﻻ ان اﻋﺮاض اﻟﺤﻤﻞ راﺣﺖ زي اﻟﺘﻄﺮﯾﺶ وﻣﺎ اﻟﻰ ذﻟﻚ وﺗﺴﺘﻐﺮب وﺗﺮوح ﺗﻜﺸﻒ)وﺑﻌﺾ اﻻﺣﯿﺎن‬
‫ ﻣﻦ ﻓﺤﺺ ﺑﺎﻟﺒﯿﺖ ﯾﻌﻨﻲ ﻣﻮ دﻗﯿﻖ ﺑﺎﻻﺳﺎﺑﯿﻊ ﻓﺤﺼﮭﺎ‬٢٨ ‫ وﻛﻤﺎن ﯾﻘﻮل اﺳﺒﻮع‬،‫ وﺗﻮھﺎ ﺗﺴﺘﻮﻋﺐ وﺗﺠﻲ‬١٨ ‫ ﻓﻤﻤﻜﻦ اﻟﺒﯿﺒﻲ ﻣﯿﺖ ﻣﻦ اﺳﺒﻮع‬،(‫ﻧﺰﯾﻒ ﺧﻔﯿﻒ‬

‫ﺑﺲ ﺷﻔﺖ ﻛﺜﯿﺮ ﻣﺮﺳﻠﯿﻨﮫ ﺑﺎﻟﺮاﺑﻂ ﻗﻠﺖ ﺧﻞ ادﻟﯿﻠﻜﻢ ﻣﻦ دﻟﻮي 😂 ﻓﺤﻠﻲ ھﺬا ﺷﻲء ﯾﺨﺼﻨﻲ وﻟﺤﺪ ﯾﻘﻮﻟﻲ ﺟﯿﺒﻲ ﻣﺼﺪر ﻷن ﻣﺎﻓﻲ ﻣﺼﺪر وﺳﺆال ﺧﺎﯾﺲ‬

‫ وﻣﻤﻜﻦ ﻗﺒﻞ ﻟﻮ ﻋﻨﺪھﺎ اﻛﺜﺮ ﻣﻦ ﺑﯿﺒﻲ‬١٦ ‫وﺑﺎﻟﻨﺴﺒﮫ ﻟﻠﻔﯿﺘﺎل ﻣﻮﻓﻤﻨﺖ ھﻲ ﺗﺒﺪأ اﺻﻼ ﻣﻦ اﺳﺒﻮع‬
UTD: Maternal perception of fetal movement typically begins in the second trimester at around 16 to 20 weeks of
gestation and occurs earlier in parous women than nulliparous women

Patient at 8 weeks gestation, presenting to the ED with vaginal bleeding and abdominal pain, her cervical OS is open
and tissue can be see within the cervical os. What is your diagnosis?
A- Threatened abortion
B- Incomplete abortion
C- Inevitable abortion
D- Complete abortion

Let’s Exclude!
- Threatened-> cervix will be closed

By: Wafa AlSalem 22


- Incomplete abortion-> she will give a history of some tissue passage

primigravida come to clinic with no symptoms, cervical opening, and intact baby puls, what the type of abortion?
A-thretened abortion
B-inevitable abortion
C-incomplete abortion
D-anembryonic pregnancy

Same note as the previous question

Pregnant 37 wks with IUFD, feeling guilty because she smoke 5 cigarettes / day, what to tell her?
A. Smoking not related to IUFD of IVF
B. Smoking increase rate of IUFD
C. Should stop smoking for future pregnancy
D. Cause not known and need more investigation

According to UTD
The work-up of the patient with a stillbirth is guided by several factors. Approach should be guided by clinical
data, timing of the death, the mother's medical history, whether fetal growth restriction was present, and
sonographic and histopathologic findings.
‫ﯾﻌﻨﻲ ﻣﻮ ﺑﺲ ﻋﺸﺎﻧﮭﺎ ﺗﺪﺧﻦ اﺣﻂ اﻟﻤﺸﻜﻠﮫ ﺑﺎﻟﺘﺪﺧﯿﻦ ﻻزم اﺑﺤﺚ اول وش اﻟﺴﺒﺐ‬

By: Wafa AlSalem 23


Ectopic Pregnancy
Topic Overview:
Classification
- Fallopian tube is the most common location of ectopic implantation, accounting for more than 90% of cases. The
ampulla (70 percent) is the most frequent site, followed by isthmic (12 percent), fimbrial (1 1 percent), and
interstitial tubal pregnancies (2 percent)
- However, implantation in the abdomen (1%), cervix (1%), ovary (1–3%), and cesarean scar (1–3%) can occur and
often results in greater morbidity because of delayed diagnosis and treatment.
- An ectopic pregnancy also can co-occur with an intrauterine pregnancy, a condition known as heterotopic
pregnancy. The risk of heterotopic pregnancy among women with a naturally achieved pregnancy is estimated to
range from 1 in 4,000 to 1 in 30,000, whereas the risk among women who have undergone in vitro fertilization is
estimated to be as high as 1 in 100.
Risk Factors for Tubal Pregnancy
⁃ History of ectopic pregnancy
• Risk of ectopic pregnancy (women with a history of one previous ectopic) à 10%
• Risk of ectopic pregnancy (women with a history of two or more previous ectopic) à more than 25%
- Abnormal fallopian tube anatomy
- Peritubal adhesions subsequent to salpingitis, appendicitis, or endometriosis
- Previous damage to the fallopian tubes, secondary to surgeries for a prior tubal pregnancy, for fertility
restoration, or for sterilization.
- Prior sexually transmitted disease or other tubal infection, which can distort normal tubal anatomy, is another
factor. Specially, one episode of salpingitis
⁃ Women who become pregnant through the use of assisted reproductive technology(IVF), certain factors
such as tubal factor infertility and multiple embryo transfer
⁃ Women with a history of infertility also are at increased risk of ectopic pregnancy independent of how they
become pregnant
⁃ Congenital fallopian tube anomalies, especially those secondary to in utero diethylstilbestrol exposure,
⁃ Other less significant risk factors include a history of cigarette smoking and age older than 35 years.
⁃ Women who use an intrauterine device (IUD) have a lower risk of ectopic pregnancy than women who are
not using any form of contraception because IUDs are highly effective at preventing pregnancy. However,
up to 53% of pregnancies that occur with an IUD in place are ectopic
- With some contraceptive method failures, the relative number of ectopic pregnancies is increased. Examples
include tubal sterilization, copper and progestin-releasing intrauterine devices (IUDs), and progestin-only
contraceptives
Clinical Presentation:
- Classic triad is delayed menstruation, pain, and vaginal bleeding or spotting.
- Abdominal palpation: tenderness.
- Bimanual pelvic examination: cervical motion tenderness.
- The uterus can also be slightly enlarged due to hormonal stimulation.
- The posterior vaginal fornix may bulge from blood in the rectouterine cul-de-sac, or a tender, boggy mass may
be felt beside the uterus.
Tubal Rupture
- Acute course with sudden and severe lower abdominal and pelvic pain is usually severe and frequently described
as sharp, stabbing, or tearing.
- Signs of hemorrhagic shock (e.g., tachycardia, hypotension, syncope)

By: Wafa AlSalem 24


Diagnosis:
Serum B-hCG level
Rapid and accurate determination of pregnancy is essential to identify an ectopic pregnancy.

Transvaginal Ultrasound (TVUS)


- Performed to look for findings indicative of uterine or ectopic pregnancy.
- During endometrial cavity evaluation, an intrauterine gestational sac is usually visible between 4 1/2 and 5
weeks. The yolk sac appears between 5 and 6 weeks, and a fetal pole with cardiac activity is first detected at 5
1/2 to 6 weeks
- Adnexal Findings: The sonographic diagnosis of ectopic pregnancy rests on visualization of an adnexal mass
separate from the ovary. If fallopian tubes and ovaries are visualized and an extrauterine yolk sac, embryo, or
fetus is identified, then an ectopic pregnancy is confirmed.

Medical Management (ACOG+Williams Obstetrics)


- Methotrexate MTX is the preferred treatment option when all of the following characteristics are present:
• Hemodynamic stability.
• Serum beta-human chorionic gonadotropin (hCG) concentration ≤5000 milli-international units/mL.
• No fetal cardiac activity detected on transvaginal ultrasound (TVUS).
• Ectopic mass size less than 4 cm
• Patients are willing and able to comply with post-treatment follow-up and have access to emergency
medical services within a reasonable time frame in case of a ruptured fallopian tube.
- Absolute Contraindications to Methotrexate MTX therapy (ACOG)
• Intrauterine pregnancy
• Evidence of immunodeficiency
• Moderate to severe anemia, leukopenia, or thrombocytopenia
• Sensitivity to methotrexate
• Active pulmonary disease
• Active peptic ulcer disease
• Clinically important hepatic dysfunction
• Clinically important renal dysfunction
• Breastfeeding
• Ruptured ectopic pregnancy
• Hemodynamically unstable patient
• Inability to participate in follow-up
- Relative Contraindications to Methotrexate MTX therapy (ACOG)
• Embryonic cardiac activity detected by transvaginal ultrasonography
• High initial hCG concentration >5000 milli-international units/mL.
• Ectopic pregnancy greater than 4 cm in size as imaged by transvaginal ultrasonography
• Refusal to accept blood transfusion

By: Wafa AlSalem 25


Methotrexate Treatment Protocols (ACOG)
Single-dose regimen Two-dose regimen Fixed multiple-dose regimen
- Administer a single dose of - Administer methotrexate at a dose of - Administer methotrexate 1 mg/kg
methotrexate at a dose of 50 mg/m2 50 mg/m2 intramuscularly on day 1 intramuscularly on days 1, 3, 5, 7;
intramuscularly on day 1 - Administer second dose of alternate with folinic acid 0.1 mg/kg
methotrexate at a dose of 50 mg/m2 intramuscularly on days 2, 4, 6, 8
intramuscularly on day 4
Measure hCG level on posttreatment day 4 Measure hCG level on posttreatment day 4 Measure hCG levels on methotrexate dose
and day 7 and day 7 days and continue until hCG has decreased
— If the decrease is greater than 15% — If the decrease is greater than 15% by 15% from its previous measurement
à measure hCG levels weekly until reaching à measure hCG levels weekly until reaching — If the decrease is greater than 15%
nonpregnant level nonpregnant level à discontinue administration of
— If decrease is less than 15% — If decrease is less than 15% methotrexate and measure hCG levels weekly
à readminister methotrexate at a dose of à readminister methotrexate 50 mg/m2 until reaching nonpregnant levels (may
50 mg/m2 intramuscularly and repeat hCG intramuscularly on day 7 and check hCG ultimately need one, two, three, or four
level levels on day 11 doses)
— If hCG does not decrease after two doses — If hCG levels decrease 15% between day 7 — If hCG does not decrease after four
à consider surgical management and day 11 doses
à continue to monitor weekly until reaching à consider surgical management
nonpregnant levels
— If the decrease is less than 15% between
day 7 and day 11
à readminister dose of methotrexate 50
mg/m2 intramuscularly on day 11 and check
hCG levels on day 14
— If hCG does not decrease after four
doses
à consider surgical management
If hCG levels plateau or increase during follow-up àconsider administering methotrexate for treatment of a persistent
ectopic pregnancy

What surveillance is needed after methotrexate treatment?


After administration of methotrexate treatment, hCG levels should be serially monitored. Serum B-hCG
determinations are measured weekly until undetectable.

Failure of Medical Treatment:


- When the B-hCG level plateaus or rises, fail to decrease adequately by 15% from days 4 to 7 postinjection.or
the tube ruptures.
- Managed by:
o A repeat single dose of methotrexate (as indicated)
o Surgical management (if maximum doses of methotrexate have reached or rupture of tubes)

By: Wafa AlSalem 26


Surgical Management (ACOG+Williams Obstetrics)
- Indications:
• Hemodynamic instability
• Symptoms of an ongoing ruptured ectopic mass (such as pelvic pain), or signs of intraperitoneal
bleeding.
• When a patient meets any of the absolute contraindications to medical management and should be
considered when a patient meets any of the relative contraindications.
o Hemodynamic instability.
o Intrauterine pregnancy, including a heterotopic pregnancy with coexisting viable intrauterine
pregnancy
o Signs or symptoms of impending or ongoing rupture of ectopic mass (eg, pelvic or abdominal pain
or evidence of intraperitoneal bleeding suggestive of rupture).
o Clinically important abnormalities in baseline hematologic, renal, or hepatic laboratory values
o Medical conditions such as immunodeficiency, active pulmonary disease (eg, tuberculosis), and
peptic ulcer disease
o Hypersensitivity to MTX.
o Breastfeeding.
• When a patient who initially elects medical management experiences a failure of medical management
(When the B-hCG level plateaus or rises or the tube ruptures).

- Surgical management generally is performed using:


• Laparoscopic salpingectomy (removal of part or all of the affected fallopian tube)
• Or laparoscopic salpingostomy (removal of the ectopic pregnancy while leaving the affected fallopian
tube in situ).
• Laparotomy typically is reserved for unstable patients, patients with a large amount of intraperitoneal
bleeding, and patients in whom visualization has been compromised at laparoscopy.
Laparoscopy is the preferred surgical treatment for ectopic pregnancy unless a woman is hemodynamically unstable
Ruptured tubal pregnancies with hemoperitoneum-can safely be managed laparoscopically.

- Salpingectomy Versus Salpingostomy:


• Salpingectomy
o Standard procedure if the condition of the tube with the ectopic gestation is damaged
(ruptured or otherwise disrupted), bleeding is uncontrolled, or the gestation appears too large
to remove with salpingostomy.
o For patients who have completed childbearing, bilateral salpingectomy may be performed as
permanent sterilization.
o Advantage of salpingectomy is that it avoids the need for further treatment for persistent
trophoblast
• Salpingostomy
o Preservation of the tube for potential future fertility.
o Preferred for patients who desire future childbearing and in whom the contralateral tube is
absent or damaged.
o Requires monitoring the patient with serial hCG measurement to ensure resolution of ectopic
trophoblastic tissue. If there is concern for incomplete resection, a single prophylactic dose of
methotrexate may be considered.

By: Wafa AlSalem 27


- Persistent Trophoblast
• After surgery, B-hCG levels usually fall quickly and approximate 10 percent of preoperative values by
day 12.
• Persistent trophoblast is rare following salpingectomy, but complicates 5 to 1 5 percent of
salpingostomies.
• Bleeding caused by retained trophoblast is the most serious complication.
• Incomplete removal of trophoblast can be identified by stable or rising B-hCG levels.
• Monitoring approach: measure serum B-hCG levels weekly levels.
• With stable or increasing B-hCG levels, additional surgical or medical therapy is necessary.
o Without evidence for tubal ruptureà standard therapy for this is single-dose MTX, 50 mg/m 2
X body surface area (BSA).
o With evidence of rupture and bleeding à require surgical intervention.

Follow Up after treatment:


- Serial beta-hCG measured weekly after treatment until the level is undetectable.
- **Exception (According to UTD):
o For patients who undergo salpingectomy-> if the pathology evaluation confirms a tubal gestation, many
surgeons do not check a postoperative hCG, and others check a single postoperative hCG to confirm a
large decline in the level.

Pt had previous ectopic pregnancy asked about the percentage in the next pregnancy to be ectopic?
A. 5%
B. 10%
C. 30%
D. 50 %

Note to Remember
- Risk of ectopic pregnancy (women with a history of one previous ectopic) à 10%
- Risk of ectopic pregnancy (women with a history of two or more previous ectopic) à more than 25%

By: Wafa AlSalem 28


40 years old female, P4 with a history of tubal ligation 4 years ago after her last CS delivery. Her last Menstrual
period was 6weeks ago.
Presented to ER with vaginal spotting. NO abdominal pain
O/E: OS is closed. Tenderness and fullness in her abdomen in RLQ. What is the best Initial investigation?
A- Pregnancy test
B- US
C-CT Abdomen

According to Williams Obstetrics


TUBAL STERILIZATION

LONG-TERM COMPLICATIONS:
- Contraceptive failure
Approximately 30 percent ofpregnancies that follow a failed tubal sterilization procedure are ectopic. his rate is
20 percent for those following a postpartum procedure (Peterson, 1996, 1 997). hus, any symptoms of pregnancy in
a woman after tubal sterilization must be investigated, and an ectopic pregnancy excluded.

According to Comprehensive Gynecology Textbook:


CONTRACEPTION FAILURE AFTER STERILIZATION

In an analysis of the long-term risk of pregnancy after tubal sterilization reported by Peterson and coworkers, it
was found that within 10 years after the procedure, the cumulative life table probability of pregnancy was 1.85%.
The 10-year failure rate after bipolar coagulation of the oviducts was 2.48%, which rose to 5.43% if the
sterilization procedure was performed when the woman was younger than 28 years of age. These investigators
reported that for all 143 pregnancies occurring after tubal sterilization, 43, or 32.9%, were ectopic pregnancies
(Peterson, 1997).

Because about one third of pregnancies that occur after all tubal sterilizations are ectopic, women should be
counseled that if they do not experience the expected menses at any time following tubal sterilization before
menopause, a test to detect human chorionic gonadotropin (HCG) should be performed rapidly, and if they are
pregnant, a diagnostic evaluation to exclude the presence of ectopic pregnancy is necessary.

Case of ectopic pregnancy received methotrexate her bHCG were:


on day 1 ( 3000)
on day 4 ( 3100)
on day 7 ( 2900 ) what to do next?
A. Nothing
B. Surgery
C. Multi dose methotrexate regimen
D. Consider second dose of methotrexate

According to ACOG
Between 4 and 7 days after methotrexate is a adminstered, the HCG levels should fall at least 15%. If this amount
of decrease does not occur, an additional dose of methotrexate should be given

Our patient B-hCG on day4 was (3100) -> 15% decrease would be (3100-465=2635)
-> if our paitent B-hCG levels on day 7 is ≤2635 -> no further doses of methotrexate is required.

By: Wafa AlSalem 29


**But our patient has <15% decrease of B-hCG levels between day 4 and 7 -> which is an indication of readminister
methotrexate

Ectopic pregnancy. Start methotrexate weak ago. Follow up result:


D1 : BHCG 1000
D4: BHCG 1200
D7: BHCG 2200
What is the most appropriate next step?
A- Immediately laparotomy
B- Second dose methotrexate
C- Multiple dose methotrexate

Same note as the previous question

Ectopic pregnancy gave her methotrexate her Bhcg was 3100


after four days it became 3000
and after seven days it became 2900.
What to do?
A- give another dose of methotrexate
B- don’t give
C- observation

Same note as the previous question

Patient with ectopic pregnancy, her husband is in a military mission, she lives 80Km away fron hospital, brought by
her neighbor, US showed unviable fetus, abscent heart rate, 4cm fetus, BHCG is 5000, what is the indication for
surgical intervension?
A. BHCG
B. Fetal heart
C. Fetal size
D. far away from hospital Or distance

ANOTHER RECALL

BhCG 2500, 3cm, what is the contraindications for methotrexate ttt?


A. BhCg and ultrasound findings
B. Far away from hospital
C. Her vital signs

According to ACOG

- Absolute Contraindications to Methotrexate MTX therapy (ACOG)


• Intrauterine pregnancy
• Evidence of immunodeficiency
• Moderate to severe anemia, leukopenia, or thrombocytopenia
• Sensitivity to methotrexate
• Active pulmonary disease
• Active peptic ulcer disease
• Clinically important hepatic dysfunction
• Clinically important renal dysfunction
By: Wafa AlSalem 30
• Breastfeeding
• Ruptured ectopic pregnancy
• Hemodynamically unstable patient
• Inability to participate in follow-up
- Relative Contraindications to Methotrexate MTX therapy (ACOG)
• Embryonic cardiac activity detected by transvaginal ultrasonography
• High initial hCG concentration >5000 milli-international units/mL.
• Ectopic pregnancy greater than 4 cm in size as imaged by transvaginal ultrasonography
• Refusal to accept blood transfusion

A female patient diagnosed as ectopic pregnancy, where she lives far away from the hospital. Her B-HCG level is
6000 with an absent fetal heartbeat. What is the best management for her?
A. Methotrexate
B. Surgical intervention
C. Expectant management

Same note as the previous question

Female patient present in the emergency with lower abdominal pain and bleeding, ultrasound done and a 3cm ectopic
pregnancy found in the ovarie. The B-HCG was 15000. from the above history what will make the medical treatment
contraindicated
A. the size
B. the abdominal pain
C. B-HCG level

Same note as the previous question

Ecopic pregnancy at 7 or 9 week, 4 cm mass. (Bhcg not mentioned). Mx?


A. MTX
B. Surgical
C. Observation

Case of ectopic pregnancy, the ectopic size is less than 3.5 cm, BhCG is 2500 how will you manage?
A Medical
B Surgical

According to ACOG
- Methotrexate MTX is the preferred treatment option when all of the following characteristics are present:
• Hemodynamic stability.
• Serum beta-human chorionic gonadotropin (hCG) concentration ≤5000 milli-international units/mL.
• No fetal cardiac activity detected on transvaginal ultrasound (TVUS).
• Ectopic mass size less than 4 cm
• Patients are willing and able to comply with post-treatment follow-up and have access to emergency
medical services within a reasonable time frame in case of a ruptured fallopian tube.

Case of ectopic pregnancy Bhcg 3400, 3 cm what do?


A. Medical treatment
B. Surgical

By: Wafa AlSalem 31


Same note as the previous question

In a case of Ectopic pregnancy, B-HCG was 5000. What is management?


A. Surgery
B. Methotrexate

Same note as the previous question

A female patient her LMP was 6 weeks ago presented with mild abdominal pain. Vitally stable, Closed OS. US shows
no intrauterine pregnancy, but a 3 cm sac in the fornix area with no cardiac activity, BhCG was 3000. what is you
management?
a. Medical management
b. surgical management
c. medical and surgical management
d. Medical management given that she has access to the hospital

Same note as the previous question


A and D are both correct. But D is more accurate.

Case ectopic pregnancy ( history given) mother refusing OR. her B-hcg 3500 size of sac 3 non viable no signs of
rupture. what will u do?
A. explain that failure is high
B. treat medically but sign consent
C. laparoscopy
D. laparotomy

Same note as the previous question

A case of ectopic pregnancy, plateau B-HCG for 3 weeks. What to do?


A- Consider methotrexate
B- Reassure and follow up
C- Salpingostomy or Surgical management

According to ACOG
If hCG levels plateau or increase during follow-up àconsider administering methotrexate for treatment of a persistent ectopic
pregnancy

Case of ectopic pregnancy treated by Salpingostomy. On regular follow up her BHCG was decreasing until the last
three visits the BHCG results plateaued. BHCG was 3200 on the last visit (normal is less than 5000). How would you
manage the case?
A. Consider giving Methotrexate
B. Start OCP
C. Surgical intervention
D. Reassure

According to Williams Obstetrics


By: Wafa AlSalem 32
Persistent Trophoblast

- After surgery, B-hCG levels usually fall quickly and approximate 10 percent of preoperative values by day 12.
- Persistent trophoblast is rare following salpingectomy, but complicates 5 to 1 5 percent of salpingostomies.
- Bleeding caused by retained trophoblast is the most serious complication.
- Incomplete removal of trophoblast can be identiied by stable or rising B-hCG levels.
- Monitoring approache: measure serum B-hCG levels weekly levels.
- With stable or increasing B-hCG levels, additional surgical or medical therapy is necessary.
o Without evidence for tubal ruptureà standard therapy for this is single-dose MTX, 50 mg/m 2
X body surface area (BSA).
o With evidence of rupture and bleeding à require surgical intervention.

Bchg 108,000 Treated and under observation


1st week 700
2 week 300
3 week 180
4 week 70
5 week 300
What to do ??
1-Give MTX
2- Observing

Same note as the previous question

Patient had a salpingostomy, she is following up with the hcg every week, they noticed the hcg plateaued for 3
weeks on 3442, what’s next?
A. Laparotomy
B. OCP
C. Methotrexate
D. Reassurance

Same note as the previous question

A 34-year-old lady pregnant, complaining of amenorrhea, bleeding, and abdominal pain. β-HCG done showed levels of
1600, she was given methotrexate. One week later she still has severe abdominal pain despite analgesia. β-HCG
done showed 6000 units. What is the best management?
A. Salpingostomy
B. Salpingectomy
C. Continue methotrexate
D. Exploratory laparotomy

According to Williams Obstetrics and ACOG

Laparoscopy is the preferred surgical treatment for ectopic pregnancy unless a woman is hemodynamically unstable
Ruptured tubal pregnancies with hemoperitoneum-can safely be managed laparoscopically.
• Salpingectomy
o Standard procedure if the condition of the tube with the ectopic gestation is damaged
(ruptured or otherwise disrupted), bleeding is uncontrolled, or the gestation appears too large
to remove with salpingostomy.
By: Wafa AlSalem 33
**The severe abdominal pain indicates rupture of ectopic pregnancy-> which is an indication of laparoscopic
salpingectomy (unless hemodynamically stable-> laparotomy)

Ectopic pregnancy bhcg 3500, she is hypotensive and tachypnic. What is the most appropriate management?
A.Methotrexate
B.Laparoscopy slpingectomy
C. Laparotomy slpingectomy

According to Williams Obstetrics and ACOG

• Laparoscopy is the preferred surgical treatment for ectopic pregnancy unless a woman is
hemodynamically unstable
• Ruptured tubal pregnancies with hemoperitoneum-can safely be managed laparoscopically.
• Laparotomy typically is reserved for unstable patients, patients with a large amount of intraperitoneal
bleeding, and patients in whom visualization has been compromised at laparoscopy.

**Stable -> Laparoscopy


**Unstable-> Laparotomy

Female 25 years old, presented to the ED with vaginal bleeding, nausea, and right lower abdominal pain and
tenderness. History of open appendectomy due to perforated appendix. Her BP:90/50 HR:120 RR:25 What is the
possible site of bleeding?
A. Cervix
B. Uterus
C. Fallopian tube

ANOTHER RECALL

Female 25 years old, presented to the ED with vaginal bleeding, nausea, and right lower abdominal pain and
tenderness. History of open appendectomy due to perforated appendix. Her BP:90/50 HR:120 RR:25 What is the
diagnosis?
A. Ectopic pregnancy
B. Overian toration
C. Haemogragic cyst
D. Abscess

According to Williams Obstetrics


- Peritubal adhesions subsequent to salpingitis, appendicitis, or endometriosis are a risk factor for Ectopic
Pregnancy

Patient with tubal Rupture will present with:


- Acute course with sudden and severe lower abdominal and pelvic pain is usually severe and frequently described
as sharp, stabbing, or tearing.
- Signs of hemorrhagic shock (e.g., tachycardia, hypotension, syncope)

Female 25 years old, presented to the ED with Vague abdominal pain and amenorrhea for 2 months. History of open
appendectomy due to perforated appendix 14 year ago. Her B-hCG 1800. Whats the most appropriate Management
?
A. Surgical intervention after stabilizing
By: Wafa AlSalem 34
B. Gs review for acute abdominal
C. Strong analgesic
D. Methotrexate

According to Williams Obstetrics


- Peritubal adhesions subsequent to salpingitis, appendicitis, or endometriosis are a risk factor for Ectopic
Pregnancy
This is a case of ectopic pregnancy and according to the history she has no contraindications to Methotrexate
therapy.

Female had previous ectopic pregnancy want to conceive:


A. Early detection of pregnancy
B. Folic acid therapy

According to Comprehensive Gynecology Textbook

Women with an ectopic pregnancy who become pregnant again should be monitored by ultrasound early in
pregnancy.
Only about one of three nulliparous women who have had an ectopic pregnancy ever conceives again (35%), and
about one third of these conceptions are an ectopic pregnancy,

‫ وﻻ ﻻ‬early US ‫ﷲ ﻻ ﯾﺤﯿﺮ ﻣﺴﻠﻢ ﻛﻞ اﻻﺟﺎﺑﺘﯿﻦ ﺻﺢ ﺑﺲ ﻣﺪاﻣﮫ ﻗﺎل اﻛﺘﻮﺑﻚ اظﻨﮫ ﯾﺒﻲ ﯾﺸﻮف اﻧﺖ ﺗﻌﺮف اﻧﻨﺎ ﻣﻔﺮوض ﻧﺴﻮﯾﻠﮭﺎ‬

After salpingectomy how to follow?


A. One bhcg to confirm decline
B. Hystosaplingogram
C. Weekly bhcg
D. Pelvic ultrasound after 6 days

Follow Up after treatment:


- Serial beta-hCG measured weekly after treatment until the level is undetectable.
- **Exception (According to UTD):
o For patients who undergo salpingectomy-> if the pathology evaluation confirms a tubal gestation, many
surgeons do not check a postoperative hCG, and others check a single postoperative hCG to confirm a
large decline in the level.

Ectopic pregnancy managed with salpingostomy. Bhcg postoperative was 3500 . how to follow up the B HCG?
A. No need follow up
B. Pelvic ultrasound
C. Weekly measurements of B HCG until undetectable

Same note as the previous question

A LADY is pregnant at 7 weeks, she came to emergency room complaining of left iliac fossa pain and brownish
vaginal discharge, your provisional diagnosis:
A. ECTOPIC
B. APPENDICITIS
C. IRRITABLE BOWEL
D. Threatened miscarriage

By: Wafa AlSalem 35


She has a localized pain -> indicates ectopic more than threateend abortion
30 years old pregnant women by in vitro fertilization (IVF) and diagnosed with ectopic pregnancy and scheduled of
laproscopic removal of the prgnency. On US imaging: 4 cm tubal pregnancy on the right, and hydrosalpinx on the
left. What is the management?
A. Salpingotomy on the right only
B. Salpingectomy on the right only
C. Removal of both tubes (Bilateral tuboectomy)
D. Methotrexate

According to Comprehensive Gynecology Textbook


- If the hydrosalpinx is large and clearly visible on ultrasound, it is preferable to perform laparoscopic
salpingectomy prior to IVF-ET because the pregnancy rate with IVF-ET may be decreased by as much as 40%
- A significantly dilated tube (hydrosalpinx) on ultrasound is an indication for salpingectomy before carrying out
IVF.
According to Dewhurst’s Textbook of Obstetrics and Gynecology:
- In women undergoing IVF, the presence of hydrosalpinx is associated with early pregnancy loss and poor
implantation and pregnancy rates.
- Hydrosalpinges large enough to be visible on ultrasound are associated with the poorest outcome, including
increased miscarriage rates. Women with hydrosalpinges should therefore be offered treatment before IVF
because this improves the chance of a live birth.
- Various surgical treatments including salpingectomy, salpingostomy, proximal tubal ligation or clipping, and
transvaginal aspiration have all been used to improve IVF outcome.

‫اﻟﺠﺎي‬IVF ‫ ﻣﻤﻜﻦ ﯾﺨﻠﻲ ال‬hydrosalpinx ‫ ﻣﺎﻣﻨﮭﻢ ﻓﺎﯾﺪه وﺑﺎﻟﻌﻜﺲ ال‬fallopian tubes ‫ وﻋﻨﺪھﺎ اﻧﻔﯿﺮﺗﯿﻠﻲ ﯾﻌﻨﻲ ال‬IVF ‫ﯾﻌﻨﻲ ﻣﺮﯾﻀﺘﻨﺎ اﻟﺮﯾﺪي ﺟﺎﻟﺴﮫ ﺗﺴﻮي‬
‫ ﻓﻼزم اﺷﯿﻠﮫ‬،‫ﯾﻔﺸﻞ‬

Patient ectopic pregnancy on methotrexate and bhcg elevated in day 4 and 7 what to do?
A. recheck bhcg after 48 hours
B. recheck bhcg after one week
C. salpingectomy
D. discharge

According to Williams Obstetrics and ACOG:

Failure of Medical Treatment:


- When the B-hCG level plateaus or rises, fail to decrease adequately by 15% from days 4 to 7 postinjection.or
the tube ruptures.
- Managed by:
o A repeat single dose of methotrexate (as indicated)
o Surgical management (if maximum doses of methotrexate have reached or rupture of tubes)

Patient with a history of perforated ectopic and previous management , came now with lower abdominal pain .
Pregnancy test Labs show: 18000 bhcg Hb low. How to mange?
A. Consult surgery for possible acute abdomen
B. Misoprostol
C. Surgical management after stabilization

By: Wafa AlSalem 36


This is a case of ectopic pregnancy and her B-hCG level is an indication for surgical intervention

Women diagnosed as ectopic pregnancy asks what is the most common predisposing factor:
A. Previous tubal pregnancy
B. Pelvic Inflammatroy DiseasePID

According to UTD:

Female with amenorrhea for 6 weeks, pregnancy test positive, presented with abdominal pain, (can't recall if there
was bleeding or not), US showed no intrauterine pregnancy with minimal fluid in cul-de-sac and mass of 1.2 cm in
tube. She was vitally stable. What's your action?
A. Methotrexate
B. Salpingostomy
C. Salpingectomy

According to Williams Obstetrics


- A small amount of peritoneal fluid is physiologically normal.

According to UTD
- A small amount of clear free fluid in the pelvis is a normal sonographic finding. There is no established
threshold for the volume of fluid that is physiologic and the volume is difficult to measure sonographically.
However, fluid that is anechoic and isolated to the pelvic cul-de-sac and adjacent recesses is likely physiologic
- The presence or absence of peritoneal free fluid is not a reliable indicator of whether an ectopic pregnancy has
ruptured

**if large amount of fluid (indicates hemoperitoneum=ruptured ectopic)—> salpingectomy

Case about ectopic pregnancy (they didn’t mention the dx) ask about next step
A- BHCG quantitative
B- BHCG qualitative
C- Transvaginal Ultrasound

According to UTD
Human chorionic gonadotropin — Serum, rather than urine, hCG is the preferred test for a pregnant patient with
pain and/or bleeding

By: Wafa AlSalem 37


**Serum = quantitative

Ectopic pregnancy received MTX and gradually result in bHCG decreased, what is the next step?
A- Laparoscopy
B- Give 2nd dose MTX
C- observe

Answered by exclusion
..‫ واﺻﻼ اﻟﺘﺮﯾﺘﻤﻨﺖ ﻣﻊ اﻟﻤﯿﺜﻮﺗﺮﯾﻜﺴﯿﺖ ﯾﻨﺰل ﺑﺸﻜﻞ ﺗﺪرﯾﺠﻲ ﻣﻮ ﻓﺠﺄه‬..‫ﻣﺎﻋﻄﺎﻧﻲ ﻣﻌﻄﯿﺎت وﻻ ارﻗﺎم‬

Ectopic pregnancy is a defect in which process?


A. Implantation
B. Fertilization

22 years old female pregnant, with pregnancy test positive at home, came with sever abdominal pain, in examination
cervical is closed, there is fluid collection 15*13. Thin endometrium and empty uterus. what is the diagnosis?
A. Intact ectopic pregnancy
B. Ruptured ectopic pregnancy
C. Luteal phase

Severe abdominal pain and fluid collection indicates ruptured ectopic.


Intact tubes ?? we cant tell if the tubes are intact on US?? ‫ﺣﺮﻓﯿﺎ ﻣﺎﻋﺮف وش ﻗﺼﺪه ﻟﻤﺎ ﻗﺎل‬intact
‫اﺣﺴﮫ ﺑﺎد رﯾﻜﻮل‬

By: Wafa AlSalem 38


Gestational Trophoblastic Disease
(Molar pregnancy and Choriocarcinoma)
Topic Overview: WILLIAM OBSTETRICS + UTD
Gestational trophoblastic disease (GTD) is the term used to encompass a group of tumors typified by abnormal
trophoblast proliferation.
Divided into benign hydatidiorm moles and gestational trophoblastic neoplasia (GTN)

HYDATIFORM MOLE
Is a premalignant disease. It can be categorized as complete hydatidorm mole and partial hydatidiorm mole, which
differ by gross morphology, histopathology, karyotype, and risk of malignancy

RISK FACTORS
The main risk factors for HM are extremes of maternal age and a history of previous mole
- Prior molar pregnancy –The risk for repeat molar pregnancy after the first mole is approximately 1 to 1.5
percent (approximately 10 to 15 times the risk for the general population).
The recurrence rate after two molar pregnancies has been reported to range from 11 to 25 percent
- Extremes of maternal age (≤15 and >35 years)
- Asian and American Indian ancestry

CLINICAL PRESENTATION
Common features:
- Vaginal bleeding
- Pelvic pressure or pain
- Enlarged uterus (greater than normal)
- Hyperemesis gravidarum
Less common or late features
- Hyperthyroidism — Due to elevation of hCG >100,000 mIU/mL for several weeks. These patients may present
with tachycardia, warm skin, and tremor. Laboratory evidence of hyperthyroidism is commonly detected in
asymptomatic patients with HM
- Ovarian theca lutein cysts — Are a form of ovarian hyperstimulation resulting from high circulating levels of
hCG and prolactin
- Preeclampsia <20 weeks of gestation
- Passage of hydropic vesicles from the vagina
***** Partial mole has less severe symptoms than in complete mole (due to lower levels of B-hCG)

By: Wafa AlSalem 39


DIAGNOSIS
- Serum B-HCG Measurements (initial test)
o Markedly elevated (higher than expected for the gestational age)
o Higher in complete mole compared to partial mole
If the initial hCG level is high (>100,000 mIU/mL), a transvaginal ultrasound should be performed
- Transvaginal Ultrasound (TVUS)
• Complete mole
o Absence of an embryo or fetus.
o Absence of amniotic fluid.
o Central heterogeneous mass with numerous discrete anechoic spaces – This has classically been
described as a "snowstorm or Swiss cheese pattern"
o Ovarian theca lutein cysts

• Partial mole — Based upon ultrasound findings, a partial mole is diagnosed as a missed or incomplete
abortion in 15 to 60 percent of. These misdiagnoses are more common for partial mole than for
complete mole because only partial moles are accompanied by a fetus and amniotic fluid.
o A fetus may be identified, may be viable, and is often growth restricted.
o Amniotic fluid is present, but the volume may be reduced.
o Placenta with one or more abnormal findings – Enlarged, cystic spaces ("Swiss cheese pattern")
and/or increased echogenicity of chorionic villi.
o Increased transverse diameter of the gestational sac – These changes in the shape of the
gestational sac may be part of the embryopathy of triploidy.
o Theca lutein cysts are usually absent.
- HM is a histologic diagnosis, based upon a uterine evacuation specimen. (Definitive Diagnosis and Treatment)

MANAGEMENT
- Molar evacuation by suction curettage is usually the preferred treatment.
- Hysterectomy is a reasonable alternative for patients who have completed childbearing, particularly those with
a known or presumptive complete mole and the following risk factors for gestational trophoblastic neoplasia:
o Signs of trophoblastic proliferation (uterine size greater than gestational age, serum human chorionic
gonadotropin [hCG] levels >100,000 milli-international units/mL, ovarian theca lutein cysts >6 cm in
diameter)
o Age >40 years

POSTTREATMENT MANAGEMENT
- Anti-D immune globulin if Rh D-negative (in case of partial mole because it has fetal tissues)
- Initiate effective contraception
- Review pathology report
- Serum hCG levels: within 48 hours of evacuation, weekly until undetectable, then monthly for 6 months

By: Wafa AlSalem 40


POSTTREATMENT SURVEILLANCE
Serum hCG levels:
Close biochemical surveillance for persistent gestational neoplasia follows each hydatidiform mole evacuation.
This monitoring is by serial measurement of serum B-hCG to detect persistent or renewed trophoblastic
proliferation.
- The initial B-hCG level is obtained within 48 hours after evacuation. (baseline)
- Then weekly until undetectable
- Once B-hCG is undetectable(for three consecutive weeks), this is confirmed with monthly for another 6
months
**Importantly, during B-hCG level surveillance, either increasing or persistently plateaued levels mandate
evaluation for trophoblastic neoplasia. If the woman has not become pregnant, then these levels signify increasing
trophoblastic proliferation that is most likely malignant.

**Several factors predispose a patient to trophoblastic neoplasia following molar evacuation:


- Most important, complete moles have a 1 5 to 20 percent incidence of malignant sequelae, compared with 1
to 5 percent following partial moles.
- Other risk factors are older maternal age(>40), B-hCG levels > 100,000 mIU/mL, uterine size that is large
for gestational age, theca-lutein cysts >6 cm, and slow decline in B-hCG levels

Interpretation and management of hCG levels during monitoring

Decreasing and undetectable hCG levels — is defined as a level that progressively decreases >10 percent across
four values during a three week period (eg, on days 1, 7, 14, and 21).

Increasing hCG levels — is defined as a level that progressively increases >10 percent across three values during at
least a two week period (eg, on days 1, 7, and 14)

Plateaued hCG levels — is defined as four measurements that remain within ±10 percent over at least a three week
period (eg, days 1, 7, 14, and 21)

By: Wafa AlSalem 41


Contraception:
Concurrently, reliable contraception is imperative to avoid confusion caused by rising B-hCG levels from a new
pregnancy. Most recommend:
- Combination hormonal contraception
- Injectable depot medroxyprogesterone acetate (particularly useful if poor patient compliance is
anticipated.)
- Progestin implant (particularly useful if poor patient compliance is anticipated.)
**Intrauterine devices are not used until B-hCG levels are undetectable because of the risk of uterine perforation
if there is an invasive mole.
**Although not recommended, if a woman conceives during surveillance, livebirth rates and risk for congenital
anomalies appear to mirror the general population.
**After these 6 months, monitoring is discontinued and pregnancy allowed.

By: Wafa AlSalem 42


GESTATIONAL TROPHOBLASTIC NEOPLASIA (GTN)
Refers to a group of malignant neoplasms that consist of abnormal proliferation of trophoblastic tissue, and may
follow a hydatidiform mole or a nonmolar pregnancy.
GTN is comprised of the following histologic types:
- Invasive mole
- Choriocarcinoma
- Placental site trophoblastic tumor
- Epithelioid trophoblastic tumor.
**These tumors almost always develop with or after some form of recognized pregnancy. Half follow hydatidiform
mole, a fourth follow miscarriage or tubal pregnancy, and another fourth develop after a preterm or term
pregnancy

CLASSIFICATION AND CLINICAL BEHAVIOR


METASTATIC SITES
The risk of metastases varies depending on the histologic type and the duration of disease.
- Pulmonary (80 percent)
- Vagina (30 percent)
- Central nervous system (CNS) metastases (10 percent) are usually in the brain and, rarely, in the
meninges. Virtually all patients with CNS metastases have concurrent pulmonary and/or vaginal
involvement.
- Hepatic (10 percent)
- Other sites (kidney, gastrointestinal tract, spleen)
Choriocarcinoma
- Most common the most aggressive histologic type of GTN
- Characterized by early vascular invasion and widespread metastases.
- Metastases often develop early and are generally blood-borne(hematogenously)
- The clinical presentation is often due to bleeding from a metastatic site
- The most common sites are the lungs (presents with hemoptysis) and vagina
- Commonly accompanied by ovarian theca-lutein cysts.

CLINICAL PRESENTATION
GTN has a varied clinical presentation depending upon the antecedent pregnancy, extent of disease, and histologic
type.
- Elevated human chorionic gonadotropin (hCG) — An elevated hCG is what brings GTN to medical attention
after molar pregnancy.
- Abnormal uterine bleeding or amenorrhea
- Pelvic pain or pressure — If an enlarged uterus or ovarian cysts are present
Symptoms of metastases
- Pulmonary — Dyspnea, chest pain, cough, or hemoptysis may occur due to lung metastases.
- Vaginal — typically present with vaginal bleeding or purulent vaginal discharge.
- Central nervous system — may be asymptomatic initially, but as the disease progresses, patients develop
neurologic signs and symptoms due to increased intracranial pressure or hemorrhage, including: headache,
neuropathy, dizziness, nausea, slurred speech, visual disturbances, and/or hemiparesis
- Hepatic — Jaundice, epigastric, or back pain may occur in patients with liver metastases, but fewer than
one-third of patients with liver metastases are symptomatic
Patients with liver metastases may be at risk for intra-abdominal hemorrhage if the tumors rupture, which
represents a medical emergency. Hepatic lesions should not be biopsied because of risk of hemorrhage.
Other

By: Wafa AlSalem 43


- GTN is rarely associated with nephrotic syndrome or virilization. Virilization may be due to long-standing
stimulation of ovary by hCG, causing theca cell hyperplasia with subsequent elevated testosterone levels

*** If there are findings on examination or imaging that suggest metastatic disease, this supports the diagnosis.
Biopsies should not be performed because GTN lesions are highly vascular and may cause vigorous bleeding.

DIAGNOSIS AND STAGING


- The key elements of the diagnostic evaluation are to confirm an elevated human chorionic gonadotropin
(hCG) and evaluate for metastatic disease or hCG stimulation effects. The most common metastatic site is
the vagina or lungs. Uterine curettage or other biopsies have a limited role in the diagnosis of GTN,
because of risk of hemorrhage and because GTN is a clinical diagnosis.

- GTN is a clinical diagnosis made based upon elevation of serum human chorionic gonadotropin (hCG), after a
nonmolar pregnancy and after other etiologies of an elevated hCG have been excluded. Imaging findings of
uterine enlargement or pathology consistent with GTN, bilateral ovarian theca lutein cysts, or metastatic
disease support the diagnosis

- Unlike other solid tumors, a tissue diagnosis is not required prior to treatment, biopsy is not required and
may cause significant bleeding.

Once the diagnosis is verified, in addition to a baseline serum B-hCG level and hemogram, a search for local disease
and metastases includes:
- Tests of liver and renal function, transvaginal sonography, chest radiograph, and brain and abdominopelvic
CT scan or MR imaging.
- Less commonly, positron-emission tomographic (PET) scanning and cerebrospinal fluid B-hCG level
determination are used to identify metastases.

**GTN is staged clinically using the system of the International Federation of Gynecology and Obstetrics (FIGO)
and the World Health Organization (WHO) Prognostic Scoring System.

MANAGEMENT
Women with GTN are best managed by oncologists
**Chemotherapy alone is usually the primary treatment
- Single-agent chemotherapy for nonmetastatic or low-risk metastatic neoplasia
Monotherapy protocols with either methotrexate or actinomycin D
- Combination chemotherapy is given for high-risk disease

**Regimens are repeated until serum B-hCG levels are undetectable.


**Consolidation therapy — After complete remission is attained, consolidation therapy should be administered to
prevent relapse. This generally consists of three courses of the last effective regimen.
By: Wafa AlSalem 44
MONITORING DURING TREATMENT
All patients with GTN should be monitored with serial measurements of serum human chorionic gonadotropin (hCG)
at the start of treatment and then at weekly intervals during therapy.

**Defining remission — A disease remission requires three consecutive weekly normal hCG values (less than 5
mIU/mL). Treatment should then be continued for three consecutive courses of the last effective regimen to
reduce the risk of relapse
**Persistent or progressive disease — (or chemotherapy resistance) is defined as an increase or a plateau in two
consecutive hCG values over a two-week interval. Other generally accepted criteria include detection of new
metastases

POSTTREATMENT SURVEILLANCE
- After remission is achieved(-hCG levels are undetectable), serum human chorionic gonadotropin (hCG)
should be measured monthly in asymptomatic patients until one year of normal hCG levels has been
documented
- However, if following completion of one year of hCG surveillance the patient develops new symptoms, such
as abnormal bleeding, then recurrence should be considered, and an hCG value should be obtained.

Contraception:
- Estrogen-progestin contraceptives are preferred because of their low failure rate and relatively low
incidence of irregular bleeding, since this symptom may raise concern about recurrence
- During this time, effective contraception is crucial to avoid any teratogenic efects of chemotherapy to the
fetus and to mitigate conusion from rising B-hCG levels caused by superimposed pregnany.
- For those who conceive despite this within the surveillance year following treatment, pregnancy may
continue since most will have a favorable outcome. Importantly, this group is advised of the low but
important risk of delayed diagnosis if tumor recurs during the pregnancy.

Patient with LMP at 12 weeks while physical exam is at 19 weeks. US reveals molar. What is the management?
A- D & C
B- Methotrexate
C- Suction & Evacuation

Note to Remember

Molar pregnancy -> Uterine evacuation is for definitive diagnosis and treatment
**if she presents with HEMOPTYSIS-> it means its Choriocarcinoma à answer will be admit for staging and
chemotherapy (Methotrexate or dactinomycin)

Molar pregnancy case treated by dilatation and suction. What is the MOST COMMON early complication?
A. Perforation

By: Wafa AlSalem 45


B. Pneumonia
C. Amniotic fluid embolism…

According to UTD
Uterine perforation — is the most common immediate complication of D&C.

Common complication AFTER uterine Evacuation?


A. Perforation
B. Infection
C. Bleeding
D. Abscess

Note to Remember
Uterine perforation is the most common immediate complication of D&C.
After the procedure -> Infection (rare), intrauterine adhesions

Patient diagnosed with molar pregnancy did evacuation and curettage, bHcg monitoring after 1 month did not
change (didnt increase or decrease) what to do?
A. Repeat evacuation
B. Refer to oncology to start methotrexate
C. Observe as long its not elevated

According to Williams Obstetrics and UTD


POSTTREATMENT SURVEILLANCE
Serum hCG levels:
Close biochemical surveillance for persistent gestational neoplasia follows each hydatidiform mole evacuation.
This monitoring is by serial measurement of serum B-hCG to detect persistent or renewed trophoblastic
proliferation.
- The initial B-hCG level is obtained within 48 hours after evacuation. (baseline)
- Then weekly until undetectable
- Once B-hCG is undetectable (for three consecutive weeks), this is confirmed with monthly for another 6
months
**Importantly, during B-hCG level surveillance, either increasing or persistently plateaued levels mandate
evaluation for trophoblastic neoplasia. If the woman has not become pregnant, then these levels signify increasing
trophoblastic proliferation that is most likely malignant.

By: Wafa AlSalem 46


According to Williams Obstetrics
Women with GTN are best managed by oncologists

Molar pregnancy follow up of HCG?


A. Weekly until 3 negative
B. Weekly then monthly for 6 month
C. Monthly then weekly
D. Monthly until negative

Same note as the previous question

Paitent with choriocarcinoma (scenario) : having uterus size more than GA+ hemoptysis. Next?
A. Biopsy the mass
B. admit for D&E
C. admit for staging and chemo
D. admit for hysterectomy

According to Williams Obstetrics


In case of choriocarcinoma
Once the diagnosis is verified, in addition to a baseline serum B-hCG level and hemogram, a search for local disease
and metastases includes:
- Tests of liver and renal function, transvaginal sonography, chest radiograph, and brain and abdominopelvic
CT scan or MR imaging.
- Less commonly, positron-emission tomographic (PET) scanning and cerebrospinal fluid B-hCG level
determination are used to identify metastases.

**GTN is staged clinically using the system of the International Federation of Gynecology and Obstetrics (FIGO)
and the World Health Organization (WHO) Prognostic Scoring System.

Pregnant in her 10th week, presented with nausea ,vomiting and abdominal pain and hemoptysis, her fundal hight is
(large for gestational age ,forgot the number), bhcg levels are extremely high, speculum exam showed irregular
aggressive mass protruding from the cervix and liable to bleeding when touched, what’s your next step in
management?
A. biopsy the mass
B. Dilatation and evacuation
C. Staging and chemo
D. Hysterectomy

SAME NOTE AS THE PREVIOUS QUESTION


*** If there are findings on examination or imaging that suggest metastatic disease, this supports the diagnosis.
Biopsies should not be performed because GTN lesions are highly vascular and may cause vigorous bleeding.

Case of Choriocarcinoma with very very very high bhcg with hemoptysis ask about first thing to do?
A.evacuation
B.radio
C.chemo
D.chest x Ray

SAME NOTE AS THE PREVIOUS QUESTION

By: Wafa AlSalem 47


Chest xray is part of the staging.

Case of molar pregnancy with very high B-Hcg, treated and following up with her every week. B-hcg level given
weekly, dropping until week 5 it was 1. What will you do?
A. Next week B-hCG
B. Next month B-hCG
C. Discharge.
D. Give methotrexate.

According to UTD
POSTTREATMENT SURVEILLANCE
Interpretation and management of hCG levels during monitoring

Decreasing and undetectable hCG levels — is defined as a level that progressively decreases >10 percent across
four values during a three week period (eg, on days 1, 7, 14, and 21).

Increasing hCG levels — is defined as a level that progressively increases >10 percent across three values during at
least a two week period (eg, on days 1, 7, and 14)

Plateaued hCG levels — is defined as four measurements that remain within ±10 percent over at least a three week
period (eg, days 1, 7, 14, and 21)

‫ ﺑﺲ ﺑﻜﻞ اﻷﺣﻮال ﻣﺎﻧﺴﻮي ﺷﻲء وﻧﻘﯿﺴﮫ اﻷﺳﺒﻮع اﻟﻠﻲ ﺑﻌﺪه‬١ ‫اﻟﺼﺮاﺣﮫ ﻣﺪري وش ﯾﻌﻨﻲ ﺻﺎر‬

42 yo hx of molar pregnancy 2 years ago, she wants to conceive. What to do regarding her history:
A) early follow up in pregnancy
B) contraception she shouldn’t get pregnant

According to Williams Obstetrics


Women with prior hydatidiform mole sonographic evaluation is recommended in early pregnancy

12 gestation with fundal hight 19 wks and bhcg270000 ( very high) most likely dx:
A. Partial mole
B. Complete mole
C. Ectopic

According to Williams Obstetrics and UTD


CLINICAL PRESENTATION OF HYDATIFORM MOLE
Common features:
- Vaginal bleeding
- Pelvic pressure or pain
- Enlarged uterus (greater than normal)
- Hyperemesis gravidarum
Less common or late features
- Hyperthyroidism — Due to elevation of hCG >100,000 mIU/mL for several weeks. These patients may present
with tachycardia, warm skin, and tremor. Laboratory evidence of hyperthyroidism is commonly detected in
asymptomatic patients with HM
- Ovarian theca lutein cysts — Are a form of ovarian hyperstimulation resulting from high circulating levels of
hCG and prolactin
- Preeclampsia <20 weeks of gestation
- Passage of hydropic vesicles from the vagina
By: Wafa AlSalem 48
***** Partial mole has less severe symptoms than in complete mole (due to lower levels of B-hCG)

**THE VERY HIGH B-HCG AND ENLARGED UTERUS GOES MORE WITH COMPLETE MOLE THAN PARTIAL

Pregnant 8 weeks came with very high HCG, severe vomiting and nausea, abdominal pain, and heavy bleeding, passed
some vesicle, on vaginal examination you find some part of tissue and cervical os opening, uterine examination was
bulk, abdominal was tender and more than 8 weeks, What is the diagnosis ?
A. complet abortion
B. incomplete abortion
C. threatened abortion
D. complete hydatidiform mole

SAME NOTE AS THE PREVIOUS QUESTION

Case of snow storm appearance of uterus on ultrasound, counsel the patient on:
A- This condition is highly malignant
B- Risk of infertility must be addressed

According to Williams Obstetrics


- Women with prior hydatidiform mole generally do not have impaired fertility, and their pregnancy outcomes are
usually normal

- Hydatidiform mole is a premalignant disease.

Pregnant woman at 12th week gestation beta HCG is very high


A) Down syndrome
B) molar pregnancy

According to UTD

In case of Down syndrome


- Second-trimester quadruple test: measures the level of the biochemical markers AFP, unconjugated estriol
(uE3), human chorionic gonadotropin (hCG), and dimeric inhibin A (DIA) in maternal serum
Ideally performed at 15+0 to 18+6 weeks of gestation

In case of Molar pregnancy


- If the initial hCG level is high (>100,000 mIU/mL), a transvaginal ultrasound should be performed and will likely
demonstrate molar disease if present.
- If the hCG level is high and the ultrasound shows an apparently normal singleton gestation, the ultrasound and
hCG should be repeated in one week to exclude the possible presence of a twin conception with normal fetus
and coexistent molar pregnancy.

‫ ﺷﺪﺧﻞ اﻟﺪاون ﺳﻨﺪروم؟ اذا ﻛﺎن ﻣﺮه ﻋﺎﻟﻲ اول ﺷﻲء اﺷﻚ ﻓﯿﮫ ھﻮ اﻟﻤﻮﻻر ﺑﺮﯾﻘﻨﺎﻧﺴﻲ وﺣﺘﻰ ﻟﻮﺷﻔﺖ ﺣﻤﻞ طﺒﯿﻌﻲ ارﺟﻊ اﻋﯿﺪ اﻟﻔﺤﺺ ﺑﻌﺪ أﺳﺒﻮع‬،‫ھﺎﻟﺴﺆال ﺣﻠﯿﺘﮫ ﺑﺎﻟﻠﻮﺟﻚ‬
‫اﺗﺄﻛﺪ اﻧﮫ ﻣﻮ ﻣﻮﻻر! ﻣﺎش اﻟﺪاون ﺳﻨﺰوم ﻣﺎدﺧﻞ ﻣﺰاﺟﻲ ودﻟﯿﺖ ﺑﺪﻟﻮي‬

Post partum three months, came with hx of something that protruded from the cervix bleeding on touch, mx:
A. immediate D&C
B. measure hcg after 1 week
C. Biopsy
By: Wafa AlSalem 49
D. Tests for metastasis

This is a case of Choriocarcinoma following a normal gestation


According to UTD
DIAGNOSIS AND STAGING
GTN is a clinical diagnosis made based upon elevation of serum human chorionic gonadotropin (hCG), after a
nonmolar pregnancy and after other etiologies of an elevated hCG have been excluded.
Imaging findings of uterine enlargement or pathology consistent with GTN, bilateral ovarian theca lutein cysts, or
metastatic disease support the diagnosis
Unlike other solid tumors, a tissue diagnosis is not required prior to treatment, biopsy is not required and may
cause significant bleeding.

Laboratory evaluation
hCG — An elevated human chorionic gonadotropin (hCG) is often the first evidence of possible GTN. A serum
quantitative hCG should be drawn in all patients with suspected GTN.
- For women with a prior molar pregnancy, serial measurement of hCG is part of posttreatment surveillance,
and an elevation, plateau, or persistence of hCG suggests the development of GTN.
- For women with no prior history of a molar pregnancy, an elevated hCG may be initially presumed to be a
normal pregnancy. GTN may be suspected if pelvic ultrasound does not confirm a nonmolar pregnancy (viable
intrauterine pregnancy, spontaneous abortion, or ectopic pregnancy), or in some cases, if the patient is
certain that she has not conceived recently.

Following a nonmolar pregnancy — Women who develop GTN after a nonmolar pregnancy typically undergo evaluation
with serum hCG and ultrasound only after they become symptomatic.
- The diagnosis is made based upon an elevated hCG, with the exclusion of any other explanation than GTN. This
is sufficient for diagnosis even if there is no uterine enlargement and no evidence of metastatic disease. It is
critical to exclude a normal viable pregnancy and abnormal pregnancies (eg, spontaneous abortion, ectopic
pregnancy), ectopic hCG production by a nontrophoblastic tumor, or other causes of persistent low-level hCG.
- If there are findings on examination or imaging that suggest metastatic disease, this supports the diagnosis.
Biopsies should not be performed because GTN lesions are highly vascular and may cause vigorous
bleeding. Thus, hemorrhage is common, particularly from the uterus due to repeated trauma by dilation and
curettage. Life-threatening hemorrhage may necessitate embolization or resection of the affected organ.
Unlike other tumors, histologic confirmation is not necessary for diagnosis, although on rare occasions, a biopsy
may be needed if there is significant question about the diagnosis of GTN.
- Another exception to obtaining a histologic diagnosis is in patients who present with postpartum or postabortal
bleeding, uterine enlargement, or evidence of uterine disease on imaging. In these patients, a uterine curettage
may be performed and the diagnosis can be confirmed based upon the pathology evaluation of the curettage
specimen.
‫ اﻟﺤﯿﻦ ﻣﻮ ﺑﻌﺪ أﺳﺒﻮع ﻋﺸﺎن اﺷﺨﺼﮭﺎ‬bhcg and TVUS ‫ﯾﻌﻨﻲ ﻣﻔﺮوض اﺳﻮﯾﻠﮫ‬
..‫ﯾﻌﻨﻲ ﯾﻌﻨﻲ ﺑﺴﻮي ﺳﺘﯿﺠﻨﻖ ﻟﮭﺎ‬metastasis ‫وھﻲ اﻟﺮﯾﺪي ﺟﺎﯾﺘﻨﻲ ب‬

By: Wafa AlSalem 50


Cervical Insufficiency
Topic Overview: Summary from ACOG
Indications for Cervical Cerclage in Women With Singleton Pregnancies
History Indicated Cerclage:
• History of one or more second-trimester pregnancy losses related to painless
cervical dilation and in the absence of labor or abruptio placentae
• Prior cerclage due to painless cervical dilation in the second trimester

Placed at approximately 13–14 weeks of gestation.


Physical Examination Indicated Cerclage:
• Painless cervical dilation in the second trimester (known as emergency or rescue
cerclage)
Ultrasonographic Finding With a History of Prior Preterm Birth Indicated Cerclage:
• Current singleton pregnancy, prior spontaneous preterm birth at less than 34 weeks of gestation, and
short cervical length (less than 25 mm)
Before 24 weeks of gestation

ACOG:

Which patients should not be considered candidates for cerclage?


• Incidentally detected short cervical length in the second trimester in the absence of a prior singleton
preterm birth is not diagnostic of cervical insufficiency, and cerclage is not indicated in this setting.
• Vaginal progesterone is recommended as a management option to reduce the risk of preterm birth
in asymptomatic women with a singleton gestation without a prior preterm birth with an incidentally
identified very short cervical length less than or equal to 20 mm before or at 24 weeks of gestation
• Cerclage may increase the risk of preterm birth in women with a twin pregnancy and an ultrasonographically
detected cervical length less than 25 mm and is not recommended

By: Wafa AlSalem 51


G2P0 20 weeks of gestation, with cervical incompetence (cervix length 30mm), whats the most appropriate
management?
A. Cervical cerclage
B. Strict bed rest
C. Progesterone supplementation

Female with previous 2 preterm labour, now she is in 20 weeks of gestation and her cervix opened 30 mm, what you
will do?
A- Immediate cervical cerclage
B- Give tocolytic & wait
C- Strict bed rest
D- progesterone supplement

Note to Remember
For THIS CASE
- If her Cervical length <25mm WITH timing <24 weeks (ideally 13-16 weeks) à cervical cerclage
- Or if she has previous second trimester pregnancy and NOW SHE IS AT OR BEFORE 14 weeks GA à Cervical
cerclage
- So, she not a candidate for neither History o r US indicated cerclage
- So? Choose progesterone supplements.
!!‫ أﺳﺒﻮع ﻛﺎن ﺳﻮﯾﻨﺎ ﺳﯿﺮﻛﻼج ﺑﺲ ھﻲ ﺟﺎﯾﮫ ﻣﺘﺄﺧﺮ‬١٤ ‫ﯾﻌﻨﻲ ھﻲ ﻟﻮ ﺟﺎﯾﮫ ﺑﺪري ﻗﺒﻞ‬
‫!! ﯾﻌﻨﻲ ﻣﺎﺗﻨﻄﺒﻖ ﻋﻠﻰ ﻛﻼ اﻟﻘﺎﻋﺪﺗﯿﻦ‬٣٠ ‫ ﺑﺲ ھﻲ‬،‫ او اﻗﻞ ﻛﺎن ﺳﻮﯾﻨﺎ ﺳﯿﺮﻛﻼج‬٢٥ ‫وﻟﻮ ﻛﺎﻧﺖ ﺑﺎﻻﺗﺮاﺳﺎوﻧﺪ‬

Which of the following is the indication of cervical cercalge


A- cervical length less than 30
B- Cervical length less than 35
C- cervical length less than 25
D- cervical length less than 20

18 weeks pregnant with cervical incompetence, history of previous fetal passage at 28 weeks. What is the
management?
A. Cervical cerclage
B. OCPs
C. Follow up by serial us visits

Note to Remember
I would go for Cervical cerclage if:
-> Her cervix now is <25mm, and now she is at <24 weeks of gestation (US indicated cerclage)
OR
-> She has ahistory or preterm birth, and now she is at 13 or 14 weeks of gestation (History indicated cerclage)

According to UTD:
For women with a singleton pregnancy and a history of prior spontaneous preterm birth, we begin TVUS cervical
length screening at 14 to 16 weeks of gestation, and if her Cervical length is:
- >25mm-> we perform serial examinations.
- <25mm-> Cervical cerclage placement before 24 weeks of gestation

By: Wafa AlSalem 52


Pregnant in 8 weak gastation had misscarrage in previous prgnancy in 20 weak what should you do at this
pregnancy
A-Close antenatal follow up
B-Cerclage at 16 18 weak

Pregnant at 13 weeks of gestation with history or spontaneous fetal loss at 20 week. What is the most appropriate
action to do?
A- Regular F/U without specific intervention
B- Cervical cerclage now

According to UTD:
For women with a singleton pregnancy and a history of prior spontaneous preterm birth, we begin TVUS cervical
length screening at 14 to 16 weeks of gestation, and if her Cervical length is:
- >25mm-> we perform serial examinations.
- <25mm-> Cervical cerclage placement before 24 weeks of gestation

According to ACOG:
Indications for Cervical Cerclage in Women With Singleton Pregnancies
History Indicated Cerclage:
• History of one or more second-trimester pregnancy losses related to painless cervical dilation and in the
absence of labor or abruptio placentae
• Prior cerclage due to painless cervical dilation in the second trimester

Placed at approximately 13–14 weeks of gestation.

According to Williams Obstetrics:


**For women with an unequivocal history of second-trimester painless delivery, prophylactic cerclage placement is
an option and reinforces a weak cervix by an encircling suture.

**Cervical length screening is now recommended by both the American College of Obstetricians and Gynecologists
and the Society for Maternal-Fetal Medicine for women with prior preterm birth. Between 16 and 24 weeks'
gestation, sonographic cervical measurement is completed every 2 weeks.
- If an initial or subsequent cervical length is 25 to 29 mmà then a weekly interval is considered.
- If the cervical length measures <25 mmà cerclage is offered to this group of women.

!١٤ ‫ﯾﻌﻨﻲ زﺑﺪه اﻟﻜﻼم ان ال ھﺴﺘﻮري اﻧﺪﯾﻜﯿﺘﺪ ﺳﯿﺮﻛﻼج ﺻﺢ ﺑﺲ اﻟﺮﯾﻜﻮﻣﻨﺪﯾﺸﻦ ﺗﻘﻮل اﺣﺴﻦ ﻟﻮ اﻓﺤﺼﮭﺎ ﺑﺎﻻﺳﺒﻮع ال‬
‫ ﻓﺮاح اﺧﺘﺎر اﻟﺴﯿﺮﻛﻼج اﻟﺤﯿﻦ‬،‫ ﺑﺎل اﻟﺘﺮاﺳﻮاﻧﺪ اﺷﻮف اﻟﻘﯿﺎس‬١٤ ‫ﻓﻠﻮ ﻣﺎﻟﻘﯿﺖ اﻧﻲ اﻓﺤﺼﮭﺎ ﺑﺎﻻﺳﺒﻮع ال‬

By: Wafa AlSalem 53


Hypertension in Pregnancy and Preeclampsia
Topic Overview:
Criteria for Hypertension in Pregnancy
Defined as systolic blood pressure ≥140 mmHg and/or diastolic blood pressure ≥90 mmHg.
Severe hypertension is defined as systolic blood pressure ≥160 mmHg and/or diastolic blood pressure ≥110 mmHg.
Chronic Hypertension in Pregnancy
Defined as hypertension before the 20th week of gestation or persists longer than 12 weeks postpartum.
Gestational Hypertension
Pregnancy-induced hypertension with onset after 20 weeks' gestation without proteinuria or end-organ dysfunction
Preeclampsia
New onset of hypertension with proteinuria ( ≥0.3 g or protein/creatinine ratio ≥0.3 (mg/mg)) or end-organ
dysfunction after 20 weeks of gestation

Risk factors
⁃ Nulliparity
⁃ Multifetal gestations
⁃ Preeclampsia in a previous pregnancy
⁃ Chronic hypertension
⁃ Pregestational diabetes
⁃ Gestational diabetes
⁃ Thrombophilia
⁃ Systemic lupus erythematosus
⁃ Prepregnancy body mass index greater than 30
⁃ Antiphospholipid antibody syndrome
⁃ Maternal age ≥35 years or <18 years
⁃ Kidney disease
⁃ Assisted reproductive technology (IVF)
⁃ Obstructive sleep apnea
⁃ Obesity (BMI≥30)
⁃ Hydatidiform mole
⁃ Family history of preeclampsia
Prevention:
Women with any of the high-risk factors for And those with more than one of the moderate-risk
preeclampsia: factors:
- Previous pregnancy with preeclampsia - First pregnancy
- Multifetal gestation - Maternal age of 35 years or older
- Renal disease - Body mass index BMI of more than 30
- Autoimmune disease - Family history of preeclampsia (mother or sister)
- Type 1 or type 2 diabetes mellitus - Sociodemographic characteristics, and
- Chronic hypertension - Personal history
Should receive low-dose (81 mg/day) aspirin for preeclampsia prophylaxis initiated between 12 weeks and 28
weeks of gestation (optimally before 16 weeks of gestation) and continuing until delivery
Management
o <37 weeks of gestation:
Expectant management + oral labetalol or nifedipine
o ≥37 weeks of gestation
IOL + oral labetalol or nifedipine
o ≥34 weeks of gestation with pretem labor or PPROM
IOL + oral labetalol or nifedipine
• Preeclampsia with severe features
By: Wafa AlSalem 54
The presence of one or more of the following indicates a diagnosis of "preeclampsia with severe
feature”:
Severe blood Systolic blood pressure ≥160 mmHg or diastolic blood pressure ≥110 mmHg on 2 occasions at least 4
pressure elevation hours apart while the patient is on bedrest
Symptoms of New-onset cerebral or visual disturbance, such as:
central nervous § Photopsia, scotomata, cortical blindness, retinal vasospasm
system dysfunction § Severe headache (ie, incapacitating, "the worst headache I've ever had") or headache
that persists and progresses despite analgesic therapy(acetaminophen) and not accounted
for by alternative diagnoses

Hepatic Impaired liver function not accounted for by another diagnosis and characterized by serum
abnormality transaminase concentration >2 times the upper limit of the normal range or severe persistent right
upper quadrant or epigastric pain unresponsive to medication and not accounted for by an
alternative diagnosis, or both
Thrombocytopenia <100,000 platelets/microL
Renal abnormality Renal insufficiency (serum creatinine >1.1 mg/dL [97.2 micromol/L] or a doubling of the serum
creatinine concentration in the absence of other renal disease)
Pulmonary edema The symptom complex of dyspnea, chest pain, and/or decreased (≤93 percent) oxygen saturation
Management:
o <34 weeks of gestation:
Expectant management
Admission + Corticosteroid + Magnesium sulfate (seizure prophylaxis) + IV Labetalol
**During Expectant management urgent delivery after maternal stabilization is indicated in the following conditions
irrespective of gestational age:
Maternal Fetal
§ Uncontrolled severe-range blood pressures (persistent § Abnormal fetal testing
systolic blood pressure 160 mm Hg or more or diastolic § Fetal death
blood pressure 110 mm Hg or more not responsive to § Fetus without expectation for survival
antihypertensive medication) at the time of maternal diagnosis (eg,
§ Persistent headaches, refractory to treatment lethal anomaly, extreme prematurity)
§ Epigastric pain or right upper pain unresponsive to repeat § Persistent reversed end-diastolic flow
analgesics in the umbilical artery
§ Visual disturbances, motor deficit or altered sensorium
§ Stroke
§ Myocardial infarction
§ HELLP syndrome
§ New or worsening renal dysfunction (serum creatinine
greater than 1.1 mg/dL or twice baseline) c Pulmonary
edema
§ Eclampsia
§ Suspected acute placental abruption or vaginal bleeding in
the absence of placenta previa

o ≥34 weeks of gestation


IOL after stabilizing the mother + Magnesium sulfate (seizure prophylaxis) + IV Labetalol

• Superimposed Hypertension/Preeclampsia
By: Wafa AlSalem 55
Preeclampsia that occurs in a patient with chronic hypertension

• HELLP Syndrome
A life-threatening form of preeclampsia characterized by Hemolysis, Elevated Liver enzymes,
and Low Platelets
To make the diagnosis:
- Lactate dehydrogenase (LDH) elevated to 600 IU/L or more,
- Aspartate aminotransferase (AST) and alanine aminotransferase (ALT) elevated more than twice
the upper limit of normal
-Platelets count less than 100,000
Clinical presentation:
- Right upper quadrant pain and generalized malaise (90%)
- Nausea and vomiting (50%)
Management
Women with HELLP syndrome should be delivered regardless of their gestational age

Eclampsia
Severe form of preeclampsia with convulsive seizures and/or coma (in the absence of other neurologic conditions
that could account for the seizure)
Management:
⁃ ABC: calling for help, prevention of maternal injury, placement in lateral decubitus position, prevention of
aspiration, administration of oxygen, and monitoring vital signs including oxygen saturation.
⁃ Anticonvulsive Therapy:
• Magnesium sulfate: (Most eclamptic seizures are self-limited. Magnesium sulfate is not necessary
to arrest the seizure but to prevent recurrent convulsions.)
Dose: [IV] administration of a 4–6 g loading dose over 20–30 minutes, followed by a maintenance
dose of 1–2 g/hour
Side effects: Deep tendon reflexes are lost, warmth and flushing, respiratory depression, and
cardiac arrest
Monitoring: respiration status, tendon reflexes and measuring urine output (because magnesium
sulfate is excreted almost exclusively in the urine)
When toxicity occurs? The infusion should be stopped emergency correction with calcium gluconate
10% solution, 10 mL IV over 3 minutes, along with furosemide intravenously to accelerate the rate
of urinary excretion.
• Diazepine and phenytoin are justified only in the context of antiepileptic treatment or when
magnesium sulfate is contraindicated or unavailable (myasthenia gravis, hypocalcemia, moderate-to-
severe renal failure, cardiac ischemia, heart block, or myocarditis).

- Delivery! after maternal hemodynamic stabilization

Pregnant lady 39W her routine BP throughout the pregnance was 120/80 (normal) then suddenly became 150/90
what is the diagnosis?
By: Wafa AlSalem 56
A. Eclampsia
B. Gestational hypertension
C. Chronic hypertension
D. Superimposed hypertension

Note to Remember
- Preeclampsia à new-onset gestational hypertension with proteinuria or end-organ dysfunction
- Eclampsia à severe form of preeclampsia with convulsive seizures
- Gestational hypertension à onset after 20 weeks' gestation without proteinuria or end-organ dysfunction
- Chronic hypertensionà < 20 weeks' gestation or before pregnancy
- Superimposed hypertension à chronic hypertension with superimposed preeclampsia

First line treatment of hypertension in pregnancy is?


2- Methyldopa
3- Labetolol
4- Hydralazine
5- Nifdepine

Pregnant female with Hypertension 140/90, no proteinuria, what is first line mx?
A- Methyldopa
B- Labetalol
C- Nifedipine
D- Hydralazine

Management of chronic hypertension in pregnancy?


A- Methyldopa
B- Labetalol
C- Nifedipine
D- Hydralazine

Note to Remember

According to ACOG :
For chronic maintenance treatment, oral labetalol (first line) or nifedipine (second line) are reasonable options and
are recommended above all other antihypertensive drugs. Methyldopa is generally less favored.

By: Wafa AlSalem 57


Pregnant in 20w with HTN 160/90 what is the appropriate antihypertensive drug for her?
A- Labetalol
B- Hydralazine
C- Methyldopa
D- Nifedipine

Severe preeclampsia, acute management of HTN drug?


A. hydralazine
B. Methyldopa
C. Nifedipine
D. Sodium nitroprusside

Note to Remember
Antihypertensive agents for urgent blood pressure control in pregnancy:
- IV labetalol (first line)
- IV Hydralazine (second line)

Pregnant lady had seizure and is unconscious, her baby is healthy, what to do?
A- mgSo4
B- Establish airway
C- Fluids
D-Urgent delivery

Note to Remember
Management of Eclampsia:
⁃ ABC: calling for help, prevention of maternal injury, placement in lateral decubitus position, prevention of
aspiration, administration of oxygen, and monitoring vital signs including oxygen saturation.
⁃ Anticonvulsive Therapy:
• Magnesium sulfate
- Delivery! after maternal hemodynamic stabilization
A 23-year-old primigravida presented at 32-weeks of gestation with seizure
(see lab results),

By: Wafa AlSalem 58


Blood pressure 160/110 mmHg
Heart rate 78 /min
Respiratory rate 18 /min
Temperature 36.6°C
Test :Protein +++'
Which of the following is the most appropriate next step in management?
A) steroids
B) diuretics
C) hydralazine
D) magnesium sulfate

Pregnant at 34 weeks with blurred vision, headache and her BP 170/ What to do ?
A. stabilize + mg and wait till 37 weeks
B. call anaesthesia now and deliver
C. stabilize and give MG and deliver

Note to Remember
Management of Preeclampsia without severe features
o <37 weeks of gestation:
Expectant management + oral labetalol or nifedipine
o ≥37 weeks of gestation
IOL + oral labetalol or nifedipine
o ≥34 weeks of gestation with pretem labor or PPROM
IOL + oral labetalol or nifedipine
Management of Preeclampsia with severe features
o <34 weeks of gestation:
Expectant management
Admission + Corticosteroid + Magnesium sulfate (seizure prophylaxis) + IV Labetalol
o ≥34 weeks of gestation
IOL after stabilizing the mother + Magnesium sulfate (seizure prophylaxis) + IV Labetalol

28 WEEKS PREGNANT (NUU), PRESENTS WITH GENERALIZED FATIGUS BLOOD PRESSSURE:162, PROTIN IN
URINE3+, WHAT IS YOUR NEXT STEP?
A. MGSO4
B. LABETALOL
C. METHYLDOPA

Management of Preeclampsia with severe features


o <34 weeks of gestation:
Expectant management
Admission + Corticosteroid + Magnesium sulfate (seizure prophylaxis) + IV Labetalol
o ≥34 weeks of gestation
IOL after stabilizing the mother + Magnesium sulfate (seizure prophylaxis) + IV Labetalol

First thing to do is admission, if it wasn’t in the choices, I would go with magnesium to prevent seizure first.

By: Wafa AlSalem 59


Primigravida 32 came with mild headache with no abdominal pain, or visual disturbance, Blood pressure 150/90,
urine analysis +3 protien, appropriate management?
A. Close outpatient after 1 week
B. Admission and observe
C. induction of labor
D. C/S

She has preeclampsia without severe features which is managed by outpatient close monitoring at least once
weekly (ACOG)

Pregnant present in 38 weeks in labor her BP 150/90 and elevated proteins /creatinine ratio. What is the
diagnosis?
A. Preeclampsia
B. Chronic hypertension
C. Gestational hypertension
D. Superimposed hypertension

Preeclampsia
New onset of hypertension with proteinuria ( ≥0.3 g or protein/creatinine ratio ≥0.3 (mg/mg)) or end-organ
dysfunction after 20 weeks of gestation

A female pregnant with hypertension and proteinuria, she has right upper quadrant pain what is the reason?
A. Distended Hepatic Capsule
B. Hepatic Rupture
C. Gall Bladder Stone
By: Wafa AlSalem 60
According to ACOG: Pain is thought to be due to periportal and focal parenchymal necrosis, hepatic cell
edema, or Glisson’s capsule distension, or a combination.

A pregnant woman came with abdominal pain and back pain and visual disturbance. Her blood pressure is 145/92
mmHg. Her lab shows (uric acid high, platelet 70k). What is a severe feature of preeclampsia for this condition?
a. Her abdominal pain
b. Her blood pressure
c. Platelet count
d. uric acid

The presence of one or more of the following indicates a diagnosis of "preeclampsia with severe
feature”:
Severe blood Systolic blood pressure ≥160 mmHg or diastolic blood pressure ≥110 mmHg on 2 occasions at least 4
pressure elevation hours apart while the patient is on bedrest
Symptoms of New-onset cerebral or visual disturbance, such as:
central nervous § Photopsia, scotomata, cortical blindness, retinal vasospasm
system dysfunction § Severe headache (ie, incapacitating, "the worst headache I've ever had") or headache
that persists and progresses despite analgesic therapy(acetaminophen) and not accounted
for by alternative diagnoses

Hepatic Impaired liver function not accounted for by another diagnosis and characterized by serum
abnormality transaminase concentration >2 times the upper limit of the normal range or severe persistent right
upper quadrant or epigastric pain unresponsive to medication and not accounted for by an
alternative diagnosis, or both
Thrombocytopenia <100,000 platelets/microL
Renal abnormality Renal insufficiency (serum creatinine >1.1 mg/dL [97.2 micromol/L] or a doubling of the serum
creatinine concentration in the absence of other renal disease)
Pulmonary edema The symptom complex of dyspnea, chest pain, and/or decreased (≤93 percent) oxygen saturation
Let’s Exclude!!
- Her abdominal pain -> if it was unresponsive to analgesia (indicates severe preeclampsia)
- Her blood pressure -> if it was ≥160/110 (indicates severe preeclampsia)
- Uric acid?? Not one of the criteria of severe preeclampsia

Pregnant, now bp 140/90, platelets 90k, hx of previously severe preeclampsia. What indication here of severe
preeclampsia?
A. BP
B. platelets
C. uterine contraction

Pregnant at 12 weeks, complaining of mild leg edema, blood pressure mildly elevated, Positive trace proteinuria
A- Methyldopa
B- Labetalol
C- losartan
D-Captopril

This is chronic hypertension which is treated by labetalol as a first line.


(Approximately 11% of women with chronic hypertension have proteinuria (more than 300 mg/day) at baseline
because of hypertension- related nephrosclerosis or, less frequently, undiagnosed chronic kidney disease)

By: Wafa AlSalem 61


Female 36 year, at 15w GA came with hypertension 180/110 no proteinuria, what is the diagnosis?
A. Primary HTN.
B. Pregnancy induced HTN.
C. white coat syndrome.
D. Eclampsia

Female 36 year, at 15w GA complaining of headache, blurred vision since 2 weeks ago ,with hypertension , what is
the diagnosis?
A. Primary HTN.
B. Pregnancy induced HTN.
C. white coat syndrome.
D. Eclampsia

Pregnant at 39 weeks, came with proteinuria Bp 140/90, what is your action?


A. admit and observation
B. Labor induction
C. Follow up and observe

Pregnant with 33-week gestation presented with severe headache protein urine +3, Bp 150/100, asking what to do?
A. Immediate C/S.
B. delay for a week and give steroid.
C. Admitted for observation

She has preeclampsia with severe features which is managed by:


<34 weeks = Admission + Corticosteroid + Magnesium sulfate (seizure prophylaxis) + IV Labetalol

Pregnant 34 week with preeclampsia, high BP 170 /100 what to give ?


A. Stabilize, MgSo4 and deliver
B. Stabilize, corticosteroids and deliver

She has preeclampsia with severe features which is managed by:


≥34 weeks = IOL after stabilizing the mother + Magnesium sulfate (seizure prophylaxis) + IV Labetalol

Pregnant 35w hypertensive came with sever headache, abdominal pain and feel dizzy
A- Give mg salfate and admission for delivery
B- Give steroid and admission for delivery
C- Give mg salfate and wait

She has preeclampsia with severe features which is managed by:


≥34 weeks = IOL after stabilizing the mother + Magnesium sulfate (seizure prophylaxis) + IV Labetalol

By: Wafa AlSalem 62


Pregnant 37 week with BP 160/110 every thing else was normal Her BP during pregnancy was 120/90
A. Eclampsia
B. Gestational HTN
C. Preeclampsia

Extra note:
She has gestational hypertension which is managed by:
≥37 weeks of gestation = IOL + oral labetalol or nifedipine

Pregnant 34 weeks, vaginal bleeding open cervix 6cm, she has hypertension and proteinuria, CTG shows fetal
bradycardia, what is the management?
A. MgSo4 and deliver
B.stabilize and mgso wait until 37 week
C.stabilize give steroid then labour

She has preeclampsia without severe features which is managed by:


≥34 weeks of gestation with pretem labor or PPROM= IOL + oral labetalol or nifedipine
١١٠/١٦٠ ‫ ﺑﺲ ﺷﻜﻞ اﻟﺮ ﻳﻜﻮل ﺿﻌﻴﻒ وﻛﺎن ﺿﻐﻄﻬﺎ ﻋﺎﻟﻲ‬،‫ ﻣﻔﺮوض ﻣﺎﻧﻌﻄﻴﻬﺎ ﻣﺎﻏﻨﻴﺴﻴﻮم‬severe features ‫ﻣﺪام ﻣﺎﻋﻨﺪﻫﺎ‬

34 wk preeclamsia pt came with epigastric pain, headache, blurred vision .. non stress test reassuring mx ?
a) Mg sulf + delivary
b) mg and wait for 37 wk
c) Call anesthesiologists for deliver

She has preeclampsia with severe features which is managed by:


≥34 weeks = IOL after stabilizing the mother + Magnesium sulfate (seizure prophylaxis) + IV Labetalol

Magnisium sulfate given to hypertensive pregnant to?


A. Prevent the seizure attack
B. For fetal neuroprotection
C. Reduce her BP

Pregnant with preeclampsia at 36 weeks, developed seizure. After stabilization you should give Mg Sulfate to?
A. Manage her seizure
B. Prevent the seizure attack
C. For fetal neuroprotection

Note to Remember
Most eclamptic seizures are self-limited. Magnesium sulfate is not necessary to arrest the seizure but to prevent
recurrent convulsions

Preganat c/o sever abdominal pain and uterine cotractions she was given 6mg Mg sulfate and the e contractions
become normal then decrease the dose to 4mg, then she complained of shortness of breath. What to do?
A. Give her oxygen
By: Wafa AlSalem 63
B. Change to left lateral position
C. Stop Mg sulfate and give Ca gluconate

Pregnant with seizure given 6 mg sulphate then decrease to 4mg. On Examination there is absent deep tendon
reflex, what to do?
A. Reassurance
B. Re-increase Mg sulfate dose
C. Stop Mg sulfate and give Ca gluconate

Note to Remember
- Magnesium toxicity -> Deep tendon reflexes are lost, respiratory depression, and cardiac arrest
- When toxicity occurs? The infusion should be stopped emergency correction with calcium gluconate

Patient on oxytocin, epidural and MgS04, preeclampsia. Her CTG: absence variability (or non-reactive). What's the
cause?
A. MgS04 toxicity
B. Oxytocin
C. Epidural analgesia

Explained in the CTG section.

Patient at 32 weeks presented with seizure and high blood pressure, what is the next appropriate?
A. steroid
B. hydralazine
C. magnesium sulfate
D. Abx

What is the medication for seizure in pregnancy?


a- Diazepine
b- Phenytoin
c- Magnesium sulfate
d- Lorazepam

Note to Remember
Diazepine and phenytoin are justified only in the context of antiepileptic treatment or when magnesium sulfate is
contraindicated or unavailable (myasthenia gravis, hypocalcemia, moderate-to-severe renal failure, cardiac
ischemia, heart block, or myocarditis).

Risk factor for preeclampsia


A) Maternal age

B) Gestational age

C) Multiple pregnancy

Note to Remember
Risk factors for Preeclampsia (ACOG)
⁃ Nulliparity
⁃ Multifetal gestations

By: Wafa AlSalem 64


⁃ Preeclampsia in a previous pregnancy
⁃ Chronic hypertension
⁃ Pregestational diabetes
⁃ Gestational diabetes
⁃ Thrombophilia
⁃ Systemic lupus erythematosus
⁃ Prepregnancy body mass index greater than 30
⁃ Antiphospholipid antibody syndrome
⁃ Maternal age ≥35 years or <18 years
⁃ Kidney disease
⁃ Assisted reproductive technology (IVF)
⁃ Obstructive sleep apnea
⁃ Obesity (BMI≥30)
⁃ Hydatidiform mole
⁃ Family history of preeclampsia

So, the Maternal Age is a risk factor for preeclampsia only if ≥35 years or <18 years
‫ﻓﻠﻤﺎ ﻣﺎﯾﺤﺪدﻟﻲ ﻋﻤﺮ ﻣﺎراح اﺧﺘﺎره‬

Preeclampsia what expected to be significantly decrease ?


1- Urea
2- Creatinine
3- Plasma volume
4- Platelets

Let’s Exclude!!
- Uric acid -> Serum uric acid increases with preeclampsia
- Creatinine -> it may increase with preeclampsia not decrease
- Plasma volume?? -> it increases in pregnancy in general
- Thrombocytopenia -> may occur and may reach severe levels as part of HELLP

Primigravida at 32 weeks GA with BP of 150/90 mmHg. There is edema of hands and legs. What to do?
A. Diuretics
B. Tabs Labetalol
C. Continued evaluation

The main goals in the initial evaluation of pregnant women with newly developed hypertension are to distinguish
gestational hypertension from preeclampsia. I would evaluate her more first with urinary protein excretion and
look for features of severe disease, then I will manage accordingly.

Preeclampsia scenario. What you ordered?


A. LFT
B. Platelet
C. Urinalysis

By: Wafa AlSalem 65


First thing is to do urine analysis to distinguish gestational hypertension from preeclampsia

Which of these causes IUGR?


A. Oligohydramnios
B. Polyhydramnios
C. Preeclampsia

Women with HTN what do you expect the complications?


A. IUGR
B. Abnormal placental lining

Patient hypertensive with proteinuria on labetalol she’s 32 weeks, fundal height 28. what will commonly occur with
IUGR?
a. oligohydramnios
b. polyhydramnios

According to ACOG:
Among women with preeclampsia, clinical manifestations that follow from this uteroplacental ischemia include:
- Fetal growth restriction(IUGR)
- Oligohydramnios
- Placental abruption
- Nonreassuring fetal status
- Increased risk of spontaneous or indicated preterm delivery.

What of the following decreases the risk of preeclampsia? (patient had hx of preeclampsia in her previous
pregnancy)
A. Antibiotic
B. Aspirin
C. MgSo

According to ACOG
Women with any of the high-risk factors for preeclampsia (previous pregnancy with preeclampsia, multifetal
gestation, renal disease, autoimmune disease, type 1 or type 2 diabetes mellitus, and chronic hypertension) and
those with more than one of the moderate-risk factors (first pregnancy, maternal age of 35 years or older, a body
mass index BMI of more than 30, family history of preeclampsia, sociodemographic characteristics, and personal
history factors) should receive low-dose (81 mg/day) aspirin for preeclampsia prophylaxis initiated between 12
weeks and 28 weeks of gestation (optimally before 16 weeks of gestation) and continuing until delivery

What of the following decreases the risk of eclampsia?


A. Antibiotic
B. Aspirin
C. MgSo4

According to ACOG
Magnesium sulfate is used to prevent seizures in women with preeclampsia with severe features and eclampsia
By: Wafa AlSalem 66
- Prevention of preeclampsia -> Aspirin
- Prevention of eclampsia -> Mgso

A women known chronic hypertension came to prenatal care counseling with expected pregnancy, she is on
hydrochlorothiazide and lisinopril?
A. Stop both
B. Stop lisinopril and start methyldopa
C. Continue both
D. Stop ACEI and continue thiazide

According to Williams Obstetrics:

- For most women with mild to moderate hypertension, the College recommends that treatment be withheld as
long as systolic blood pressure is < 1 60 mm Hg and diastolic blood pressure is < 105 mm Hg.
- It is controversial whether or not women who present early in pregnancy and who are already taking
antihypertensive drugs should continue to take these (Rezk, 2016).
- According to the American College of Obstetricians and Gynecologists (201 3) and the Society for Maternal-
Fetal Medicine (201 5), for women with mild to moderate hypertension, it is reasonable² to discontinue
medications during the first trimester and to restart them if blood pressures approach the severe range.
- Our practice at Parkland Hospital is to continue treatment if the woman is already taking drugs when she
presents for prenatal care. Exceptions are discontinuation of angiotensin-converting enzyme inhibitors and
receptor blockers.

‫دو ﻳﻪ ﻟﻴﻦ ﻳﺰ ﻳﺪ اﻟﻀﻐﻂ‬MZ‫ ﺗﻘﻮل ان ﻣﺪام اﻟﻀﻐﻂ ﻣﻮ ﺧﻄﻴﺮ اﻓﻀﻞ ﻧﻮﻗﻒ ﻛﻞ ا‬ACOG ‫ل اﻟﺤﻤﻞ ﺑﺲ‬ML‫م ﻳﻘﻮل ان اﻟﺜﻴﺎزاﻳﺪ آﻣﻦ ﺧ‬ML‫زﺑﺪه اﻟﻜ‬
‫ ﻓﻮﺟﻮده وﻋﺪﻣﻪ واﺣﺪ‬preeclampsia ‫ ﻣﺎﻳﺤﻤﻴﻬﺎ ﻣﻦ ال‬thiazide ‫ن أﺻﻼ ال‬MZ severe ‫و ﻳﺼﻴﺮ‬
.‫ ﻓﻴﻌﻨﻲ ﻟﻮ ﺟﺖ ﻗﺒﻞ اﻟﺤﻤﻞ ﺑﻴﻘﻮﻟﻮﻧﻠﻬﺎ وﻗﻔﻲ‬،‫ص ﺑﺨﻠﻴﻬﺎ ﺗﻜﻤﻞ‬ML‫واﺧﺮ ﺟﻤﻠﻪ ﺗﻘﻮل اﻧﻬﺎ ﻟﻮ ﺟﺘﻨﺎ وﻫﻲ اﻟﺮ ﻳﺪي ﺣﺎﻣﻞ وﻫﻲ ﻟﺴﺎ ﺗﺄﺧﺬ ادو ﻳﺘﻬﺎ ﺧ‬
!!‫ﻳﻦ ﺣﻖ اﻟﻀﻐﻂ ﺑﺎﻟﺤﻤﻞ‬MZ ‫ ﻣﻮ اﻟﺪواء اﻟﻔﺮﺳﺖ‬methyldopa ML‫واﺻ‬

By: Wafa AlSalem 67


Diabetes Mellitus and Gestational Diabetes Mellitus
Topic Overview:

Patient with long term history of DM type 1 in 12 weeks of gestation. HbA1C 12. Which of the following
complication is most likely to happen?
A. Preeclampsia
B. Polyhydroamnios
C. Congenital malformation
D. IUGR

According to ACOG:
In the second and third trimesters, an HbA1C less than 6% has the lowest risk of large-for-gestational-age
infants. Importantly, because of the association of elevated glucose values and congenital anomalies, aggressive
approaches to glycemic control early in the first trimester before or during embryogenesis may reduce the risk of
fetal anomalies.

Extra note:
Women with true GDM are not at increased risk of having an infant with congenital malformations because the
onset of the disorder is after organogenesis, and they do not experience diabetes-related vasculopathy because of
the short duration of the disorder.

Gestational diabetes most associated with?


A.Preeclampsia
B.Polyhydromnis
C. IUGR Intrauterine growth restriction

Note to Remember
Women with GDM have a higher risk of developing preeclampsia
GDM complications most commonly is macrosomia-> then HTN and preeclampsia

Patient diabetic and pregnant what's the most useful test that will determine prognosis for the baby?
A. OGTT
B. fasting blood glucose
C. glycosylated hemoglobin
D. US nuchal translucency

According to ACOG:
Glycosylated hemoglobin levels correlate directly with the frequency of anomalies.

Diabetic mother came at 10 weeks what test will tell you the risk of chromosomal anomalies?
A. glycosylated hemoglobin
B. US nuchal translucency

Nuchal translucency scan is a prenatal screening scan to detect chromosomal abnormalities in a fetus. Done by
ultrasound between 10+3 and 13+6 gestational weeks.

By: Wafa AlSalem 68


36 yo female diabetic on metformin, delivered a baby with congenital anomalies, she wants to conceive?
A- switch to insulin
B- Genetic counseling
C- Folic acid 400 mcg

According to UTD:
Genetic counseling referral is indicated if the personal or family history includes a confirmed clinical diagnosis with
a known genetic etiology such as hemophilia, neurofibromatosis, or Marfan syndrome

According to ACOG:
At least 400 micrograms of folic acid should be prescribed to all women contemplating pregnancy. This is
particularly important in women with diabetes given their increased risk of neural tube defects.

Female hypertensive and diabetic, on ACEI, insulin, Metformin, she decided to get pregnant soon. Labs: heavy
proteinuria, Hga1c: 8, What’s your most appropriate advice for her regarding diabetic control and fetal congenital
malformations?
A. stop snacks and start 3 heavy meals a day.
B. switch ACEI to ARB’s
C. try to control her hga1c to normal or near normal as possible before pregnancy.
D. Increase dose of ACEI

According to ACOG:
Prepregnancy counseling should focus on the importance of euglycemic control before pregnancy, as well as the
adverse obstetric and maternal outcomes that can result from poorly controlled diabetes. The specific risk of
fetal embryopathy related to glycemic control is an important outcome to counsel patients about in order to
emphasize the importance of prepregnancy glycemic control (HbA1c less than 6%).

Pregnant and diabetes and hypoglycemic in ER and has to given glucose what the route that will give her and don’t
harm the baby?
A. Nasogastric
B. Orogastric
C. Peripheral venous
D. Central venous

According to ACOG:
If patients become hypoglycemic, intravenous dextrose should be given and the insulin infusion rate reduced.

At what gestational age we start diabetes screening in a pregnant woman?


A. 4 weeks
B. 12 weeks
C. 24 weeks
D. 34 weeks

By: Wafa AlSalem 69


According to ACOG:
U.S. Preventive Services Task Force made a recommendation to screen all pregnant women for GDM at or beyond
24 weeks of gestation.

Pregnant at 22 weeks gestational age oral glucose challenge test after one hour: high, after 2 hours: high, after
three hours: high. What is next?
A- Repeat same test
B- HgA1c
C- Fasting blood glucose
D- Random blood glucose

According to ACOG:
American Diabetes Association (ADA) has noted that measurement of hemoglobin A1C also can be used for
detecting pregestational diabetes or early GDM, but it may not be suitable for use alone because of decreased
sensitivity compared with OGTT approaches.

Let’s Exclude!!
- Repeat same test-> why would I repeat it if the test is considered positive if two values are greater than
established thresholds?
- Fasting blood glucose-> it supposed to be done before the 1,2,3 hours ‫أﺻﻼ‬
- Random blood glucose-> why? Useless in pregnancy

Pregnancy Uncontrolled diabetes Complications for baby after delivery


1-RDS
2-Hyperkalemia

According to ACOG:
The neonatal consequences of poorly controlled pregestational diabetes mellitus during pregnancy include profound
hypoglycemia, a higher rate of respiratory distress syndrome, polycythemia, organomegaly, electrolyte
disturbances, and hyperbilirubinemia.

Pregnant lady with one abnormal GTT reading (after 1 hr)


A. Repeat GTT
B. Lifestyle modification
C. Measure hg A1c

Serum glucose concentrations are then measured after 1, 2, and 3 hours. The test is considered positive if two
values are greater than established thresholds.

I would choose lifestyle modification if she has 2 abnormal results not only one.

By: Wafa AlSalem 70


What is the pathophysiology of GDM?
A. Decrease uteroplacental blood flow during late pregnancy
B. Human placental lactogen (human chorionic somatomammotropin)

According to ACOG:
The increase in insulin resistance is primarily the result of the effects of several placental hormones, including
human chorionic somatomammotropin (human placental lactogen), progesterone, prolactin, placental growth
hormone, and cortisol. Additionally, tumor necrosis factor a, and leptin have been implicated as contributors to the
insulin resistant state of pregnancy and resultant maternal hyperglycemia

DM pregnant, what is the complication she has a similar chance of getting it as in normal pregnancy?
A- Preeclampsia
B- Cystic fibrosis
C- IUGR
D- Polyhydroamnios

According to ACOG:
- Women with GDM have a higher risk of developing preeclampsia
- Neonates delivered to women with pregestational diabetes are at increased risk of macrosomia and, depending
on concomitant risk factors, also may be at increased risk of fetal growth restriction
- Women with pregestational diabetes mellitus have a greater risk of a wide range of obstetric complications.
For these women, the rate of primary cesarean delivery is increased; spontaneous preterm labor appears to be
more common; and for some women— particularly those with poor glycemic control the increased incidence of
polyhydramnios

Newly diagnosed mother with GDM, what the first line management
A. Diet
B.metformin
C. Oral Insulin
D.SC insulin

According to ACOG:
- Women in whom GDM is diagnosed should receive nutrition and exercise counseling, and when this fails to
adequately control glucose levels, medication should be used for maternal and fetal benefit.
- When pharmacologic treatment of GDM is indicated, insulin is considered the preferred treatment for
diabetes in pregnancy.
- In women who decline insulin therapy or who the obstetricians or other obstetric care providers believe will be
unable to safely administer insulin, or for women who cannot afford insulin, metformin is a reasonable
alternative choice

By: Wafa AlSalem 71


Fetal Conditions and Abnormalities
Topic Overview:

Mother came for antenatal care and US shows week 32 reversed end diastolic blood flow:
A-follow up 2 week and reassess
B-Immediate delivery now
C-administer steroids 1 week and delivery
D-NST

Note to Remember
According to UTD:
The presence of REDV at any gestational age beyond 32 weeks should prompt consideration for immediate
delivery. This is supported by Society for Maternal-Fetal Medicine guidelines, which recommend intense fetal
surveillance of these fetuses and continuing expectant management until 32 weeks as long as fetal surveillance
remains reassuring

Pregnant lady on 30 weeks on antenatal care on U/S: finding fetus size decrease than before with oligohyromnios ,
doppler of umbilical artery find reversed diastolic flow mother denied any loss of fetus movement, what is the
appropriate next step:
1) non-stress test
2) serial us after one week
3) serial doppler for umbilical artery after two weeks
4) kick fetal chart

Pregnant at 33 weeks gestation has reversed flow of doppler artery of umbilical, what will you do?
A. Emergent CS
B. Give steroids and wait for 1 wk
C. Wait till 37wk

28 y/o female, pregnant 35 GA presenting with decreased fetal movement, CTG was reassuring with fetal HR 130,
then 1 hr later CTG showing good variability. What is the best management for her?
A) Observe for 24 hr.
B) Induction of labor
C) C/S

By: Wafa AlSalem 72


D) Discharge her with fetal kick chart

Note to Remember
According to ACOG:
- For a pregnant individual reporting decreased fetal movement after viability, one-time antenatal fetal
surveillance at the time the decreased movement is reported may be considered. These include fetal movement
assessment, nonstress test, contraction stress test, fetal biophysical profile, modified biophysical profile and
umbilical artery Doppler velocimetry. (NOT OBSERVATION)
- If the NST is reactive, we believe that ultrasound examination is a valuable additional tool for assessment of
pregnancies complicated by persistent DFM, and is reassuring for mothers.

If there’s a biophysical profile or US examination in the choices, I would go with it. But with these options?
I would go with D

Patient at 29 weeks, didn't feel fetal movement for 1 day, CTG was reactive, Biophysical profile was 8. What to do
next?
A. Steroid and repeat Biophysical profile after 24 hours
B. Repeat Biophysical profile at 1 week
C. IOL
D. Urgent CS

Note to Remember
Her CTG is normal and Biophysical profile 8 or 10 is normal.

- For women <37 weeks of gestation with persistent DFM and normal fetal evaluation-> nonstress testing and
ultrasound examination twice weekly is recommended

Pregnant lady 32 weeks GA is worried that her baby stopped moving. What is the next more appropriate step for
this case?
a. Non-stress test
b. Biophysical profile
c. Pelvic examination
d. Pelvic US

For reduced fetal movement:


Nonstress test (First)
Then-> biophysical profile (US)

Pregnant around 30 weeks who is a case of Rh alloimmunization, fetus was found to have anemia, Management?
A. Deliver immediately
B. Duplex
C. Fetal blood transfusion

According to ACOG:
- Delivery of the infant of an alloimmunized patient is a controversial subject, and literature on the subject is
limited. Standard treatment is to prolong the pregnancy until the fetus reaches a gestational age necessary for
survival. Intrauterine transfusion up to 36 weeks of gestation when intravascular transfusion is feasible in
order to limit neonatal morbidity. Delivery can then be accomplished between 37 and 38 weeks of gestation.

By: Wafa AlSalem 73


- Douplex-> why? He already gave me the diagnosis (anemia) the next step (after MCA doppler) would be
obtaining fetal blood by cordocentesis for hemoglobin determination.

‫وﺣﺘﻰ ھﻮ ﻣﺎﻋﻄﺎﻧﻲ ﻛﻢ اﻟﻤﻌﻄﯿﺎت وﻛﻢ ﺷﺎف ﻋﻠﻰ طﻮل ﻗﺎﻟﻚ اﻧﯿﻤﯿﺎ ورﯾﺤﻚ ﯾﻌﻨﻲ اﺑﺪأ ﺑﺎﻟﻤﺎﻧﺠﻤﻨﺖ‬

Mother who is Rh negative delivered a baby who is Rh + she was given Anti-D Ig 300 microg what does it cover ?
A) 15 ml of the whole fetal blood
B) 30 ml of the whole fetal blood
C) 10 ml of the whole fetal blood
D) 30 ml of Rh(D) positive fetal RBCs

According to ACOG
A prophylactic dose of 300 micrograms of anti-D immune globulin can prevent Rh D alloimmunization after exposure
to up to
- 30 mL of Rh D-positive fetal whole blood or 15 mL of fetal red blood cells

According to Williams Obstetrics


The dosage of anti-D immune globulin is calculated from the estimated volume of the fetal-to-maternal
hemorrhage
- One 300-µg dose is given for each 15 mL of fetal red cells or 30 mL of fetal whole blood to be neutralized.

By: Wafa AlSalem 74


Pregnancy Related Medical and Surgical Conditions
Topic Overview:

Female patient wants to get pregnant. TSH high, T4 normal. What is the most appropriate management?
A. Follow up
B. Give thyroxine now
C. Give thyroxine during pregnancy
D. Proceed to pregnancy without any management

Pregnant women with brown spot in her face?


A. Melasma
B.Chloasma

Cholasma: is the occurrence of melasma during pregnancy

Pregnant, C/O vomiting multiple times, dry mouth and oral thrush, decreased skin turgor, urinalysis ordered, what
finding in UA will confirm the diagnosis?
A. Urine leukocyte
B. Urine Proteins
C. Urine Ketones
D. Urine Glucose

Hyperemesis gravidarum -> Urine ketones

Physiological changes in pregnancy that can lead to heart failure in patient with mitral stenosis
A. Increase minute ventilation
B. Increase RBC mass
C. Increase renal plasma flow
D. Increase plasma volume

Note to Remember
According to hacker: During pregnancy, the mechanical obstruction associated with mitral stenosis worsens as
cardiac output increases.

According to UTD: In MS, the stenotic mitral valve restricts diastolic left ventricular filling, resulting in an
elevated transmitral gradient and left atrial pressure that are further increased by the physiologic hypervolemia
and increased heart rate during pregnancy, thereby increasing the risk of pulmonary congestion or pulmonary
edema

IN SUMMARY: mitral stenosis worsens with increased cardiac output! So what increases the cardiac output?
- Increased HR
By: Wafa AlSalem 75
- Increased blood volume (plasma volume+ packed RBC’s)
But!! Plasma volume increases way more than the packed RBC’s!! which causes physiological anemia..
So, I would go with increased plasma volume. If plasma volume is not in the choices? Go for Increased RBC ‫ﺑﺲ‬
‫اﻟﺒﻼزﻣﺎ ﻓﻮﻟﯿﻮم اﺻﺢ اﻛﯿﺪ وﺷﺮﺣﺖ اﻟﻤﻮﺿﻮع ﻛﺎﻣﻞ ﻋﺸﺎن ﺗﻔﮭﻤﻮن ﻣﻮ ﺑﺲ ﺗﺤﻔﻈﻮن‬

A pregnant woman presents with vague pain. US showing an ovarian cyst measuring 9 x 7 cm. How will you manage
this patient?

A. Reassurance
B. Laparoscopic drainage
C. Immediate laparotomy
D. Analgesia and Observation

Note to Remember

Adnexal Mass in Pregnancy: ACOG


Most adnexal masses in pregnancy appear to have a low risk of malignancy or acute complications and may be
managed expectantly. 51–92% of adnexal masses will resolve during pregnancy
Surgical intervention is warranted for:
⁃ Symptomatic masses (acute abdomen) “OUR PATIENT HAS NO ACTUE ABDOMEN”
⁃ Or for suspected malignancy based on the results of radiologic imaging (US finding suggest malignancy:
cyst greater than 10cm) “OUR PATIENT IS LESS THAN THAT”, serum marker testing, or both.

Let’s Exclude!!
⁃ Reassurance -> because it will resolve during pregnancy and no need for follow up!
⁃ Laparoscopic drainage -> if more than 10cm
⁃ Immediate laparotomy -> if ACUTE ABODMEN (sudden abdominal pain and tenderness with nausea and
vomiting), although exploratory laparoscopy is preferred
⁃ Analgesia and observation -> why would I observe her? What is my indication of observing her?

ACOG Reference for you to read:

By: Wafa AlSalem 76


By: Wafa AlSalem 77
Pregnancy Related Hematological Problems
Topic Overview:

Sickle cell anemia patient pregnant, antenatal complication associated with her condition?
A. Low birth weight (IUGR)
B. Chest infection
C. UTI

Williams Obstetrics:

ACOG:

By: Wafa AlSalem 78


Infections in pregnancy
Topic Overview:

Pregnant HBs positive what will u give the baby in first 12 hrs ?!
A- hep b vaccine + immunoglobulins
B -hep b vaccine only
C-immunoglobulins only

UTI in Pregnancy
Topic Overview:

By: Wafa AlSalem 79


Investigations and Screening Tests During Pregnancy
Topic Overview:

Pregnant of twins, one has increased nuchal translucency in Ultrasound. What will he have?
A. Congenital cardiac malformation (all chromosomal syndromes have cardiac disease)
B. Turner syndrome
C. Neural tube defect

How to know fetal weight intrapartum in 37w?


A- femur length
B- head circumflex
C-Biparietal diameter
D- abdominal circumference

GBS vaginal swab screening in pregnant women


a. 15 weeks
b. 25 weeks
c. 35 weeks
d. 40 weeks

Note to Remember
Routine antepartum GBS vagina and rectal cultures on all pregnant women at 35 to 37 weeks

Women deliver baby with Down syndrome, and she wants to know about future pregnancy. Which of the following is
BEST choice of her?
A. Karyotype of infant
B. Karyotype of infant and mother.
C. U/S in next pregnancy
D. Amniocentesis in next pregnancy

Note to Remember
Down syndrome prenatal screening:
- Combined test “US determination of nuchal translucency NT + determination of biochemical markers associated
with aneuploidy
Down syndrome definitive prenatal diagnosis:
- Chorionic villi sampling (CVS) or Amniocentesis

Pregnant who has a child with down syndrome. She’s concerned about having another child with down syndrome.
What is the best test to rule out down syndrome in the second trimester?
A-Amniotic fluid sample
B- Chorionic villous sample
C- Triple test

Note to Remember
Chorionic Villi Sampling (CVS): is the procedure of choice for first trimester testing (between 10-13 weeks.)

By: Wafa AlSalem 80


Amniocentesis: is the procedure of choice for second trimester testing optimally performed (ACOG: between 15-
20 weeks.), (Uptodate: between 15-17+6)

Best indicator of chromosomal abnormalities in which week?


A-16 to 18wk
B-18 to 22wk
C-13 to 16wk
D-24 to 28wk

Note to Remember
It looks like a difficult question, but ACTUALLY, IT IS VERY SIMPLE!!

What does the Question mean? “The correct timing of the DIAGNOSTIC test for chromosomal abnormalities”

So, what are the diagnostic tests? Chorionic Villi Sampling (CVS) and Amniocentesis!!
Chorionic Villi Sampling (CVS): is the procedure of choice for first trimester testing (between 10-13 weeks.)
Amniocentesis: is the procedure of choice for second trimester testing optimally performed (ACOG: between 15-
20 weeks.), (Uptodate: between 15-17+6)

12 Weeks pregnant, what will her blood test show?


A. Decrease in serum creatinine
B. Increase in plasma sodium
C. Increase in plasma BUN
D. Decrease in BUN

SAME questions different recall: (1) (2)


(1) Female pregnant, what of the following true regarding Elevated BhCG?
A. High BhCg indicator of ectopic pregnancy
B. High BhCg in second Trimester indicator of molar pregnancy
C. High BhCg in second Trimester is the most sensitive marker of Down syndrome
D. High Bhcg can cause depression of TSH

Another Recall,

(2) Female pregnant, what of the following is true regarding elevated BhCG?
A. High BhCg indicator of ectopic pregnancy
B. High BhCg in second trimester indicator of molar pregnancy.
By: Wafa AlSalem 81
C. High BhCg in second trimester is the most sensitive marker of Down syndrome.
D. High Bhcg can cause elevation of TRH which causes hyperthyroidism

Note to Remember
- Hyperthyroidism in pregnancy is caused by direct stimulation of the maternal thyroid gland by elevated levels
of human chorionic gonadotropin (hCG), which can be associated with a transient lowering in serum TRH and
TSH levels
- Second-trimester (QUADRUPLE test) total levels of hCG, dimeric inhibin A (DIA), AFP, unconjugated estriol
(uE3) are the most sensitive test (its QUADRUPLE test not BhCG alone)

So, for our CASES:


( In the first recall) (1)
- D is the correct answer because high B-HCG causes reciprocal suppression of TSH and TRH.
- Why not C? it’s true that high BHCG is a sensitive marker for down syndrome (but not ALONE!! It’s a quadruple
test)‫ ﻓﮭﻤﺘﻮ؟‬،‫ﯾﻌﻨﻲ ھﺬا ﺻﺢ ﺑﺲ د اﺻﺢ ﻣﻨﮫ‬

(In the second recall) (2)


- D is wrong! High bhCG never causes elevation of TRH levels!! (it causes suppression of TRH and TSH due to the
direct stimulation of the thyroid gland” hyperthyroidism”)
- So, in that case C is the only correct answer so I would go with C
ACOG Reference:

What would you expect in pregnancy?


A. Hematocrit decrease by 20-25%
B. Hematocrit decrease by 40-45%
C. blood volume increase by 20-25%
D. blood volume increase by 40-45%

ANOTHER RECALL

Pregnant in first trimester, what are physiological changes during pregnancy?


A- increased 20-25% blood volume
B- increased 40-45% blood volume
C- increased 60% blood volume

ANOTHER RECALL

By: Wafa AlSalem 82


Pregnant in first trimester, physiological changes during first trimester?
A- increased 20-25% blood volume
B- increased 40-45% blood volume
C- increased 60% blood volume

‫ ﺑﺲ ﻓﻲ‬،‫ اﻛﯿﺪ اﻧﮭﻢ ﺟﺎﻟﺴﯿﻦ ﯾﺴﺄﻟﻮن ﻋﻦ ﻛﻞ اﻟﺤﻤﻞ ﻣﻮ ﺑﺲ اول ﺗﺮاﯾﻤﺴﺘﺮ‬،‫ﻟﺤﺎﻟﮭﺎ اﺑﺪا اﺑﺪا ﻣﺎﻟﻘﯿﺘﮭﺎ ﺑﺄي ﻣﻜﺎن واﺣﺲ ان اﻟﺮﯾﻜﻮل ھﺬا ﻏﻠﻂ‬first trimester ‫طﺒﻌﺎ ﻧﺴﺒﮫ ال‬
!‫ھﺬي اﻟﺤﺎﻟﮫ راح ادﻟﯿﻠﻜﻢ ﻣﻦ دﻟﻮي وﻣﻤﻜﻦ أﻛﻮن ﻏﻠﻄﺎﻧﮫ‬
!!٪٥٠-٤٠ ‫ ﻋﺸﺎن ﯾﻮﺻﻞ‬٣٤ ‫ أﺻﻼ ! وﯾﺴﺘﻤﺮ ﻟﯿﻦ أﺳﺒﻮع‬first trimester ‫ ﯾﻌﻨﻲ ﺑﻨﺺ ال‬blood volume starts from the 6th week ‫اﻟﺤﯿﻦ زﯾﺎده ال‬
٢٥ ‫ ﻓﯿﻌﻨﻲ ﻧﺎﺧﺬ اﻟﺮﻗﻢ اﻷﺻﻐﺮ ﻣﻨﮫ اﻟﻠﻲ ھﻮ‬.‫ﯾﻌﻨﻲ ﻣﺴﺘﺤﯿﻞ ﻣﺴﺘﺤﯿﻞ اﻧﮭﺎ ﺑﺄول ﺗﺮاﯾﻤﺴﺘﺮ ﺑﺘﺰﯾﺪ اﻟﺮﻗﻢ ﻛﺎﻣﻞ‬
‫وﻧﻜﺮر دﻟﯿﺖ ﻣﻦ دﻟﻮي ﺑﺲ ﻗﻠﺖ اﺳﺎﻋﺪﻛﻢ ﺑﺎﻟﺘﻔﻜﯿﺮ‬

Pregnant women Last menstrual period 7th of May, she has regular period and is sure about it. What is the
Expected date of delivery?
A. 10 February next year
B. 10 December same year
C. 25 December next year
D. 30 February next year

The accurate date is 14th of Feb next year

How to calculate the Estimated Delivery Date?


- Day + 7 / Month +9 / Year +1 or 0 (depending on the month)
- E.g., Patient LMP was 18th of May 2020, calculate the EDD?
So, 18+7 / 5+9 / 2020 +1 or 0 (depending on the month)
Her EDD is 25/2/2021

What is the time interval between ovulation and cleavage in dichorionic diamniotic twins?
A. 0-3 days
B. 4-8 days
C. 9-12 days
D. >12 days

By: Wafa AlSalem 83


Abnormal Placenta Implantation
Topic Overview:

Placenta in implanted in the uterine wall, what is that?


A. Placenta previa
B. Placenta accrete
C. Placenta increta
D. Placenta perecreta

Note to Remember
Abnormal Placental Implantation
Placenta Accrete: chorionic villi Attach to the myometrium
Placenta Increta: chorionic villi Invade into the myometrium
Placenta Percreta : chorionic villi Penetrate though the myometrium, penetrate the serosa

By: Wafa AlSalem 84


Antepartum Hemorrhage
Topic Overview:
- Vaginal bleeding after 20 weeks of gestation that is unrelated to labor and delivery
Digital examination of the cervix should be avoided until placenta previa has been excluded by ultrasound
examination.
Differential diagnosis:
- Placenta abruption
- Placenta previa
- Vasa previa
- Uterine rupture

A pregnant woman at 32 weeks gestation presents with severe abdominal pain. she denies any abdominal bleeding.
She has CRL of 34 weeks. Examination reveals a tender and tense uterus. what is the most appropriate next step?
A. Perform an US
B. Cesarean section
C. Check CTG

The first thing to do in antepartum hemorrhage is US to exclude placenta previa. Then CTG.

36 years old G4P3+0 , 38 weeks , a case of polyhydramnios, you did ARM followed by vaginal bleeding and CTG
showed fetal Bradycardia. What is the possible cause ?
A- Abruptio placenta
B- placenta previa
C- Vasa previa

Note to Remember
Vasa Previa hallmark:
i. Painless vaginal bleeding that occurs suddenly after ROM
ii. Fetal bradycardia or sinusoidal pattern

Pregnant women 32 weeks complaining with vaginal bleeding, there were also contractions and dilation. What is the
type of bleeding?
A. Early postpartum bleeding.
B. Late postpartum bleeding.
C. Antepartum bleeding.
D. Intrapartum bleeding

ANOTHER RECALL

Pregnant women 32 weeks complaining with vaginal bleeding, there is no history of contractions or cervical dilation.
What is the type of bleeding?
A. Early postpartum bleeding.
B. Late postpartum bleeding.
C. Antepartum bleeding
D. Intrapartum bleeding
By: Wafa AlSalem 85
Note to Remember
- Antepartum hemorrhage: Vaginal bleeding after 20 weeks of gestation that is unrelated to labor and delivery
- Intrapartum hemorrhage: Vaginal bleeding occurring in the course of normal labor and delivery.

A pregnant woman presented with massive vaginal bleeding from the abruption placenta and her Hgb: 8.6, BP
84\40, HR140. What is the best management to save her life?
A- Admit to ICU
B- Immediate Transfusion of 2 packs FFP
C- Call multidisciplinary and rapid response team (RRT)
D- Immediate Delivery

33 years old pregnant women presente with lower abdominal pain and moderate vaginal bleeding K/C of BA +
epilepsy + smocker What is the highest risk factor for her condition?
A-Age
B- Smocking
C- Bronchial Asthma
D-Epilepsy

Note to Remember
This is Placental abruption. Smoking is one of the risk factors.
Also, maternal age >35 is a risk factor. (our patient is 33) and still even if she was >35 years of age, smoking is a
stronger risk factor than the age. ‫ﻓﺒﻜﻞ اﻟﺤﺎﻻت ﺑﺨﺘﺎر اﻟﺘﺪﺧﯿﻦ‬

Admitted to the labor room. Patient received prostaglandin. patient massively bleed with stop in uterine
contraction. What is the cause of her condition?
1- placenta previa
2- Uterine rapture
3- Placenta abruption
4- Prostaglandin hypersensitivity

Typical scenario of uterine rupture! Patient on oxytocin or prostaglandin then sudden pause of uterine contractions!
è Managed by immediate laparotomy and emergency C-section

Third trimester Pregnant woman with vaginal bleeding, abdominal exam shows a Length less than the gestational
age, CTG shows late decelerations, diagnosis?
A. Placenta previa
B. Vasa previa
C.placenta abrubtion

If he didn’t mention pain in the question -> placenta previa


If he mention painless vaginal bleeding upon rupture of membrane-> vasa previa
If he mentioned painful vaginal bleeding-> placenta abruption.

ANOTHER RECALL

By: Wafa AlSalem 86


Third trimester Pregnant woman with abdominal pain and vaginal bleeding, abdominal exam shows a Length less than
the gestational age, CTG shows late decelerations, diagnosis?
A. Placenta previa
B. Vasa previa
C.placenta abrubtion

Its painful so its placenta abruption.


Placenta previa and vasa previa are painless!

Pregnant in 27 GA, came with minimal bleeding us showed placenta totalis. What is the most imp Mx?
A)abx
B)tocolytics
C)steroids

According to UTD: In patients with placenta previa A course of antenatal corticosteroid therapy is administered to
patients who experience bleeding.

Pregnant in 38W GA, with polyhydramnios and PROM recently. Presented with painful vaginal bleeding and uterine
tenderness, CTG finding shows persistent fetal bradycardia, what would be the cause?
A. Cord prolapse
B. Abruptio placenta
C. Vasa previa
D. Placenta previa

Painful vaginal bleeding + uterine tenderness= placenta abruption.


Also, polyhydramnios especially after rapid decompression is a risk factor for placenta abruption.
Placenta previa and vasa previa are painless!
Cord prolapse will not present with bleeding and tenderness.

Pregnant lady in her 38/39 wk presents with painful vaginal bleeding and tense uterus after spontaneous rupture of
membrane. CTG shows persistent bradycardia What’s the dx?
A. Vasa previa B. Placental abruption C. Cord prolapse

Pregnant came with PROM 4 weeks ago and she came with low BP and low hb
A. Admit ICU under the OB CARE
B. Rapid response team and multi-specialty
C. Observation

Prolonged PROM is a risk factor for placenta abruption -> which causes DIC -> must be managed by Rapid
response team with multidisciplinary intervention

By: Wafa AlSalem 87


Painless mild painless vaginal bleeding at 34 weeks, next step?
A. Admission
B. C/S
C. Steroid
D. Antibiotic

Management of placenta previa:

Placenta previa when to do C/S?


A-36-37 wks
B-38
c-39
D-40

A 30 yo woman G2 P1 at 34+2 weeks gestation presents to emergency room reporting painless vaginal bleeding.
Immediately transvaginal ultrasound shows placenta completely overlying the cervical os. A fetus in cephalic
presentation, and an amniotic fluid index of 14. The cervical appears long and closed on speculum examination. She
has slow, continuous vaginal bleeding. Fetal heart is monitored (image missing)
BP 110/78
HR 106
RR 14
Temperature 36.9C
Which of the following is the most appropriate in management?
A. Hospitalization
B. Betamethasone
C. Cesarian section
D. Magnesium sulfate

The mother is vitally stable! And she has slow bleeding (not significant or heavy), I will admit her and try to
resuscitate her and monitor the fetus.

According to UTD:

By: Wafa AlSalem 88


An actively bleeding placenta previa is a potential obstetric emergency. These patients should be admitted to the
Labor and Delivery Unit for maternal and fetal monitoring, and the anesthesia team should be notified.
The major goals in managing a patient with an acutely bleeding placenta previa are to:
●Achieve and/or maintain maternal hemodynamic stability
●Determine if emergency cesarean birth is indicated

So, for our case when cesarean section is indicated? If Significant vaginal bleeding after 34+0 weeks of gestation
or mild bleeding with category 3 fetal heart tracing.

heavy or severe bleeding ‫ وﻛﺎﺗﺐ ﻧﺰﯾﻒ ﺑﻄﺊ ﯾﻌﻨﻲ ﻟﻮ ﯾﺒﻲ ﯾﺮوﻋﻨﻲ ﻛﺘﺐ‬..‫ اذا وﻗﻒ ﻻ ﺧﻼص‬،‫ﺑﺴﻮي ادﻣﺸﻦ واﺷﻮف اﻟﻔﻮﻟﯿﻮم اذا زاد وﺻﺎر ﻛﺜﯿﺮ ﺑﻮﻟﺪھﺎ‬

Let’s Exclude!!
- Betamethasone-> she at 34 weeks, so not given!
- Cesarean section -> if she has a significant bleeding ≥34 weeks OR mild bleeding with category 3 fetal heart
tracing.
- Magnesium sulfate if <32 weeks

Patient 34 GA, came after hx of fall at home with abdominal tenderness and noticed reduced fetal movement, 4cm
cervix and 80 effacement. Fetal heart rate 150, and moderate abnormal utrine contractions every 3 to 4 minutes.
Whats diagnosis?
A. vasa previa
B. placenta abruption
C. placenta previa
D. latent phase

Concealed abruptio placentae: In ∼ 20% of cases, the hemorrhage is mainly retroplacental; vaginal bleeding
does not occur and presents only with history of one of the risk factors for placenta abruption (e.g. HTN, trauma,
smoking or cocaine use) and abdominal tenderness

Placental abruption and fetal distress and ph 6 wt improve mortality now ?


A. Mild hypothermia
B. IVF
C. electrolytes

Hypothermia treatment, also known as brain cooling, is a relatively new treatment option that lowers a
newborn’s body temperatures in order to reduce neurological injury. Traumatic births restrict the flow of oxygen
to a baby’s brain and a brain injury called hypoxic ischemic encephalopathy (HIE) often results. Research indicates
that the initiation of hypothermia treatment within six hours of an oxygen-depriving insult significantly reduces
chances of death and neurological injuries like cerebral palsy.

A primgravida (at 28 weeks?) and a heavy smoker presented with severe vaginal bleeding and abdominal pain. Most
likely cause is?
A. Rupture of fetal artery
B. Uterine rupture
C. Vasa previa
D. Placental abruption

By: Wafa AlSalem 89


Typical scenario of placenta abruption: smoking or HTN, abdominal pain or tenderness with bleeding.

Pregnant unbooked present with painless vaginal bleeding, fundal high 34 weeks. She lives far away and has
difficulty in trasport. What is the most appropriate thing to do?
a) Corticosteroid induction
b) Deliver by CS
c) US
d) Admit to ward

Any woman with placenta previa can’t be discharged from the hospital unless she is able to return to the
hospital within 20 minutes

Pregnant with history of placental abruption 2 times before came in 3rd preg with same condition and severe
bleeding she's on 37 week, when to admit patient?
A-admit now
B- wait till next bleeding
C-wait until determination of labor day
D-discharge and reassure

According to UTD
We deliver all pregnancies with suspected acute abruption at ≥36+0 weeks of gestation
‫ﺧﻼص ادﺧﻠﮭﺎ واﺳﻮﯾﻠﮭﺎ رﯾﺴﺴﺘﯿﺸﻦ واوﻟﺪھﺎ‬

Most condition cause pregnancy DIC?


A. Placenta abruption
B. Retained product of conception
C. Placenta previa
D. Congenital hemorrhagic disorders

According to UTD
The type and frequency of pregnancy-related conditions that triggered DIC
- Placental abruption (37%)
- Postpartum hemorrhage (PPH) (29%)
- Preeclampsia/eclampsia/HELLP syndrome (14%)
- Acute fatty liver (8%)
- Amniotic fluid embolism(6%)
Pregnancy-related sepsis (6%)

Pregnant patient came with Hx of vaginal bleeding and 3 years try to conceive with infertility what is the next step
in management:
a. coagulation profile request
b. vaginal examination
c. know the cause of being infertile for 3 y

According to Uptodate

By: Wafa AlSalem 90


A positive pregnancy test in a patient with vaginal bleeding should be followed by a pelvic ultrasound (US)
examination
**A manual pelvic examination should NOT be performed on pregnant women with vaginal bleeding after 20 weeks
gestation until placenta previa has been ruled out definitively by ultrasound examination.
è So if there’s ultrasound, it would be the correct answer. Otherwise? I would go with vaginal examination.

Our patient came with bleeding; first thing to do BHcg followed by US, then vaginal examination (if placenta previa
was ruled out) for determining the source and volume of bleeding.

Let’s Exclude!!
- Coagulation -> less likely
- Know the cause of being infertile->She is pregnant now, why would I be concerned about her being
infertile?

By: Wafa AlSalem 91


Labor and CTG monitoring
Topic Overview:
How to Read a CTG:
DR C BRaVADO Rule!

By: Wafa AlSalem 92


Labor Overview:
Stages of Labor
First stage of labor
⁃ Latent Phase: Occurs during the onset of labor and ends at 6 cm of cervical dilation
⁃ Active Phase: Occurs after the latent phase at ≥ 6 cm of cervical dilation and ends with complete (∼ 10 cm)
cervical dilation
Second stage of labor
• A stage of labor that begins once the cervix is completely dilated and ends with the birth of the infant
Third stage of labor
• A stage of labor that begins with the birth of the infant and lasts until the complete expulsion of
the placenta
Abnormal Labor
First stage of labor
⁃ Prolonged Latent Phase: Slow progression with a cervical dilation ≤ 6 cm (greater than 20 hours in
nulliparous women and greater than 14 hours in multiparous women)

Managed By:
Supportive care
Unless ROM? -> Augmentation with Oxytocin
⁃ Prolonged Active Phase:
≥ 6 cm cervical dilation and one of the following:
• No change in cervical dilation after 6 hours of inadequate contractions
• No change in cervical dilation after 4 hours of adequate contractions
Managed By:
-Augmentation with Oxytocin for hypotonic contractions (with cervical ripening for unfavorable cervix 6cm or less)
-Amniotomy (Rupture of membrane)

⁃ Arrested Active Phase:


≥ 6 cm cervical dilation with ruptured membranes and no cervical change after one of the following:
• ≥ 4 hours of adequate contractions
• > 6 hours of inadequate contractions despite oxytocin adminstration
Managed By:
Cesarean Section!
Prolonged Second stage of labor
⁃ Arrest of fetal descent (greater than 3 hours in nulliparous women and greater than 2 hour in multiparous
women) (in patients who received an epidural add an extra hour)
Managed By:
-Instrumental Delivery (if the head is engaged at +2 station and beyond)
-Cesarean Section (if 3 attempts of instrumental delivery failed)
Prolonged Third stage of labor
• Placenta has not been delivered 30 minutes after the birth
Managed By:
-Manual removal
-Hysterectomy (if manual removal failed)
By: Wafa AlSalem 93
Summary of Abnormal Labor Management

ACOG RECOMMENDATIONS:

By: Wafa AlSalem 94


Best way to deliver intrauterine growth restriction fetus with reassuring CTG?
A. Spontaneous vaginal
B. C-section
C. Ventouse
D. Forceps

Primigravida, during labor, cervix fully dilated, CTG show variable deceleration, patient have strong contraction and
head is engaged (station 0), cephalic presenting part, what is the most appropriate management?
A-ventose
B-forceps
C- c-section
D- wait 2 hours

Note to Remember
- We can’t use instrumental delivery (Ventose and forceps unless the CERVIX IS FULLY DILATED AND AT
STATION +2 and BEYOND!) never ever choose ventose and forceps in stations that are less than +2.
- I excluded C-section because it’s the last resort and should be chosen if the CTG is category3
- The CORRECT answer is in utero resuscitation measure e.g. lateral positioning of the mother, O2, IV fluid, stop
oxytocin, administer toocolytic drugs. If not in the choices? I would go for D

Pregnant at term 38 weeks, with irregular uterine contractions, CTG with no acceleration or deceleration but
normal variability, heart rate 140, cervix is 2 cm dilated, what to do next?

A. Oxytocin
B. Prostaglandin
C. C/S
D. Observation Or send home

Note to Remember
Her CTG is Category 1 and reassuring. She is still 2 cm dilated (latent phase) no need for intervention.

Pregnant on epidural and oxytocin suddenly contractions stopped CTG Picture as shown above, what would you do?

A. stop oxytocin and immediate delivery


B. stop epidural
C. increase oxytocin
D. change to lateral position and recheck after 20 mins

By: Wafa AlSalem 95


Note to Remember
Her CTG picture showing: Prolonged decelerations and stoppage of uterine contraction
(According to the recall, in the exam the CTG picture was showing: variable deceleration) ‫ﻣﻼﺣﻈﮫ ھﺬي اﻟﺼﻮره ﺗﻘﺮﯾﺒﯿﮫ ﻓﻘﻂ‬
!‫واﺟﺘﮭﺎد ﺷﺨﺼﻲ وﻟﯿﺴﺖ اﻟﺼﻮرة اﻟﻠﻲ ﺑﺎﻻﺧﺘﺒﺎر‬

Oxytocin side effects: uterine tachysystole and Category II or III FHR tracings are the most common side
effects. Uterine tachysystole may result in abruptio placentae or uterine rupture.

- The patient had sudden contraction stoppage! Which indicates uterine rupture (unlike abruptio placenta causes
hypertonic contractions).
- The common clinical manifestation of uterine rupture is abnormal fetal heart rate pattern (bradycardia,
variable, or late decelerations)
- Uterine rupture is managed by immediate delivery by CS

Primigravid come with labor for 4h Dilated 5 cm, effaced 80%, station +1 after 5h there is no change in cervix, and
contraction occur every 3 min. and stay for 60 sec. What to do?
A. Instrument use
B. C/S
C. IV oxytocin
D. Wait for 2h

Note to Remember
She is still 5 cm dilated (latent phase) no need for intervention she can stay in the latent phase up to 20 hours.

When to choose IV oxytocin? -> if rupture of membrane is mentioned in the question. To avoid infections,
otherwise? NO INTERVENTION IN THE LATENT PHASE AT ALL.

A primigravida patient presented in labor. O/E: the cervix is 5cm dilated and the fetus is in a station O with
cephalic presenting part and this state for 4 hours even the oxytocin had been taken. CTG Picture as shown above.
what is the management for this patient?
b. stop oxytocin
c. immediate CS
d. Wait for 2h
e. instrumental delivery

Note to Remember

By: Wafa AlSalem 96


(According to the recall, in the exam the CTG picture was showing: variable deceleration) ‫ﻣﻼﺣﻈﮫ ھﺬي اﻟﺼﻮره ﺗﻘﺮﯾﺒﯿﮫ ﻓﻘﻂ‬
!‫واﺟﺘﮭﺎد ﺷﺨﺼﻲ وﻟﯿﺴﺖ اﻟﺼﻮرة اﻟﻠﻲ ﺑﺎﻻﺧﺘﺒﺎر‬

Her CTG is category 2 and is managed by in utero resuscitation measure e.g. lateral positioning of the mother, O2,
IV fluid, stop oxytocin, administer toocolytic drugs.

Let’s Exclude!!
- Immediate CS -> if her CTG is category3
- Wait for 2 hours -> if her CTG is category1
- Instrumental Delivery -> If her cervix is fully dilated and the head is engaged +2 and beyond.

Pregnant in labor was induced by oxytocin, CTG showing late deceleration (pic). what to do to reverse condition?
A-give epidural anesthesia
B-give morphine
C-let mother sleep supine
D-stop oxytocin

Note to Remember

Our patient has CTG category2!

Which is managed by in utero resuscitation measure e.g. lateral positioning of the mother, O2, IV fluid, stop
oxytocin, administer toocolytic drugs.

*Never supine!!

A 39-week pregnant patient, history of caesarean section, due to breech presentations, now she is in labor, with
regular contractions 4 every 5 min, cervix fully dilated, full effacement, station +3, What is your management?
A. Ventouse delivery
B. caesarian section
C. examine her after 2 hours

Note to Remember
According to ACOG:
Similar standards should be used to evaluate the labor progress of women undergoing Vaginal delivery after
cesarean and those who have not had a prior cesarean delivery. (‫)ﯾﻌﻨﻲ اﻟﻠﻲ ﻣﺴﻮﯾﮫ ﻋﻤﻠﯿﺔ ﻣﻦ ﻗﺒﻞ ﺣﺎﻟﮭﺎ ﺣﺎل اﻟﻠﻲ ﻣﺎﺳﻮت ﻋﻤﻠﯿﺔ‬

Let’s Exclude!!
- Ventose Delivery -> because her cervix is fully dilated, and the head is engaged +2 and beyond.
CTG category 2 or maternal exhaustion or prolonged 2nd stage ‫واﻟﺴﺆال ﻧﺎﻗﺺ اﻟﺼﺮاﺣﮫ اﻛﯿﺪ ﻓﯿﮫ ﺷﻲء ﯾﺨﻠﯿﻨﺎ ﻧﺨﺘﺎر ال ﻓﯿﻨﺘﻮز ﻣﺜﻞ‬
of labor
- Immediate CS -> if her CTG is category3
- Examine her after 2 hours -> if she wasn’t fully dilated and in active pushing state!! ‫اﻟﺒﺰر راﺳﮫ ﺑﯿﻄﻠﻊ واﻻم ﻻزم ﺗﺪف‬
‫ﺧﻼص وش ﺳﺎﻋﺘﯿﻦ ﻧﺘﺮﻛﮭﺎ‬

if the choice was WAIT 2 more hours (NOT EXAMINE AFTER 2hrs) I would go with it,
‫ ﺑﺲ ھﻮ ﺑﺎﻟﺴﺆال ﻣﻮ ﻣﻮﺿﺤﻠﻲ ﻻ اﻟﻮﻗﺖ اﻟﻠﻲ ﻛﺎﻧﺖ ﺟﺎﻟﺴﮫ ﺗﺪف ﻓﯿﮫ ﻋﺸﺎن أﻗﻮل ﻓﯿﻨﺘﻮز‬،‫ ﺳﺎﻋﺎت واذا ﻣﺎوﻟﺪت ﻧﺴﺘﺨﺪم اﻟﻔﯿﻨﺘﻮز ﻧﺴﺤﺐ اﻟﺒﯿﺒﻲ‬٣ ‫ﯾﻌﻨﻲ اﻟﻤﻔﺮوض اﻧﮭﺎ ﺗﺪف ﺧﻼل‬
‫واﻧﺎ ﻣﺮﺗﺎﺣﮫ وﺑﻨﻔﺲ اﻟﻮﻗﺖ ﻛﻞ اﻟﺨﯿﺎرات اﻟﺜﺎﻧﯿﮫ ﺧﻄﺄ ﻓﯿﻌﻨﻲ اﻻﺻﺢ ھﻮ ﻓﯿﻨﺘﻮز‬

By: Wafa AlSalem 97


Pregnant patient in labor. O/E: the cervix is fully dilated for 2 hr, head in a station - 2, but the patient is got
exhausted from pushing. What is your next step?

A. CS delivery
B. Wait for another 2 hours
C. Ventouse delivery
D. Forceps delivery

Note to Remember
**If this patient is multiparous woman, 2 hours of fully dilated cervix (without epidural) is a prolonged second
stage of labor
**If this patient is primigravida woman, 3 hours of fully dilated cervix (without epidural) is a prolonged second
stage of labor! (we can’t wait for another 2 hours!!)

Both conditions are an indication of:


-Instrumental Delivery (if the head is engaged at +2 station and beyond) (OUR PATIENT DOES NOT FULL FILL
THE CRITERIA)
-Cesarean Section (if the instrumental delivery is contraindicated) (LIKE OUR PATIENT)

41 Weeks pregnant with non-reassuring CTG and she has fibroid, what is the most appropriate step in the
management?
A. induce the labor
B. CS
C. CTG daily

Note to Remember
*Non-reassuring CTG means CTG Category 2 or 3,
But as long as he didn’t specify the category, then anticipate the worst and manage accordingly,
- Category 3 is an indication for urgent C/S
As simple as that!
- We can’t induce labor (oxytocin) if she is category 2 ‫( اﺻﻼ‬The management of category 2 is to stop oxytocin)

42 weeks in labor 7 cm dilated, meconium staining, regular and strong contractions CTG 100 fetal hearts?
A-C/S
B- Augmented labor

Note to Remember
Management of Meconium stained amniotic fluid: induction of labor and continuous fetal monitoring. (expectant
management in case of reassuring CTG is acceptable also)
**Evaluation and interventions are implemented in cases with abnormal tracings indicative of fetal stress to reduce
the likelihood of perinatal asphyxia. We agree that FHR monitoring identifies signs of hypoxemia and allows the
caregivers to initiate prompt interventions in order to reduce the risk of MAS.

- So, our patient has bradycardia CTG category2!! Which is managed by in utero resuscitation measure e.g.
lateral positioning of the mother, O2, IV fluid, stop oxytocin, administer toocolytic drugs.
- For me? I would go with C/S!! (Why would I induce labor in CTG category2?!+she already has strong
contractions)
By: Wafa AlSalem 98
Pregnant female primigravida in labor for couple of hours, 6cm dilated, 80% effacement, 0 station, was managed
with oxytocin and ruptured membrane for 20 hours. CTG showed late deceleration (see above pic) what is the
appropriate mx?
A. C/S
B. Stop oxytocin
C. Amnioinfusion
D. Ampicillin

Note to Remember

- The patient had prolonged active phase of labor-> she was managed by amniotomy and oxytocin
- Now she is having arrested active phase-> which is managed by C/S!

Let’s Exclude!!
- Stop oxytocin-> I would do it as a next step (not the most appropriate) initiating in utero resuscitative
measures while I’m waiting for the C/S (Change of maternal position is a reasonable first treatment option,
followed by O2, IV fluid, stop oxytocin, administer toocolytic drugs.)
- Amnioinfusion -> is the second line option after in utero resuscitative measures (so, i will not choose it as a next
step or most appropriate)
- Ampicillin -> If he says next not most appropriate.

20-years-old primigravida 42 weeks with closed cervix. Induction of labor with prostaglandins gel was started. Her
CTG was “fetal HR 140-160” after 1 h fetal HR 80 and uterine contraction last 2 minutes, most important step in
management?
A. CS
B. Oxygen mask
C. SC terbutaline
D. check cord prolapses

Note to Remember

Uterine Tachysystole: >5 contractions in 10 minutes, averaged over a 30-minute window. Or uterine
hypersystole/hypertonus (a contraction lasting at least 2 minutes) .
- Uterine tachysystole is one of the causes of fetal bradycardia.
- Managed by: discontinue oxytocin or cervical ripening agents + administer tocolytics (e.g. terbutaline)

By: Wafa AlSalem 99


**check cord prolapses-> if rupture of membrane (hint) was in the question, but they didn’t mention if she was
ruptured or not.

Pregnant lady during labor, CTG show fetal persistent bradycardia, what is the cause of her condition?
A. Placental insufficiency
B. Congenital heart disease

Note to Remember
According to ACOG:
Rarely, bradycardia occur in fetuses with congenital heart abnormalities or myocardial conduction defects, such as
those associated with maternal collagen vascular disease.
Most often the onset of bradycardia associated with congenital heart block occurs in the second trimester; it is
extremely unlikely that new onset intrapartum bradycardia would be due to this condition.

Pregnant lady ,41 GA in labor on epidural analgesia, mg sulfate for pre-eclampsia and oxytocin, CTG showed
prolonged deceleration and the mother was hypotensive, most likely cause of the CTG finding:
f. Mg sulfate
g. Oxytocin
h. Epidural analgesia

Pregnant 38 weeks. Diagnosed with preeclampsia and managed with magnesium sulfate. Shes in labor and epidural
anesthesia was started. Oxytocin infusion is started as well. Normal regular contraction. CTG picture: as shown
above, Whats the cause of this CTG finding?
A. Oxytocin infusion
B. Magnesium sulfate infusion
C. Epidural anesthesia
D. Head position of the baby

By: Wafa AlSalem 100


Note to Remember
Her CTG picture showing: normal regular uterine contraction but very minimal variability(reduced)
Medications that can affect fetal heart rate: (ACOG)
- Mg Sulfate: causes minimal or reduced variability
- Epidural analgesia: causes maternal hypotension -> uteroplacental insufficiency-> late or prolonged
decelerations
- Oxytocin: late or prolonged decelerations + uterine hyperstimulation

ACOG Reference:

Reduced variability is caused by?


A- Magnesium sulfate
B- Epidural anesthesia

Woman had C-section. What is the best way to prevent adhesions?


A- perform the C/section preterm
B- add adhesion barrier consisting of oxidized regenerated cellulose before closing the wound
C- closure of the peritoneum
D- Add a layer the incision site

Normal vaginal delivery, Baby weight 4.2kg, Laceration reaching rectal mucosa, which degree:
A-First
B-Second
C-Third
D-Forth

Note to Remember
First degree= Skin
Second degree = Muscle
Third Degree= Sphincter
Fourth degree= Rectal mucosa

By: Wafa AlSalem 101


35-Year-old pregnant lady with fetal death and DIC, her cervix is 4cm dilated, (her vitals are normal), what is the
management?
a) Induction of labor
b) Urgent CS

35-Year-old pregnant lady with fetal death and DIC, her cervix is 4cm dilated, (her vitals show hypotension), what
is the management?
a) Induction of labor
b) Urgent CS

Note to Remember
DIC in Pregnancy
- In Hemodynamically unstable mother OR fetal distress OR contraindication to vaginal delivery
Cesarean delivery is indicated

Why? if the mother is hemodynamically unstabel, Vaginal delivery is not the safest maternal option if hemodynamic
instability from ongoing brisk uterine bleeding persists despite vigorous transfusion of blood and blood products
In these cases, cesarean delivery is indicated to save the mother's life

- In Hemodynamically stable mother with dead or nonviable fetus


Induction of Labor is indicated

Why? avoiding cesarean delivery because of the risk of uncontrollable hemorrhage from surgical incisions and
lacerations.
Delivery is initiated, as removal of the products of conception removes the trigger for DIC

DM pregnant lady 38 weeks in active labor and having DKA profile and fetus in distress (CTG that is suggested of
bradycardia) what to do:
A. Change the mother’s position of labor
B. Stop and do C/S

Note to Remember

Diabetic KetoAcidosis in Pregnancy DKA (ACOG+UTD)


- C/P includes abdominal pain, nausea and vomiting, and altered sensorium.
- Laboratory findings: include hyperglycemia (usually >250 mg/dL [13.9 mmol/L]), acidemia (arterial pH <7.30), an
elevated anion gap (>12 mEq/L), ketonemia, low serum bicarbonate (<15 mEq/L), elevated base deficit (>4
mEq/L), and renal dysfunction
- Continuous fetal heart rate monitoring: minimal or absent variability and absent accelerations, as well as
repetitive decelerations. These abnormalities usually resolve with resolution of DKA
- Treatment: Intravenous insulin, appropriate volume replacement, correction of electrolyte abnormalities
(including potassium, phosphate, and magnesium), monitoring acidosis, and a search for precipitating causes.
Emergency delivery before maternal stabilization should be avoided because it increases the risk of
maternal morbidity and mortality and may result in delivery of a hypoxic, acidotic preterm infant.

So, delivery is not indicated in DKA!!

By: Wafa AlSalem 102


If IV insulin and hydration in the choices I would choose it.
If not? in utero resuscitation measure is also correct.

Urine Dipstick pic with: +2 protein, very high glucose, +ve ketones in pregnant lady 39 weeks with effacement 90%
and cervix dilation 2 cm, what is your most appropriate action?
A. IOL
B. CS
C. Expectant management

Note to Remember
When to deliver in case of Diabetes Mellitus? (ACOG)
- At 39+0 to 39+6 weeks if well-controlled glucose levels and no vascular disease;
- At 36+0 to 38+6 weeks if poorly controlled glucose levels or vascular disease (even earlier if severity of
complications warrants earlier delivery)
- Expectant management beyond 40+0 weeks is not recommended

Delivery before maternal stabilization should be avoided!!

For this patient the correct answer is to manage the DKA (by insulin and hydration) and stabilize the mother!
-> Then induction of labor after correction of her status.!!

If IV insulin and hydration in the choices I would choose it. If not? IOL
‫ اﻧﮫ وش ﻣﻔﺮوض اﺳﻮي ﺑﻌﺪ ﻣﺎ اﻋﺎﻟﺞ ارﺗﻔﺎع اﻟﺴﻜﺮ؟ ھﻞ اﺗﺮﻛﮭﺎ وﻻ اوﻟﺪھﺎ؟ واﻟﺠﻮاب ﺗﻮﻟﯿﺪ‬the most appropriate ‫ﺣﻠﯿﺘﮭﺎ ﻋﻠﻰ أﺳﺎس ان ﻗﺼﺪھﻢ ﻣﻦ ﻛﻠﻤﮫ‬
!!‫واﻻﺻﺢ طﺒﻌﺎ اﻧﻲ اﻋﺎﻟﺞ اﻟﺴﻜﺮ اﻟﻤﺮﺗﻔﻊ‬

While the obstetrician closes the caesarean incision, patient developed bleeding. What is the cause?
A. Liver haemangioma
B. Spleen aneurysm
C. Perforated peptic ulcer
D. Mesenteric ischemia

Note to Remember
- Splenic artery aneurysms-> are the third most common true aneurysm occurring in the abdomen after aortic
and iliac artery aneurysms. Splenic artery aneurysms are more common in women (female:male = 4:1) and are
commonly associated with conditions of increased flow, such as pregnancy (particularly multiparity, because the
risk increases with increasing parity)
Approximately, 95% of SAA rupture occurs during pregnancy, most commonly during the third trimester.
If a woman has an existing SAA, the risk of rupture during pregnancy is 20–50%.
Though the rupture of a SAA during pregnancy is a rare event, it carries a high risk of maternal and fetal
mortality. The mortality in the general population when a SAA ruptures is 25%. In pregnant women, this rate
increases to a 75% maternal mortality rate and a 95% fetal mortality rate

Obstetricians and other emergency providers should consider a ruptured SAA in any pregnant woman who
presents with an acute surgical abdomen.

By: Wafa AlSalem 103


In the rare minority of women of childbearing age who are discovered to have an asymptomatic SAA prior to
rupture, a proactive approach to management should be undertaken due to the high risk of rupture in
pregnancy.

- Liver hemangioma-> May increase in size during pregnancy or with estrogen therapy. But, risk of lesion rupture
is similar for pregnant and nonpregnant women

Elective Cesarean section which week?


A. 36
B. 37
C. 38
D. 39
Another recall:
A. 36-37
B. 38
C. 39
D. full term

Note to Remember
UTD: When a primary cesarean delivery is indicated for maternal or fetal reasons, but preterm birth is not
indicated, there is consensus that planned term cesarean delivery should be scheduled in the 39th or 40th week of
gestation

Full term means between 39 and 40 which is the correct timing. ٣٩ ‫اﺻﺢ ﻣﻦ‬

Female pregnant, polyhydramnios had ruptured membrane, on CTG persistent fetal bradycardia?
A- Rapid fetal descend
B- Cord prolapse
C- Anomaly

Signs of fetal distress in CTG:


A-less contractions
B-early declaration
C-late declaration

Which of the following decreases the risk of PPH in a C/S?


A- Curate uterus with sponge

B- Fundal pressure
C-Spontaneous separation of the placenta

Fundal pressure is part of the active management of the 3rd stage to prevent PPH
So, I would go for it.

What's the best way to deliver placenta in c section?


A. Spontaneous separation
B. Fundus pressure
C. manual removal

‫ ﺑﺨﺼﻮص ھﺬا اﻟﻤﻮﺿﻮع وﺑﻘﻮﻟﻜﻢ وش ﺗﻘﻮل‬ACOG ‫ﻓﯿﮫ ارﺗﻜﻞ ﻧﺎﺷﺮﯾﻨﮭﺎ‬


By: Wafa AlSalem 104
Two methods are frequently used to deliver the placenta at C/S:
- Cord traction
- Manual removal

The best method to deliver the placenta during C/section is by cord traction followed by fundal message and
pressure (active management of the 3rd stage to prevent PPH)

manual removal ‫ ﺑﺮوح ﻣﻊ‬cord traction ‫ اﻓﻀﻞ ! ﻓﻠﻮ ﻣﺎﻓﯿﮫ‬cord traction ‫ ﻣﯿﺜﻮدز ﻛﻠﮭﻢ ﻛﻮﯾﺴﯿﻦ ﺑﺲ ال‬٢ ‫ﯾﻌﻨﻲ ھﺬول ال‬
‫ اﺳﻮﯾﮫ ﺑﻌﺪ ﻣﺎ اطﻠﻌﮭﺎ ﻓﻌﺸﺎن ﻛﺬا ﻣﺎراح اﺧﺘﺎره‬،‫واﻟﻔﻨﺪال ﻣﺴﺎج ﻣﻮ طﺮﯾﻘﮫ ﻋﺸﺎن اوﻟﺪ اﻟﺒﻼﺳﯿﻨﺘﺎ أﺻﻼ‬

Evidence of uteroplacental insufficiency, placenta previa, nonreassuring fetal monitoring, hypertension, intrauterine
growth restriction, oligo- hydramnios, or a history of previous uterine surgery are contraindications to external
cephalic version.

Pregnant G3P2 37 weeks with a history of CS because of nonreassuring CTG. She is in labor with a 4 cm dilation.
The presentation is breech. What is the absolute contraindication for ECV?
A. History of CS
B. Active labor
C. Variable decelerations

External Cephalic Version: Should be offered in all cases ≥ 37 weeks who would like to attempt a vaginal delivery,
unless there are contraindications.
Contraindication
- Absolute:
• Prior classical cesarean delivery
• Prior uterine surgery that entered the endometrial cavity, such as myomectomy
• Placenta previa
• Nonreassuring fetal heart rate
• Unexplanied APH
• Multiple pregnancy
• Suspected macrosomia (typically 5000 grams in women without diabetes, 4500 grams in women
with diabetes)
• Mechanical obstruction to vaginal birth (eg, large fibroid, severely displaced pelvic fracture,
severe fetal hydrocephalus)
• Uterine rupture
- Relative:
• Early labor
• Oligohydramnios or rupture of membranes
• Known nuchal cord
• Structural uterine abnormalities
• Fetal growth restriction IUGR
• Prior abruption or its risks e.g. preeclampsia

By: Wafa AlSalem 105


Pregnant 34 at labor in examination showed transverse presentation she want to know if she can do ECV Ultrasound
shows bicornuate uterus with normal fetus What is the contraindication for ECV?
A. Bicornuate uterus
B. Breech presentation
C. Hx of previous cs

- Bicornuate uterus is relative Contraindication to ECV


- History of previous C/S is not a contraindication !! unless he says "CLASSICAL"

37weeks pregnant came with breech presentation what is your next step?
A. Cesarean section
B. External cephalic version

Lady is 34 wks gestation, had a previous one C/S, on pelvic exam only the cervix was 3 cm dilated, on US placenta
was anterior and laying low. Why is ECV contraindicated in this case?
A. Gestational age
B. Vaginal exam findings
C. US findings.
D. The previous C/S

Pregnant, twins one cephalic and another is breech presentation, how to deliver?
A- Cesarean section
B- Normal delivery

According to ACOG:
Perinatal outcomes for twin gestations in which the first twin is in cephalic presentation are not improved by
cesarean delivery. Thus, women with either cephalic/cephalic-presenting twins or cephalic/noncephalic-presenting
twins should be counseled to attempt vaginal delivery

Unbooked female came to the ED with labor, after investigation she has 100,000 colony bacteria of streptococcus
and she has asthma with using salbutamol, what do you wanna give her now and after deliver?
A. Ampicillin
B. Oxytocin
C. Other two drugs not related

According to ACOG:
- If GBS bacteriuria at any colony count is detected during pregnancy, the woman is at increased risk of GBS
colonization during labor. A notation should be made in her medical record, she should be made aware of her
GBS status, and antibiotic prophylaxis should be administered empirically during labor based on the risk factor
of antepartum GBS bacteriuria

- Intravenous penicillin remains the agent of choice for intrapartum prophylaxis, with intravenous ampicillin as an
acceptable alternative.

Pregnant at 39 weeks now in labor during the delivery you noticed the amniotic fluid is mixed with dark black-green
what is the cause of this color?
A- Meconium aspiration syndrome
By: Wafa AlSalem 106
B- Fetal distress
C- Placenta abruptio
D- Preterm labour

According to UTD:
Fetal stress may result in meconium passage, due to increased peristalsis and relaxation of the anal sphincter from
increased vagal outflow associated with umbilical cord compression or increased sympathetic inflow during hypoxia

Fetal distress -> causes meconium-stained amniotic fluid -> which causes meconium aspiration syndrome

DM female otherwise all normal full term in delivery fetus had tachycardia how to prevent this?
A. Oxytocin’s
B. Change mother position
C. Mg gluconate

Prompt relief of the compromising event, such as correction of maternal hypotension, can result in fetal recovery.
blood vessels ‫ ﯾﻀﻐﻂ ﻋﻠﻰ ال‬uterus ‫ ﻣﻦ اﻟﺴﺪﺣﮫ ﻋﻠﻰ ظﮭﺮھﺎ ﻷن ال‬hypotension ‫ﻣﻤﻜﻦ ﯾﺠﻲ اﻻم‬

Epidural anesthesia, what it’s sparring?


A. Perianal
B. Rectum

Epidural analgesia covers the pudendal nerve, which innervates:


• Sensory: The external genitalia of both sexes and the skin around the anus, anal canal and perineum
• Motor: Pelvic muscles, the external urethral sphincter and the external anal sphincter.

After delivery of the placenta by manual extraction contracting, Retroverted uterus happened but was back in
place. Where was the placenta in the uterus?
A. Anterior
B. Posterior
C. Lateral
D. Fundus

Retroverted uterus means the uterus is tipped backwards (fundus is aimed toward the rectum)
For example, if there’s fibroids in the fundus of the uterus it will cause retroversion of the uterus.
So, I will go with fundus.

Pregnant women during vaginal delivery, what can make her has fourth degree perineal tear?
A. Unrestrained legs and squatting position
B. Unrestrained legs and sitting on chair
C. Restrained legs and use of forceps and other metallic instrument

According to ACOG:
The strongest risk factors for OASIS (Obstetric Anal Sphincter Injuries) including forceps delivery, vacuum-
assisted delivery, midline episiotomy, and increased fetal birth weight. Midline episiotomy combined with forceps

By: Wafa AlSalem 107


delivery substantially increases the risk of third-degree laceration and fourth-degree laceration. The risk of anal
sphincter trauma with operative delivery and episiotomy is increased in primigravid women and multigravid women.

Which of the following positions of a patient in labor would most likely result in the development of a third or
fourth-degree laceration?
A- Unrestrained legs and squatting
B- Unrestrained legs and semi setting
C- Unrestrained legs and in chair
D- Restrained legs and stirrups

According to ACOG:
Upright positions (including walking, sitting, standing, and kneeling), were associated with a possible increase in
second-degree perineal tears

Pregnant in labor and signs of meconium stain how to manage baby?


A. Oropharynex suction before deliver the body
B. NICU after delivery
C. TOCOLYTIC
D. Intratracheal suctioning

According to ACOG:
If the infant born through meconium-stained amniotic fluid presents with poor muscle tone and inadequate
breathing efforts, the initial steps of resuscitation should be completed under the radiant warmer.
Appropriate intervention to support ventilation and oxygenation should be initiated as indicated for each infant.
Infants with meconium-stained amniotic fluid should no longer routinely receive intrapartum suctioning, whether
they are vigorous or not.
In addition, meconium-stained amniotic fluid is a condition that requires the notification and availability of an
appropriately credentialed team with full resuscitation skills, including endotracheal intubation.

By: Wafa AlSalem 108


Preterm labor, Prelabor Rupture of Membrane (PROM)
and Preterm Prelabor Rupture of Membrane (PPROM)
Topic Overview:
Preterm Labor
Regular uterine contractions with cervical effacement, dilation, or both before 37 weeks gestation.
Management:
- ≥34 weeks of gestation
• Admitted for delivery!
- <34 weeks of gestation
• A course of betamethasone
o A single course of corticosteroids is recommended for pregnant women between 24 weeks
and 34 weeks of gestation who are at risk of delivery within 7 days.
o A single repeat course of antenatal corticosteroids should, therefore, be considered in
women who are less than 34 weeks of gestation, who are at risk of preterm delivery within
the next 7 days, and whose prior course of antenatal corticosteroids was administered more
than 14 days previously
• Tocolytic drugs for up to 48 hours
o To delay delivery and enable administration of antenatal corticosteroid
• Magnesium sulfate for 24 to 32 weeks of gestation
o Reduces the risk and severity of neurological disorders (e.g., cerebral palsy)
Prelabor Rupture of Membrane (PROM)
Rupture of membranes before the onset of labor at term (≥37 weeks)
Investigations:
⁃ Speculum Examination: Pooling of amniotic fluid in the posterior vaginal vault observed is the gold standard
Management:
• Induction of labor is recommended.
• Expectant management for up to 12–24 hours is reasonable in otherwise uncomplicated
pregnancy and in the absence of infection.
• GBS Prophylaxis is indicated in case of rupture of membrane for 18hrs and more or fever 38C!
Preterm Prelabor Rupture of Membrane (PPROM)
Rupture of membranes before labor that occurs before 37 weeks of gestation
Management:
- Late preterm 34-36 6/7 weeks of gestation
• Either expectant management or immediate delivery is a reasonable option
- Preterm 24-33 6/7 weeks of gestation
• Expectant management
Expectant management:
§ A course of betamethasone
§ GBS prophylaxis
§ Tocolytic drugs (Should not be administered for more than 48 hours. Or to patients who are
in advanced labor (>4 cm dilation) or who have any findings suggestive of subclinical or overt
chorioamnionitis.)
§ Magnesium sulfate (if preterm delivery <32 weeks is anticipated “at risk of imminent
delivery)
• Prompt delivery in:
§ Patients with signs of intrauterine infection, abruptio placentae, nonreassuring fetal testing,
or a high risk of cord prolapse is present or suspected

By: Wafa AlSalem 109


Woman G2 P1 at 32 weeks’ gestation presents to Emergency Department complaining of lower
abdominal and back pain, which has increased in frequency and intensity over the last few hour Abdominal
examination shows fundal height equals to 32 cm longitudinal lie fetus and cephalic presentation, Fetal heart was
positive and cardiotocography is reactive with 29 uterine contractions per 10 minutes . Vaginal examination shows
dilated cervix, 70 % effacement, -3 station and cephalic. Which of the following is the most appropriate step in
management?
A. Inform neonatologist, administer corticosteroids, and strict bed rest
B. Inform neonatologist, administer corticosteroids, and hydrate the patient
C. Inform neonatologist, administer tocolytics, and start intravenous antibiotics
D. Inform neonatologist, administer intravenous antibiotics, and strict bed rest

Note to Remember
Preterm labor at 32 weeks is managed by:
Corticosteriod+Tocolytics
Never ever choose bed rest!! ‫اووﻋﻜﻚ ﺗﺨﺘﺎر اﻧﮭﺎ ﺗﺠﻠﺲ ﺑﺎﻟﺴﺮﯾﺮ ھﺬا ﺑﺮاﻛﺘﺲ ﻗﺪﯾﻢ واﻟﺤﯿﻦ ﻣﻌﺪ ﯾﺴﻮوﻧﮫ‬

Let’s Exclude!!
A-> is wrong because of the “bed rest”
C-> is wrong because we don’t give antibiotic in preterm delivery unless indicated!!
D-> ‫ﻋﺎد ذا ﻣﺎﻓﯿﮫ ﺷﻲء ﺻﺎﺣﻲ‬

Woman almost full term diagnosed as active labor now. She had clear fluid discharge before her contraction or
labor starts, what is the DX?
a. preterm premature ROM
b. premature ROM
c. preterm ROM

Note to Remember
- Premature rupture of membranes (PROM) is rupture of membranes before the onset of labor at term
- Preterm premature rupture of membranes (PPROM) is rupture of membranes before labor that occurs before
37 weeks of gestation

PROM How to confirm its amniotic fluid?


A) Sterile speculum examination
B) Abx
C) chemical testing
D) Fetal hormones

By: Wafa AlSalem 110


Patient 27wks in labor, cervix 6cm dilatation. What to give?
A-Steroid
B-Abx
C-Arbostan
D-MgSo4

Note to Remember
According to ACOG:
The most beneficial intervention for improvement of neonatal outcomes among patients who give birth preterm is
the administration of antenatal corticosteroids. A single course of corticosteroids is recommended for pregnant
women between 24 weeks and 34 weeks of gestation who are at risk of delivery within 7 days.
‫اﻟﺼﺪق ﺳﺆاﻟﮭﻢ ﺗﺤﺘﺎر ﻣﺎﺗﺪري وش ﯾﺒﻲ ﻣﻦ اﻟﺤﯿﺎة وﻛﻞ أ و د ﺻﺢ!! ﺑﺲ ﺳﺘﯿﺮوﯾﺪ طﺒﻌﺎ طﺒﻌﺎ اھﻢ ﻓﺒﺨﺘﺎر اﻟﻤﮭﻢ‬

Healthy lady with Preterm labor what to give her?


A. Indomethacin
B. magnesium sulfate
C. Nifedipine
D. Terbutaune or nitroglycerin

Note to Remember
Tocolysis choice in Preterm Labor:
• First-line:
§ Indomethacin (24-32 weeks)
§ Nifedipine (32-34 weeks or women who have a contraindication to indomethacin)
• Second-line: if the first-line drug does not inhibit contractions, we discontinue it and begin therapy with
another agent.
§ Nifedipine (24-32 weeks)
§ Terbutaline (32-34 or for those who received nifedipine as a first-line agent at 24 to 32 weeks)

All answers are correct!! Tocolytic choice depends on the gestational age and maternal contraindication…
So, in this case I will go with the first choice (Indomethacin)
‫ھﻢ ﻣﺎﻋﻄﻮﻧﻲ ﺑﺎﻟﻤﻌﻄﯿﺎت أﺷﯿﺎء اﻧﻘﻲ اﻟﺪواء ﻋﻠﻰ أﺳﺎﺳﮭﺎ ﻓﺮاح اﺧﺘﺎر اول دواء ﯾُﻔﻀﻞ اﺳﺘﺨﺪاﻣﮫ‬

24-year-old lady with rupture of membrane at 34 weeks gestation. What to do first?


A. Tocolytics
B. Steroids
C. Antibiotics

Note to Remember
Management of PPROM 34-36 6/7 weeks of gestation
• Either expectant management or immediate delivery is a reasonable option
Expectant management:
§ A course of betamethasone (if steroids not previously given, if proceeding with induction or
delivery in <24 hours and no more than 7 days, and no evidence of chorioamnionitis)

By: Wafa AlSalem 111


27 weeks lady with hx of PPROM. The ctg is reassuring, normal CBC, she is medically free, she denies any uterine
contractions. Management?
A- Corticosteroids
B- Antibiotics
C- Tocolytics
D- Go for C/S

Note to Remember
Management of PPROM 24-33 6/7 weeks of gestation
• Expectant management
Expectant management:
§ A course of betamethasone
§ GBS prophylaxis
§ Tocolytic drugs (Should not be administered for more than 48 hours. Or to patients who are
in advanced labor (>4 cm dilation) or who have any findings suggestive of subclinical or overt
chorioamnionitis.)
§ Magnesium sulfate (if preterm delivery <32 weeks is anticipated”at risk of imminent
delivery”)
• Prompt delivery in:
§ Patients with signs of intrauterine infection, abruptio placentae, nonreassuring fetal testing,
or a high risk of cord prolapse is present or suspected

‫ھﺬا اﻟﺴﺆال ﯾﺴﺘﮭﺒﻞ ﻻن أ و ب و ﺳﻲ ﻛﻠﮭﻢ ﺻﺢ !! ﺑﺲ اول ﺷﻲء واھﻢ ﺷﻲء ھﻮ ﺳﺘﯿﺮوﯾﺪ‬

A pregnant female at 31+5 weeks of gestation, presents with preterm premature rupture of membranes. CTG
shows several variable decelerations. What is the next step in management?
A. Tocolytics And Steroids
B. MgSO4
C. Expectant Management
D. Urgent Delivery

Note to Remember
- In Preterm labor or PPROM, Tocolysis is contraindicated in case of non-reassuring fetal status. And prompt
delivery is indicated.
Route of delivery — In the absence of contraindications to labor and vaginal birth, most patients will
deliver by spontaneous or induced vaginal delivery. Cesarean delivery is performed for standard indications;
otherwise, labor is induced.

‫ و اذا‬in utero resuscitative measures ‫ وطﻮﻟﻨﺎ اﻟﻮﻻده اﻟﺒﯿﺒﻲ ﻣﻤﻜﻦ ﯾﺘﺄﺛﺮ ﻓﻌﺸﺎن ﻛﺬا ﻧﺤﺎول ﻧﺴﻮي ال‬tocolysis ‫ﯾﻌﻨﻲ ﻧﺨﺎف اذا ﻋﻄﯿﻨﺎھﺎ‬
‫ ﺳﺎﻋﮫ‬٢٤ ‫ ﻷن ھﻮ ﯾﻨﻌﻄﻰ ﻗﺒﻞ اﻟﻮﻻده ب‬MgSo4 ‫< ﺧﻼص ﻧﻮﻟﺪھﺎ ﻋﻠﻰ طﻮل وﻣﺎﻓﻲ وﻗﺖ ﻧﻌﻄﻲ ﻓﯿﮫ ال‬- ‫ ﺣﻘﮭﺎ‬CTG ‫ﻣﺎﻋﺪﻟﺖ ال‬

Primigravida, preterm known case of DM 1 came with sever contractions and closed cervix What to give?
A. Steroids + insulin
B. Steroids + insulin +tocolytics.
C. Steroids and tocolytics
D. Insulin and tocolytics

By: Wafa AlSalem 112


Note to Remember
For Pregestational diabetes mellitus:
- ACOG recommends intravenous insulin infusion for intrapartum glycemic management

For Preterm Contractions Without Cervical Change (ACOG):


- Especially those with a cervical dilation of less than 2 cm, generally should not be treated with tocolytics.

Preterm 30 week, 80% effacement, 2 cm dilatation, stable mom and fetus. Admitted to labor. What to do?
A- call NICU and labor
B- give dexamethasone, GBS swab, nifedipine and labor
C- give dexamethasone and labor
D- this is false labor

Pretem labor <34 weeks of gestation is managed by:


- A course of betamethasone
- Tocolytic drugs for up to 48 hours
- Magnesium sulfate for 24 to 32 weeks of gestation

‫اﻟﺼﺮاﺣﮫ ﻣﺎﻋﺠﺒﻨﻲ ﻻ اﻟﺴﺆال وﻻ اﻹﺟﺎﺑﺎت ﺑﺲ ﺑﻲ ھﻮ اﻟﻮﺣﯿﺪ اﻟﻠﻲ ﯾﺘﺮﻗﻊ اﻟﺒﺎﻗﻲ ﻛﻠﮫ ﻏﻠﻂ‬

Pregnant 31+5 weeks normal CTG + normal progression of labor 4 cm dilated then 5 cm 80% effaced. Management?
A-Tocolytics + steroids
B-Prostaglandin
C-Reassure

Pretem labor <34 weeks of gestation is managed by:


- A course of betamethasone
- Tocolytic drugs for up to 48 hours
- Magnesium sulfate for 24 to 32 weeks of gestation

A Female at 31wks gestational age presented with cervical dilatation and no uterine contractions. What is the best
thing to do?
A. Give Bed Rest
B. Do A Cerclage
C. Tocolytics
D. Antibiotics

By: Wafa AlSalem 113


‫ ﺣﺮﻓﯿﺎ ﻣﻌﺪ ﯾُﺴﺘﺨﺪم ﻷﻧﮫ ﯾﺴﺒﺐ اﺿﺮار اﻛﺜﺮ ﻣﻦ ﻧﻔﻌﮫ ﺑﺲ ھﻮ اﻟﺸﻲء اﻟﻮﺣﯿﺪ اﻟﻠﻲ ﯾﺘﺮﻗﻊ ﺑﮭﺬي‬bed rest historical ‫ ﺗﺴﻤﻲ ال‬ACOG ‫ھﺬا اﻟﺴﺆال ﻏﺒﻲ ﻏﺒﻲ ﺟﺪا‬
‫اﻟﺨﯿﺎرات ﻻن ﻛﻠﮭﺎ ﻏﻠﻂ‬
‫و أ ﯾﻤﺸﻲ ﺑﺎﻟﺪف ﺑﺲ اھﻢ ﺷﻲء ﯾﻤﺸﻲ‬
**Cerclage is not done after 24 weeks
**Tocolysis ?? Why?? She doesnt have contractions or labor
**antibiotic? What is my indication?

Bed rest ‫!!ﻣﺎﻧﺨﺘﺎره ﺑﺎﻟﻌﺎده ﺑﺲ ﺑﮭﺬا اﻟﺴﺆال‬

‫ اذا ﺟﺎﻛﻢ ﻛﺬا اﺧﺘﺎروه‬by exclusion only

Patient on tocolytic beta-mimetics (terbutaline) what is the most side effect?


A-Oliguria
B-Palptation
C-Abdominal pain
D- Vaginal bleeding

According to ACOG
Terbutaline side effects (beta-adrenergic receptor agonists)
Maternal:
- Tachycardia, hypotension, tremor, palpitation, SOB, chest discomfort, pulmonary edema, hypokalemia, and
hyperglycemia
Fetal:
- Fetal tachycardia

G3P2 with gestational age of 30. Came with rupture of membrane and has no contractions. What you’ll give?
A) Ampicilin + oxytocin
B) Dexa + nifedipine
C) Dexa + magnesium sulphate
D) Dexa + erythromycin + ampicilin

This is a case of PPROM which is managed by:


- 24-33 6/7 weeks of gestation
• Expectant management
Expectant management:
A course of betamethasone
§
GBS prophylaxis
§
Tocolytic drugs (Should not be administered for more than 48 hours. Or to patients who are
§
in advanced labor (>4 cm dilation) or who have any findings suggestive of subclinical or overt
chorioamnionitis.)
§ Magnesium sulfate (if preterm delivery <32 weeks is anticipated” at risk of imminent
delivery”)
• Prompt delivery in:
§ Patients with signs of intrauterine infection, abruptio placentae, nonreassuring fetal testing,
or a high risk of cord prolapse is present or suspected

By: Wafa AlSalem 114


For GBS Prophylaxis (ACOG):
The recommended antibiotic regimen during expectant management of women with preterm PROM who are at less
than 34 0/7 weeks of gestation
- A 7-day course of therapy of latency antibiotics with a combination of intravenous ampicillin and erythromycin
- Followed by oral amoxicillin and erythromycin
** Some centers have replaced the use of erythromycin with azithromycin in situations in which erythromycin is
not available or not tolerated.

Tocolysis choice in Preterm Labor(UTD):


• First-line:
§ Indomethacin (24-32 weeks)
§ Nifedipine (32-34 weeks or women who have a contraindication to indomethacin)

!‫ و أﺻﻼ اﻟﺪواء اﻟﻠﻲ ﺣﺎطﯿﻨﮫ ﻣﻮ ھﻮ اﻟﻔﯿﺮﺳﺖ ﻻﯾﻦ‬Antibiotic! ‫اﻟﺘﻮﻛﻮﻻﯾﺴﺰ ﻣﺶ ﻏﻠﻂ ﺑﺲ ﻛﻤﺎن ﻣﺎﻋﻨﺪھﺎ ﻛﻮﻧﺘﺮاﻛﺸﻦ وﻣﻨﺐ ﺣﺮﯾﺼﮫ ﻋﻠﯿﮭﺎ زي ال‬
its not indicated ! ‫ ﺳﺎﻋﮫ! ﺑﺲ واﺿﺢ ان ﻣﺎﻋﻨﺪھﺎ ﻻ ﻛﻮﻧﺘﺮاﻛﺸﻦ وﻻ ﺷﻲء ف ﺑﮭﺬي اﻟﺤﺎﻟﮫ‬٢٤‫واﻟﻤﻐﺎﻧﯿﺴﯿﻮم راح اﻋﻄﯿﮭﺎ اذا اﻧﺎ ﻣﺘﻮﻗﻌﮫ ﺧﻼص ﺑﺘﻮﻟﺪ ﺧﻼل‬

ACOG ‫ﺷﻮﻓﻮ ﻣﻌﺎي اﻟﺼﻮره اﻟﻠﻲ ﺗﺤﺖ ﻣﻦ‬

By: Wafa AlSalem 115


Postpartum Hemorrhage (PPH)
Topic Overview:

Primigravida woman just delivered spontaneously baby is delivered complete and intact. Massaging of the uterine is
performed along with 20 units of oxytocin in 1000 of lactated Ringers fast drip. inspection of the genital tract,
there’s second degree laceration 2-cm left lateral vaginal wall, suturing is difficult because of bleeding from above
the site of laceration. a soft, boggy uterine fundus Blood pressure 164/92 mmHg Heart rate 130 /min Which of
the following is the best step in management?
A. Prostaglandin f2a
B. methylergonovine
C. manual exploration
D. oxytocin 10 units again

Note to Remember

Medications used in PPH Management:


1) Oxytocin 10-40 units in 500/1000ml NS/RL or 10 units IM (1st line)
2) Methylergonovine 0.2 mg IM (2nd line, Contraindicated in preeclampsia and HTN)
3) Carboprost (hemabate) PG F2a 0.25mg IM (3rd line, Contraindicated in Asthma)
4)Misoprostol 600-1000 micrograms PO, or rectal (4th line)

In case of ongoing hemorrhage and poor uterine response -> multiple uterotonic agent should be used in rapid
succession
Throw your big guns in the same time ! You have bleeding and emergency!

ACOG:

Another reference!
Williams Obstetrics:

By: Wafa AlSalem 116


During C/S, PPH with failed manual compression and oxytocin what you should do next?

A. B-lynch
B. Bakri balloon
C. Artery embolization
D. Hysterectomy

During vaginal delivery, PPH with failed manual compression and oxytocin what you should do next?

A. B-lynch
B. Bakri balloon
C. Artery ligation
D. Hysterectomy

During vaginal delivery, PPH with failed manual compression and oxytocin, and the patient is hemodynamically
unstable (hypotensive) what you should do next?

A. B-lynch
B. Artery embolization
C. Artery ligation
D. Hysterectomy

Note to Remember
- In case of cesarean section à B-Lynch, failed? à Artery Embolization (ONLY if hemodynamically stable),
Hemodynamically unstable? Laparotomy and Artery ligation, failed? à Hysterectomy
- In case of Vaginal Delivery à Bakri balloon, failed? à Artery Embolization (ONLY if hemodynamically stable),
Hemodynamically unstable? Laparotomy and Artery ligation, failed? à Hysterectomy

Pregnant with massive bleeding from abruptio placentae. She is hypotensive and vitally unstable. What is the most
appropriate thing to do to save her life?
A.admitted to ICU with obstetric team
B.Admitted with different spacilized team
C. 2 peripheral IV cannula and blood transfusion
D. Rapid response team with multidisciplinary intervention

Note to Remember
According to ACOG: Some emergencies are truly sudden and catastrophic, such as a ruptured aneurysm, massive
pulmonary embolus, or complete abruptio placentae in a trauma setting. However, many emergencies are preceded
by a period of instability during which timely intervention may help avoid disaster.
Medical emergency teams—sometimes referred to as “Ob Team Stat” for obstetric emergencies or a rapid
response team—are designated skilled responders who are ready to intervene during such emergencies.

20 years old pregnant woman presenting with lost of fetal movement followed by decrease urinary output and
difficulty breathing
Her labs are as follow:
aPTT prolonged, Fibrinogen low, Platelets low, what is the diagnosis?
A. acute glomerulonephritis

By: Wafa AlSalem 117


B. DIC
C. Autoimmune thrombocytopenia purpura
D. Acute amniotic embolism

Note to Remember

According to ACOG:
Amniotic fluid embolism is a rare, unpredictable, unpreventable, and devastating obstetric emergency signaled by a
triad of
1) Hemodynamic compromise
2) Respiratory compromise
3) Strictly defined disseminated intravascular coagulation

If no respiratory symptoms -> DIC


DIC+respiratory -> amniotic fluid embolism

Pregnant in labour, she takes heparin, post delivery she has heavy bleeding? What to give?
A) FFP
B) Portamine sulphate
C) Vit K

Neutralization of systemic heparin with protamine sulfate (reverse the anticoagulant effects of heparin.)

Percentage of maternal death worldwide due to post partum hemorrhage?


a) 10%
b) 15%
c) 20%
d)25%

A patient with post-partum hemorrhage who was resuscitated then they found that there is persistent bleeding at
several puncture sites what is the next important step?
A) Reversal of coagulopathy
B) Oxytocin
C) Prostaglandin

She is bleeding from the puncture sites -> suspecting DIC -> FFP and consider cryoprecipitate

Woman in delivery bleeding not stop, she wants to conceive in the future, which structure you should ligate?
By: Wafa AlSalem 118
A. Uterosacral ligament
B. External iliac artery
C. Internal iliac artery
D. Uterine vein

Case of PPH on oxytocin , while you manage the case you notice more bleeding(something like that) you examine her
but the bleeding prevents you from determining the exact source of bleeding, what to do ??
A. Call for Help
B. Misoprostol
C. Oxytocin again
D. Methylergonovine

First thing to do is to call for help!

Patient post delivery massage is done, oxytocin done. She was bleeding. On inspection you found it is due to
laceration 2 cm you tried sutures but it’s not possible due to perfuse bleeding from above, what is the most
appropriate next step?
A. Prostaglandin F2alpha
B. Oxytocin again
C. Suppurative treatment
D. Explore the uterus and examine it

To see what is the cause of bleeding, uterine atony or retained products of conception? Then manage
accordingly.

Lady presenting with vaginal bleeding with fever 15 days after C/S. Most probable Dx?
A. Wound infection
B. Retained products of conception
C. Endometritis
D. Mastitis

The most important risk factor for endometritis is Cesarean Section!

Retained product of conception will present mainly with heavy bleeding


Endometritis will give you in the history C/S and fever

Highest rate successful drug used in Postpartum Hemorrhage?


A. carboprost
B. Oxytocin
C. misoprostol
D. Methylergonovine

Most common drugs used to treat PPH after Uterine atony ?


A. Oxytocin
B. Carboprost
C. misoprostol
D. Methylergonovine

By: Wafa AlSalem 119


Bleeding after 4 hrs of delivery what type of PPH?
A. Primary
B. Secondary

Primary PPH: within 24 hrs


Secondary PPH: more than 24 hr after delivery, up to 12 weeks postpartum

Patient had PPH she is known case of asthma what medication you can’t give her?
A. Oxytocin
B. Carboprost
C. misoprostol
D. Methylergonovine

Patient had PPH she is known case of HTN, what medication you can’t give her?
A. Oxytocin
B. Carboprost
C. misoprostol
D. Methylergonovine

PPH medications
- Methylergonovine 0.2 mg IM (2nd line, Contraindicated in preeclampsia and HTN)
- Carboprost (hemabate) PG F2a 0.25mg IM (3rd line, Contraindicated in Asthma)

PPH Medication?
A. Oxytocin 20 units mixed with 500ml D5 IV
B. Ergo 0.5mg IM
C. Ergo 0.2 mg IV

Medications used in PPH Management:


1) Oxytocin 10-40 units in 500/1000ml NS/RL or 10 units IM (1st line) (For patients in whom infusion of sodium
chloride must be avoided, 5 % dextrose solution may be used)
2) Methylergonovine 0.2 mg IM (2nd line, Contraindicated in preeclampsia and HTN)
3) Carboprost (hemabate) PG F2a 0.25mg IM (3rd line, Contraindicated in Asthma)
4)Misoprostol 600-1000 micrograms PO, or rectal (4th line)

Patient delivered without episiotomy, placenta was check and all parts were delivered, then pt had gush of blood
coming, what is your next step?
A-Check uterine contraction
B-Get blood for CBC
C-Get blood for coagulation profile
D-Evacuate the uterine

First thing to do in PPH is to start at the UTERUS and to call for help
‫اول ﺷﻲء ﺗﻜﻮن ﯾﺪك داﺧﻞ اﻟﯿﻮﺗﯿﺮس وﺗﻨﺎدي ﻓﻮر ھﯿﻠﺐ ﺑﻌﺪﯾﻦ اﻟﻠﻲ ﯾﺠﻮن ﯾﺴﺎﻋﺪوﻧﻚ اﻟﻤﻤﺮﺿﺎت ھﻢ ﯾﺎﺧﺬون اﻟﺪم وھﺬي اﻻﺷﯿﺎء‬

By: Wafa AlSalem 120


Most accurate method to make sure full placental seperation?
A.pressure and massage uterus
B. manual removal
C. wait for spontaneous
D. with a sponge

Ultrasonography or intrauterine manual examination is usually used to diagnose retained placental tissue.

When a retained placenta is identified, the first step is to attempt manual removal of the tissue.
‫ ﻗﺎل ﻛﯿﻒ اﺗﺎﻛﺪ اﻧﮭﺎ ﻛﻠﮭﺎ ﺑﺮا؟‬،‫ھﻮ ﻣﺎﻗﺎل وش ﻣﻔﺮوض اﺳﻮي ﻋﺸﺎن اوﻟﺪ اﻟﺒﻼﺳﯿﻨﺘﺎ‬
US ‫ﯾﺎ ﺑﯿﺪي اﺷﻮف ﯾﺎ ب‬

How to differentiate between hypovolemic shock and postpartum hemorrhage ?


A/Hemoglobin
B/Hematocrit
C/Increase Pulse
D/Visual

According to UTD
Shock refers to inadequate tissue perfusion, which manifests clinically as hemodynamic disturbances and organ
dysfunction.

emergency ‫ ﻋﻠﻰ طﻮل وﻧﺤﺎول ﻧﻌﺎﻟﺠﮫ ﺑﺴﺮﻋﮫ ﻋﺸﺎﻧﮫ‬vitals ‫ ﻻزم ﻧﻌﺮﻓﮫ ﻣﻦ ال‬shock ‫ال‬
(‫اھﻢ ﻣﻦ اﻟﺒﺎﻗﯿﻦ)ھﯿﻤﺎﺗﻮﻛﺮت‬pulse ‫ ﯾﺰﯾﺪ وھﺬا ﯾﻌﻨﻲ ان ال‬pulse ‫ ﯾﺒﺪأ ال‬shock ‫ ﻣﻦ ال‬٣ ‫ و‬٢ ‫ ﺑﺲ ﻟﻤﺎ ﻧﺮوح ﻛﻼس‬pulse ‫ ﺑﺎﻟﺼﻮره ﻣﺎﯾﺘﻐﯿﺮ ال‬١ ‫وﺷﻮﻓﻮ ﻛﻼس‬
‫ وﻻزم اﺗﺼﺮف ﻋﻠﻰ طﻮل ﻟﻤﺎ اﺷﻮﻓﮫ ﯾﺰﯾﺪ‬clinical manifestations ‫ﻋﺸﺎﻧﮫ ﻣﻦ ال‬

!‫واﺻﻼ ﻣﺎﻓﻲ ﻣﻜﺎن ﯾﻘﻮﻟﻚ ﻻﺷﻔﺖ ال ھﯿﻤﺎﺗﻮﻛﺮت رﻗﻤﮭﺎ ﻛﺬا ھﺬا ﯾﻌﻨﻲ ﺷﻮك‬

‫ﻋﺎد اﻧﺎ ﺟﺎﻟﺴﮫ اﻗﻮﻟﻜﻢ ﻣﻦ ﻓﮭﻤﻲ وﺟﺎﻧﻲ ﺑﺎﻻﺧﺘﺒﺎر وﺣﻠﯿﺘﮫ ﻛﺬا‬


+
According to ACOG
Visually estimated blood loss is considered inaccurate. So, I would exclude the last option (visual)

Female after SVD and after delivery of placenta she bleeds heavy amounts what you will do:
A-Collect sample and send for investigation
B-Send to OR to open and see

By: Wafa AlSalem 121


C-Blood transfusion with O –
D-Send for blood cross-match

According to Williams Obstetrics:


In postpartum Hemorrhage, it is essential to immediately identify uterine atony, retained placental fragments, or
genital tract lacerations. At least one and preferably more large-bore intravenous infusion systems are established
promptly with rapid administration of crystalloid solutions, while blood is made available

!‫اﻟﺴﺆال ﺟﺪا ﯾﺴﺘﮭﺒﻞ ﻣﻔﺮوض ﻓﯿﮫ ﻣﻌﻄﯿﺎت اﻛﺜﺮ ﻋﺸﺎن اﻗﺮر اﻋﻄﯿﮭﺎ دم وﻻ ﻻ‬

- Collect sample and send for investigation, ‫ھﻲ ﺻﺢ اﺳﻮﯾﮭﺎ ﺑﺲ اﻧﻲ اﻋﻄﯿﮭﺎ دم وﻓﻠﻮﯾﺪ أوﻟﻰ ﺻﺢ؟‬
- Send to OR and see ‫ﻟﯿﺶ اﻧﻂ اﺧﺮ ﺧﻄﻮه ﻋﻠﻰ طﻮل؟ اﺳﺘﻨﻰ ﻋﺎﻟﺠﮭﺎ ﺑﺎﻻدوﯾﮫ اول‬
- Blood transfusion ‫ﻣﻔﺮوض ﻣﻌﻄﯿﻨﻲ ﻣﻌﻄﯿﺎت اﻛﺜﺮ ﺑﺲ ھﻮ اﺻﺢ اﺟﺎﺑﮫ ﻻزم اﻋﻮض اﻟﺪم‬
- Send for cross match ‫ﺻﺤﺢ اﯾﻮه اﺳﻮﯾﮫ ﺑﺲ وش أوﻟﻰ اﻟﻔﻠﻮﯾﺪ واﻟﺪم وﻻ اﻟﻜﺮوس ﻣﺎﺗﺶ؟‬

By: Wafa AlSalem 122


Postpartum

Patient developed DVT, had C-section 10 days ago. What to give?


A. Heparin
B. IVC
C. Thrombolytics
D. Embolectomy

30 years old post partum woman admitted for right leg DVT and was started on enoxparin 80mg BID. Then she
developed sudden onset dyspnea and right pleuritic chest pain, on PE; She was dyspneic and apprehensive, heart
sound showed loud P2 and lungs were clear on auscultation, vital signs normal
ABG: normal HCO3 and Po2, decreased PCO2 and high pH
CT showed thrombus in right lower pulmonary artery
Which of the following is most appropriate step in management :
A. Switch Enoxparin to sodium heparin
B. Thrombolytic therapy
C. Same management
D. Thromboectomy

Note to Remember
She was admitted for DVT, so she is already on the therapeutic dose, NO NEED to change the treatment, unless if
she was UNSTABLE (hypotensive) —> in that case I would go for B. Thrombolytic therapy

Normal delivery and she did episiotomy the developed retroperitoneal collection 5x3, which was bluish and painful,
what is the treatment?
A- packing
B-aspiration
C-surgical evacuation
D-Observe

Note to Remember
Vaginal hematoma:
Surgical evacuation when (large) 5cm and more, expanding or symptomatic (painful).. otherwise Observation with
(RICE) Rest, Ice, Compression and Elevation.

Patient after SVD found placenta failed to deliver and cannot extract. The patient refused hysterectomy. It was
managed by ligating the placenta and started on Methotrexate therapy, what’s the complication of this case?
a. Bleeding
b. Infection
c. DIC

Note to Remember
The patient was managed by conservative management for placenta accreta.
The complications of uterine conservation with placenta left in situ is:
Most commonly is severe vaginal bleeding (53%). Followed by sepsis “which can lead to DIC” (6%).

By: Wafa AlSalem 123


OBGYN Related Breast diseases and Breastfeeding
Topic Overview:

Breastfeeding counseling is best done


A. before conceiving
B. 1st trimester or prenatal
C. Postpartum

Adnexal masses
Topic Overview:

How to defrinitaite between benign vs malignant ovarian tumor?


A. Hypoechoic
B. Bilateral
C. Cyst with septate

According to ACOG
What ultrasound findings suggest malignancy of adnexal mass?
- Cyst size greater than 10 cm
- Papillary or solid components
- Irregularity
- Presence of ascites
- High color Doppler flow.

**Hypoechoic mass on US means a more solid tissue.

By: Wafa AlSalem 124


Leiomyoma (Uterine Fibroids) and Leiomyosarcoma
Topic Overview:

Postmenopausal women with a history of fibroid “certain size” increased. What’s the diagnosis “endometrium were
5 mm pt on tamoxifen”
A. Leiomyosarcoma
B. Endometrial cancer
C. Adenomyosis

Note to Remember
If she is on Tamoxifen and her endometrium increased in thickness with Abnormal uterine bleeding ONLY —>
choose endometrial cancer.
BUT if she is on Tamoxifen + fibroid increased in size (regardless of the endometrial thickness) —> choose
Leiomyosarcoma

24 years old female married not complaining of anything incidental finding of subserosal fibroid how would u
manage?
A. Observe and follow up after 6 months
B. Myomectomy
C. OCP
D. Progestin

Note to Remember
Leiomyoma (Fibroids) (ACOG)
-Expectant management considered for patients who are asymptomatic or for those who do not desire intervention
-Medical treatment include agents that address only bleeding symptoms, don’t reduce the fibroid size
-Myomectomy is recommended for symptomatic leiomyomas in patients who desire uterine preservation or future
pregnancy

Uterine Polyps
Topic Overview:

By: Wafa AlSalem 125


Endometriosis
Topic Overview:

Post menepause presenting with abdominal pain and dyschezia last menstrual period was 15 months ago what to
give?
A-Depo provera injection (Progestogen only)
B- Conjecated estrogen pill
C- OCP

Note to Remember
Postmenopausal endometriosis:
- Progestogen only is the first line treatment
- OCP is the alternative first line

39 years old female who has three children and completed her family diagnosed as endometrioma which was
removed 2 years ago, right ovary cyst she presented to the clinic with mild to moderate dysmenorrhea and
dyspareunia during intercourse and chronic lower abdominal pain. Pelvic ultrasound shows: Left ovary endometrioma
cyst 6x7 in size.

A. Removal of cyst more than 10 in size


B. Aspiration of cyst content under ultrasound guidance
C. Immediate hysterectomy and salpingectomy oophorectomy
D. Removal of cyst by laparoscopic ablation of endometrioma spots.

Note to Remember
Hysterectomy is the definitive treatment for women with persistent bothersome symptoms of endometriosis who
do not plan future childbearing and who have failed both medical therapy and at least one conservative surgery
(OUR PATIENT)

By: Wafa AlSalem 126


Adenomyosis
Topic Overview:
Adenomyosis
Risk factors:
⁃ Previous uterine surgery, C/S, D&C
⁃ Childbirth
⁃ Middle age
Clinical presentation:
⁃ Commonly report symptoms similar to those reported by patients with (ddx) endometriosis.
⁃ Common complaints include menorrhagia, dysmenorrhea, metrorrhagia, chronic pelvic pain and dyspareunia
Physical Examination:
- Enlarged and tender uterus!!
Investigations:
⁃ Transvaginal ultrasound: first-line imaging choice
⁃ Magnetic resonance imaging (MRI): best imaging modality
⁃ Histologic diagnosis after hysterectomy: Definitive DIAGNOSIS

44 years women is complaining of severe dysmenorrhea and menorrhagia, pelvic examination reveals the uterus is
symmetrically enlarged and tender. Endometrial biopsy is normal
Which of following dx?
A.Adenomyosis
B.Leiomyomas
C.Endometriosis
D.Sarcoma
!!‫ ﺗﯿﺒﻜﺎل ﺳﯿﻨﺎرﯾﻮ‬،‫اﻻدﯾﻨﻮﻣﺎﯾﻮﺳﺰ ﯾﺼﺮخ ﯾﻘﻮل اﺧﺘﺎرﯾﻨﻲ‬

41-year-old P5 +3 presented to the clinic complaining of abnormal uterine bleeding her Menstrual period is regular
every 30-day associated with blood clots and pain that is not relieved by simple analgesic she had previous
myomectomy, she is a known case of PCOS And her BMI is 40?

A-Adenomyosis
B-Endometriosis
C-Uterine fibroid
D-Endometrial hyperplasia

Let’s Exclude!!
- Adenomyosis? Previous uterine surgery, multiparity and her age (all are risk factors for adenomyosis!)
- Endometriosis -> will present with dysmenorrhea mainly (presentation goes with adenomyosis more)
- Uterine Fibroid -> will present with bleeding only
- Endometrial hyperplasia -> will not present with dysmenorrhea

By: Wafa AlSalem 127


Asherman’s Syndrome
Topic Overview:
IUAs, or intrauterine synechiae, is a condition in which scar tissue develops within the uterine cavity. IUAs that
are accompanied by symptoms (eg, infertility, amenorrhea) are referred to as Asherman syndrome
- IUAs appear to result from trauma to the basalis layer of the endometrium

G3p0, A2 now at 5 weeks presented with spotting on examination open os and no active bleed. History showed 2
abortions at 2nd trimester, last one with D&C diagnosed as incompetent cervix. Your diagnosis now for the third
pregnancy of this patient?
a.Asherman syndrome
b.Incompetent cervix
c.Chromosomal abnormalities

According to ACOG:
- Approximately 50% of all cases of early pregnancy loss are due to fetal chromosomal abnormalities
- In rare cases, after a D&C has been performed after a miscarriage, bands of scar tissue, or adhesions, may
form inside the uterus. This is called Asherman syndrome

Pregnant at 5 weeks gestation with heavy bleeding and clots, she has a hx of 5 previous abortions all her Previous
abortions were at 2nd trimester, she had multiple D&C, what causes her current bleeding?
A. Asherman syndrome
B. Cervical incompetence
C. Chromosomal abnormalities

Note to Remember
The risk of asherman syndrome increases with the increase of the D&C procedures the patient had. This scenario
goes more with asherman than chromosomal abnormalities because she had MULTIPLE D&C

G3p0, A 25 weeks presented with spotting on examination open os and no active bleed. History showed 2 abortions
at 2nd trimester, last one with D&C diagnosed as incompetent cervix. Your diagnosis now for the third pregnancy
of this patient?
A. Asherman syndrome
B. incompetent cervix
C. chromosomal abnormalities

Note to Remember
The risk of asherman syndrome increases with the increase of the D&C procedures the patient had.
Asherman ‫ ﺑﺨﺘﺎر‬multiple D&C ‫ﻟﻮ ﻗﺎﻟﻲ‬
‫ﯾﻘﻮل ﻣﺎﻟﻜﺶ دﻋﻮى‬asherman ‫ ﻟﺴﺎن ﺣﺎل ال‬incompetent cervix ‫وﺑﻌﺪﯾﻦ ھﻲ اﻟﺮﯾﺪي داﯾﻘﻨﻮﺳﺪ ب‬

A woman has previous abortion, and she was managed by D&C, and now after 1 year she is presenting with
amenorrhea, what the diagnosis?
A- Asherman syndrome
B- Sheehan syndrome

By: Wafa AlSalem 128


Patient with a history of multiple D&C for recurrent miscarriages , she can't get pregnant, what is the endometrial
layer removed?
A. Stratum compactum
B. Stratum spongiosum
C. Stratum functionalis
D. Stratum basalis

Menopause
Topic Overview:

60y old lady present with lower genital bleeding, she described it as Scanty and barely stain the pad , what is the
source of bleeding?
a.Fallopian tube
b.Ovary
c.Uterus
d.Genital tract

Note to Remember
Bleeding in elderly is endometrial cancer until proven otherwise.

Valvular and Vaginal Disorders


Topic Overview:

Q about lesion in labia majora in post-menopausal female showed dysplasia (carcinoma in situ I think) what to do:

Steroid cream

Local excision

Vulvectomy

Repeat test after 6 months

By: Wafa AlSalem 129


Cervical cancer and screening
CERVICAL CANCER SCREENING GUIDELINES
The current guidelines in the United States recommend screening women for cervical cancer between the ages of
21 and 65 (Saslow, 2012).

The screening guidelines are as follows:

• Cervical cancer screening should not be performed in women younger than 21 years of age, regardless of age of
onset of sexual activity

• Screening is not recommended for women >65 years of age who have had three consecutive negative Pap tests or
two consecutive negative HPV tests, provided they have had no history of high-grade dysplasia (CIN2/3) or cancer
(CIN2+) in the past 20 years.
However, women presenting at age 65 years of age or older who have not had previous screening should undergo
Pap and HPV testing.

• Screening with Pap test or HPV testing is not recommended for women who have had a hysterectomy with removal
of the cervix and who do not have a history of CIN2+.

By: Wafa AlSalem 130


FOLLOW UP OF ABNORMAL CERVICAL CANCER SCREENING TESTS

COLPOSCOPY
- Colposcopy is often the first step in evaluation of women with abnormal cytology.
- Cervical biopsies should be performed of any acetowhite lesions noted
- For a thorough and complete exam, the entire transformation zone must be assessed (“satisfactory”
colposcopy). If some portions of the transformation zone cannot be visualized as they extend into the
endocervical canal or for other reasons, the colposcopy is considered “unsatisfactory” as the examiner is
unable determine the presence or extent of abnormal tissue.
- In the case of abnormal cytology and an unsatisfactory colposcopy, it is recommended that an endocervical
curettage (ECC) be performed.
By: Wafa AlSalem 131
CERVICAL DYSPLASIA IN PREGNANCY
In pregnancy, the cervix becomes larger, the blood supply to the cervix is increased, and decidual changes in the
epithelium can be confused with CIN.
- The ASCCP provides guidelines for the management of abnormal cytology in pregnancy (Massad, 2013).
Colposcopy is safe in pregnancy. However, biopsies should only be performed if there is suspicion for
invasive disease.
- It is highly unlikely for dysplasia to progress significantly during pregnancy, and in the majority of patients
further evaluation can be postponed until 6 to 8 weeks after delivery.
- If invasive cancer is suspected, cervical biopsies are indicated and can be performed safely during
pregnancy. However, Endocervical curettage (ECC) should never be performed during pregnancy.
- If CIN2/3 is diagnosed, further evaluation and treatment can be delayed until the postpartum period. If
there is significant concern for a dysplastic lesion, a follow-up with colposcopy or repeat cytology is
acceptable at intervals no more frequent than every 12 weeks. If invasive cancer is diagnosed, a conization
procedure under anesthesia can be performed.

BIOPSY (According to Berek and Hacker’s Gynecologic Oncology and UTD)


- Any visible lesion, tumor growth, or ulceration in the cervix should undergo office punch biopsy or loop
excision for histologic confirmation (regardless of previous benign cervical cytology results).
- Any cervix that is unusually firm or expanded should also undergo punch biopsy and endocervical curettage
(ECC) (even if the cervical cytology test does not show evidence of neoplasia)

CERVICAL INTRAEPITHELIAL NEOPLASIA (CIN)


- Is precancerous lesion of the squamous epithelium of the cervix
Diagnosis
- Based on tissue examination of a cervical biopsy specimen.

CIN Grading:
Graded as 1, 2, or 3 depending on the how much of the epithelial layer contains atypical cells.
• CIN 1, or mild dysplasia: frequently spontaneously regresses, often within weeks to months.
• CIN 2: when cellular atypia involves two thirds of the thickness of the epithelium. The process still remains
reversible at this stage, with approximately 40% regressing spontaneously without treatment.
• CIN 3 (severe dysplasia and carcinoma in situ) : When the cellular atypia involves more than two thirds of
the epithelium, and treatment is recommended.

Management:

• CERVICAL INTRAEPITHELIAL NEOPLASIA 1 (CIN 1, or low grade dysplasia)


In almost all cases, CIN 1 is a manifestation of a transient HPV infection, and the regression rates are high.
Patients with CIN 1 require follow-up to ensure that the lesion regresses.

- Cervical Intraepithelial Neoplasia 1 with Low-Grade Squamous Intraepithelial Lesion Cytology

Managed with observation.


o >24 years old -> Co-testing with cytology and HPV testing at 12 months
o 21 to 24 years old -> Repeat cytology(Pap smear) alone at 12 months
**If CIN 1 persists for more than 2 years, a definitive excisional procedure can be considered.

- Cervical Intraepithelial Neoplasia 1 with High-Grade Squamous Intraepithelial Lesion Cytology


If the diagnosis of CIN 1 is preceded by cytology showing HSIL or AGC, there is a higher chance of
underlying CIN 2/3, and more aggressive management should be considered.

By: Wafa AlSalem 132


o In patients who have completed childbearing -> an excisional procedure is recommended.
o In women who desire future fertility-> close follow-up with cytology and colposcopy at 6
months is recommended.

•CERVICAL INTRAEPITHELIAL NEOPLASIA 2/3 (CIN 2/3, or high grade dysplasia)


Most women with CIN 2/3 should be treated with an ablative or excisional procedure.
o Young women and those desiring future fertility may be managed with careful observation, including cytology
and colposcopy initially every 6 months with long-term follow-up depending on findings.
o Pregnant women with CIN 2/3 and no evidence of invasion may be observed during the pregnancy, with
evaluation delayed until 6 weeks postpartum.
**Long-term follow-up for at least 20 years is recommended, even if this extends screening past age 65

TREATMENT OF CERVICAL DYSPLASIA:


Accomplished by ablation or excision:
- Ablative methods
Treat CIN but do not provide further diagnostic information
Specific criteria for ablative therapies include the following:

• Satisfactory colposcopy with visualization of entire cervical squamocolumnar junction


• Biopsy confirming presence of CIN; abnormal cytology alone is not sufficient
• Lesion does not involve the endocervical canal and negative endocervical curettage (if available)
o Cryotherapy
Is a commonly used treatment for CIN lesions that is safe, effective.
However, it does not provide a specimen for pathology review and in many
high-resource settings has been replaced by LEEP.
o CO 2 laser ablation.
Has largely been replaced by LEEP. Similar to cryotherapy, there is no
specimen for pathologic evaluation
- Excisional procedures
Have the advantage over ablative procedures of providing a pathologic specimen for
further diagnostic information.
Specific indications for an excisional procedure over an ablative procedure include the following:

• Suspected microinvasion
• Adenocarcinoma in situ or other glandular abnormalities
• Unsatisfactory colposcopy in which the transformation zone is not fully visualized
• Lack of correlation between cytology and colposcopy/ biopsies
• Unable to rule out invasive disease
• Lesion extending into the endocervical canal
• Endocervical curettage showing CIN or a glandular abnormality
• Recurrence after an ablative or previous excisional procedure

o Loop electrosurgical excision procedure (LEEP)


Currently the most common method for the treatment of CIN 2/3 in the
United States. Can be performed safely in the office.
The removed tissue is examined histologically for diagnosis and evaluation
of margin status. Management guidelines for positive margins are provided
by the ASCCP and include reexcision versus follow-up cytology and
endocervical sampling, depending on pathology results, patient age, and the
desire for future fertility.
By: Wafa AlSalem 133
o Cold knife conization (CKC)
Offers little advantage over LEEP. However, CKC is advantageous in
patients with glandular abnormalities or suspicion of invasive cancer
o CO 2 laser conization.
**The choice of treatment modality depends on the availability of equipment and the experience and expertise of
the clinician.
**When treatment is performed, we prefer excision over ablation for all CIN grades because it provides a
diagnostic specimen
**Hysterectomy is not recommended as the initial treatment of cervical dysplasia, as it is usually unnecessary for
the treatment of CIN.

Extra reading from UTD


Management of CIN 2,3 in nonpregnant patients <25 years is as follows:
●If histologic HSIL is unspecified (reported as histologic HSIL or HSIL [CIN 2,3]):
-> Observation or treatment is acceptable.

●If CIN 2 is specified:


-> Observation is preferred.
->Treatment is an acceptable option.

●If CIN 3 is specified or if the entire SCJ or lesion is not visible on colposcopy
-> Treatment is recommended.
-> Observation is unacceptable.

**When treatment is planned, a diagnostic excisional procedure is performed; ablation is an acceptable alternative.

Management of nonpregnant patients ≥25 years is as follows


●If histologic HSIL is unspecified (reported as histologic HSIL or HSIL [CIN 2,3] without distinction of CIN 2 or
CIN 3)
-> Treatment is preferred; in these patients, CIN 3 cannot be excluded, and, therefore, patients are managed as if
CIN 3 were present.
-> Observation (with colposcopy and HPV testing at 6 and 12 months) is acceptable.

●If CIN 2 is specified


-> Treatment is recommended.
-> Observation (with colposcopy and HPV testing at 6 and 12 months for up to two years) is acceptable if all of the
following are present:
-The patient's concerns about potential adverse pregnancy outcomes after an excisional procedure outweigh the
concerns about cancer.
-The entire SCJ and lesion are visible on colposcopy, and ECC does not demonstrate CIN 2+ or ungraded CIN.

●If CIN 3 is specified, if the entire SCJ or lesion are not visible on colposcopy, or if the ECC is CIN 2+
->Treatment is recommended.
-> Observation is unacceptable.

When treatment is planned, a diagnostic excisional procedure (LEEP, cold knife cone, and laser cone biopsy) is
preferred. An ablation (with cryotherapy, laser ablation, and thermoablation) is an acceptable alternative.

By: Wafa AlSalem 134


When should married women start to have pap smear screening?
1. 20-24
2. 25-29
3. 30-34
4. 35-39

When to start HPV SCREENING for cervical cancer


a. 20-24
b. 25-29
c. 30-34
d. 35-39

Female 27 years old, she is asymptomatic, her last pap smear was 3 years ago and it showed unconcerned squamous
cells. What is the most appropriate thing to do?
A-Repeat pap with cytology
B-No need and reassure
C-Colposcopy
D-Cervical swab

20 y/o girl divorce came to clinic first visit, When to do pap smear;
A-Now
B-1 year after at 21 years
C-5 year
D-No need

22 years old, female married never did pap smear before when to do it?
A. Immediately
B. 3 years
C. 5 years
D. No need

According to Comprehensive Gynecology Textbook


Cervical cancer screening:
⁃ Cervical cancer screening should not be performed in women younger than 21 years of age, regardless of
age of onset of sexual activity
⁃ 21-29 -> pap smear every 3 years
⁃ 30-65-> pap smear every 3 years Or pap smear + HPV testing every 5 years
⁃ >65 -> no need for screening if the previous results were negative.

By: Wafa AlSalem 135


Newly married 22 years old comes to you for a regular follow-up for her gynecological health. She is medically free
and has regular 28 day cycle, complaining of mobile breast lumps bilaterally that are painful right before meneses.
Examination was normal. She wants to get pregnant, but she is worried about her breast lump. What is the
screening test to be performed?
A. Bilateral mammogram
B. Papanicolaou test
C. Breast ultrasound

There’s no indication for mammogram or US in her case (because her examination is normal).
Pap smear screening is indicated in her case because she’s married.
According to UTD
- Cyclical pain affects two-thirds of patients with true mastalgia. Cyclical pain is associated with hormonal
fluctuations of the menstrual cycle, usually presenting in the week prior to onset of menses. It is
frequently bilateral and most severe in the upper outer quadrant of the breasts.

- Women with cyclical or bilateral nonfocal breast pain usually do not require imaging

Newly married young woman came for routine check up gyne, highest diagnostic value?
A. General appearance
B. Vaginal inspection
C. Abdominal exam
D. Pelvic digital exam

According to ACOG and Williams Gynecology:


Well Woman Visit:
- A comprehensive history. This history includes symptoms; medications; allergies; and medical, surgical,
family, social, and gynecologic history, including questions on reproductive, sexual, and mental health (using
screening tools as indicated).
- Physical examination, including: breast and pelvic examination (inspection of external genitalia, speculum,
and bimanual palpation of the adnexa, uterus and bladder, and sometimes rectovaginal examination) when
indicated by medical history or symptoms.

Benefits of the pelvic examination include early detection of treatable gynecologic condition
‫ھﺪف اﻟﺰﯾﺎره اﻧﻲ اﺷﻮف ﻋﻨﺪھﺎ اﻣﺮاض اﻗﺪر اﻋﺎﻟﺠﮭﺎ زي ﻛﺎﻧﺴﺮز وﻛﺬا‬
‫ ﺑﺲ ھﻮ ﻗﺎﻟﻚ وش اﺣﺴﻦ ﺷﻲء ﺑﺎﻟﻔﺤﺺ!! ﻓﯿﻌﻨﻲ ﻧﺨﺘﺎر‬،‫ﻣﻔﺮوض ﺑﺲ اﺳﻮي ھﺴﺘﻮري ﺑﮭﺬي اﻟﺰﯾﺎره وﻋﻠﻰ ﺣﺴﺐ اﻟﺘﺎرﯾﺦ اﻟﻤﺮﺿﻲ واﻻﻋﺮاض اﻗﺮر اﺳﻮي ﻓﺤﺺ وﻻ ﻻ‬
pelvic digital examination ‫اﻻﻓﺼﻞ وھﻮ‬

Let’s Exclude!
- General appearance -> if he asked next not the highest diagnostic value
- Vaginal inspection -> I will inspect the vagina only, this is not the highest diagnostic value
- Abdominal examination-> not a component of well woman visits
- Pelvic digital exam-> I would palpate the vaginal walls, examine the cervix, adnexa, and uterus! ‫ھﺬا اﺻﺢ ﺷﻲء‬

Pregnant with suspicious cervical lesion, how to confirm the diagnosis or what is next?
A. Cone biopsy
B. Colposcopy directed biopsy
C. Pap smear
D. Endocervical curettage

By: Wafa AlSalem 136


ANOTHER RECALL

34 y/o female 30 gestation with painless vaginal bleeding, did vaginal examination found suspicious mass (see
report)
Report: US shows that the fetus corresponds to the Gestational age
What is the most appropriate next step?
A. Colposcopy
B. Cone biopsy
C. Pap smear
D. Endocervical curettage

According to Berek and Hacker’s Gynecologic Oncology and Uptodate:


- Any visible lesion, tumor growth, or ulceration in the cervix should undergo office punch biopsy or loop
excision for histologic confirmation (regardless of previous benign cervical cytology results).
- Any cervix that is unusually firm or expanded should also undergo punch biopsy and endocervical curettage
(ECC) (even if the cervical cytology test does not show evidence of neoplasia)

According to Comprehensive Gynecology Textbook:


- Colposcopy is safe in pregnancy.
- Endocervical curettage (ECC) should never be performed during pregnancy
- If invasive cancer is diagnosed during pregnancy, a conization procedure under anesthesia can be
performed.

In cervical examination patient found to have lesion what to do:


A) Pap test
B) RNA test
C) cone biopsy
D) Colposcopy directed biopsy

Same note as the previous question

Female with cervical lesion measuring 11mmx12mm with irregular borders, pap was done, no results yet, what to do?
A. Excise the lesion
B. Biopsy
C. Reassure until pap results are available
D. HPV test

Same note as the previous question

A woman presented with dyspareunia but denies any heaviness or urinary symptoms found to have a cervical mass
on examination *attached photo, what is your Mx?
A. Discharge
B. Follow up
C. Excision at the clinic
D. Pap smear

By: Wafa AlSalem 137


Young patient did pap smear in screening and showed HISL, the biopsy showed carcinoma insitue, she wants to
conserve her fertility
A. Cold knife (conization)
B. Electrosurgical loop (LEEP)
C. Cryotherapy
D. Laser

According to Comprehensive Gynecology Textbook:


CERVICAL INTRAEPITHELIAL NEOPLASIA (CIN)
**CIN 3 (severe dysplasia and carcinoma in situ) : When the cellular atypia involves more than two thirds of the
epithelium, and treatment is recommended.
**Management of CERVICAL INTRAEPITHELIAL NEOPLASIA 2/3 (CIN 2/3, or high grade dysplasia)
o Most women with CIN 2/3 should be treated with an ablative or excisional procedure.
- Ablative methods
o Cryotherapy
Is a commonly used treatment for CIN lesions that is safe, effective. However, it does not
provide a specimen for pathology review and in many high-resource settings has been replaced by
LEEP.
o CO 2 laser ablation.
Has largely been replaced by LEEP. Similar to cryotherapy, there is no specimen for pathologic
evaluation
- Excisional procedures
Have the advantage over ablative procedures of providing a pathologic specimen for further diagnostic
information.

o Loop electrosurgical excision procedure (LEEP)


Currently the most common method for the treatment of CIN 2/3 in the United States. Can be
performed safely in the office.
The removed tissue is examined histologically for diagnosis and evaluation of margin status.
Management guidelines for positive margins are provided by the ASCCP and include reexcision
versus follow-up cytology and endocervical sampling, depending on pathology results, patient age,
and the desire for future fertility.

o Cold knife conization (CKC)


Offers little advantage over LEEP. However, CKC is advantageous in patients with glandular
abnormalities or suspicion of invasive cancer
o CO 2 laser conization.
**When treatment is performed, we prefer excision over ablation for all CIN grades because it provides a
diagnostic specimen
**Hysterectomy is not recommended as the initial treatment of cervical dysplasia, as it is usually unnecessary for
the treatment of CIN.

According to UTD
Factors to consider in choosing excision versus ablation
Is Future pregnancy planned? — Patients planning a future pregnancy may choose to avoid excision because it has
been associated with an increased risk of adverse obstetric outcomes (second-trimester pregnancy loss, preterm
prelabor rupture of membranes, preterm delivery) in large observational studies. Ablation, in theory, has a lower
risk of adverse obstetric outcomes given that the cervix is better preserved than with excision. However, CIN
itself may pose an increased risk of preterm birth, regardless of treatment method. Patients should be counseled
about these issues, which are discussed in detail separately.
By: Wafa AlSalem 138
Theoretically, the integrity of the cervix is better preserved following ablation than excision

‫ ﻓﻼزم‬،‫ﻧﻔﺴﮫ ﻣﻤﻜﻦ ﺧﻄﺮ ﻋﻠﻰ اﻟﺤﻤﻞ وﻣﻤﻜﻦ ﯾﻜﻮن ﻋﻨﺪھﺎ ﻛﺎﻧﺴﺮ وھﻮ ﯾﺄﺛﺮ ﻋﻠﻰ اﻟﺤﻤﻞ ﺳﻮاء ﻋﺎﻟﺠﺘﮫ او ﻻ‬CIN3 ‫ﻗﺮﯾﺖ ﺑﺄﻛﺜﺮ ﻣﻦ ﻣﻜﺎن ان‬
.‫ ﻣﻨﮭﺎ ﻋﻼج اﻓﻀﻞ وﻣﻨﮭﺎ داﯾﻘﻨﻮﺳﺰ ﻟﻮ ﻋﻨﺪھﺎ ﻛﺎﻧﺴﺮ وﻻ ﻻ‬excision ‫اﺳﻮﯾﻠﮭﺎ‬
‫ وش اﻟﺨﻄﻮه‬LEEP ‫ وﺑﻌﺪ ﻣﺎﻧﻘﺮر ﻋﻠﻰ ﺣﺴﺐ رﯾﺰوﻟﺖ ال‬،‫ ﻋﻠﻰ اﻟﻠﻲ ﺑﺘﺤﻤﻞ‬conization ‫ ﻻﻧﮫ اﻗﻞ ﺧﻄﻮره ﻣﻦ‬LEEP ‫وأول ﺷﻲء ﻧﺴﻮي‬
‫اﻟﻠﻲ ﺑﻌﺪ‬

38years old female with non invasive carcinoma in cervix of young woman wishing to preserve fertility Best
treatment:
A-Endometrial ablation
B-Hysterectomy
C- Cold knife conization
D- Electrosurgical loop (LEEP)

Non invasive Carcinoma of the cervix = Carcinoma in situ = CIN3


Same answer and explanation as the previous question

40 years old with post coital bleeding and intermenstrual bleeding She had 3 pap smears positive and did a
cystoscopy showed intraepithelial carcinoma ,What is the next step ?
A. MRI abdominal
B. Cone biopsy
C. LEEP
D. CT abdomen chest pelvis

This is a Case of CIN3


o Loop electrosurgical excision procedure (LEEP)
Currently the most common method for the treatment of CIN 2/3 in the United States. Can be
performed safely in the office.
The removed tissue is examined histologically for diagnosis and evaluation of margin status.
Management guidelines for positive margins are provided by the ASCCP and include reexcision
versus follow-up cytology and endocervical sampling, depending on pathology results, patient age,
and the desire for future fertility.

colposcopy not cystoscopy ‫اﻧﺎ ﻣﻌﺘﺒﺮﺗﮭﺎ اﻧﮭﺎ ﺑﺎد رﯾﻜﻮل و اﻧﮫ‬


because why would they do cystoscopy??

33 years old female came to you at your office and her papsmear report was unsatisfactory for evaluation, the
best action is:
a) Consider it normal & D/C the pt.
b) Repeat it immediately
c) Repeat it as soon as possible
d) Repeat it after 6 months if considered low risk
e) Repeat it after 1 year if no risk

By: Wafa AlSalem 139


The correct answer would be repeat after 2-4 months.
->we can’t do it immediately, because we have to give the cervical cells time to renew.
->6months and 1 year period is also wrong. I would go with as soon as possible.

24 female did pap have abnormal results “exactly wriiten like this they didn’t no mention what is the result” What
you will do?
A-Colposcopy
B- Repeat pap after 3 months
C- Reassurance

By exclusion
B and C are wrong.

58 y/o female did Pap smear and showed (ASC-US), her treating physician prescribe for her topical vaginal
estrogen for 1 month, she came back after that and Pap smear repeated and it was also (ASC-US ). What you will
do for her?
A- Colposcopy
B- Punch biopsy
C- HPV testing
D- No further investigations

By: Wafa AlSalem 140


34 years old women did pap smear show ASCUS what to do next?
A- Repeat pap.
B- HPV
C- Colposcopy
D- Conization

Same note as the previous question

40y/o female patient underwent PAP smear histopathology showed ASCUS, your next step?
A. Do HPV test
B. Colposcopy.
C. Re-evaluate after 6 months
D. Surgery

Same note as the previous question

A case of ASC-H. What to do?


A. Conization
B. Colposcopy
C. HPV
D. Repeat pap

Same note as the previous question

Pap test came with high grade squamous intraepithelial lesion, next step is?
A. Colposcopy
B. repeat pap test
C. hysterectomy

Same note as the previous question

Female heavy smoker (2packes/week) for the past 7 years. and since then show was doing Pap smear and all
negative. and she had previously infected with benign warts. this time it shows LSLI, what will you do?
A- Colposcopy
B- HPV DNA
C- Pap smear

LSIL Follow Up:


- 21-24 à repeat pap smear after 1 year
- 25-29 à Colposcopy
- >30 à HPV test à (HPV positive -> Colposcopy) (HPV negative -> repeat pap smear and HPV after 1
year)

HPV ‫ ﺑﺨﺘﺎر‬٣٠ ‫ وﻟﻮ‬Colposcopy ‫ ﺑﺨﺘﺎر‬٢٩ -٢٥ ‫اﻟﺤﻞ راح ﯾﻜﻮن ﻋﻠﻰ ﺣﺴﺐ ﻋﻤﺮھﺎ !! ﻟﻮ‬
..‫ﻣﺴﺘﺤﯿﻞ ﺗﻜﻮن اﺻﻐﺮ ﻣﻦ ھﺎﻟﻌﻤﺮ ﻋﻠﻰ ھﺎﻟﺴﯿﻨﺎرﯾﻮ‬

30 yrs female did pap smear shows LSIL, What is the next step?
A/ Coloposcoy

By: Wafa AlSalem 141


B/ repeat pap after one year
C/ HPV
D/ Biopsy

Same note as the previous question

30y old female came for pap screening, all her past results were negative, now results show low grade squamous
epithelial lesion. What's the appropriate next step?
A. Humen papilloma virus
B.Colposcopy
C. evaluate after 6 months
D. Surgery .

Same note as the previous question

31 female, came for pap smear when to do it?


A. Every 5 years
B. Every 1 year
C. No need
D. If 3 is negative, no need to re-screen again

By exclusion
B and C and D are wrong.

Correct answer is:


- Every 3 years pap smear alone
- Every 5 years Pap smear + HPV testing

ANOTHER RECALL

31 female, came for pap smear when to do it?


A. Every 2yrs
B. Every 3yr
C. Every 5 years
D. If 3 is negative, no need to re-screen again

Same note as the previous question

40 y Female with hx of Wart 7 or 10 years back, last pap smear normal, what is investigation u went to do in this
visit?
A. Repeat pap smear
B. Colposcopy

By: Wafa AlSalem 142


Female patient Pap smear done before 7 years ago result was atypical hyperplasia, Now came to clinic what to do:
A. Repeat Pap smear
B. Colposcopy
C. CT

last pap smear 7 years ago! I have to repeat it and manage according to the new results.

How to do a Pap smear?


A. From transformation Zone
B. From the Ectocervix
C. From the endocervix

According to Comprehensive Gynecology Textbook:


Cervical Cytology Testing (Pap smear)
- Cells are sampled from the transformation zone, which is the area of the cervix where cervical cancer
can develop. The transformation zone includes the squamocolumnar junction, which is the area where
the squamous epithelium of the ectocervix meets the columnar epithelium of the endocervix.

Female 30-year-old her pap smear, result showed squamous cell ca (SCC), what to do next?
A. Colposcopy directed biopsy
B. Repeat pap smear
C. Total hysterectomy
D. Neoadjuvant chemotherapy

Pap smear is only a screening test, it’s not diagnostic.


We should do biopsy first after the pap smear, and manage according to the biopsy result.

Females with abnormal Pap smear and colposcopy diagnosed with invasive cervical cancer, what is the most
appropriate next step?
A- Clinical staging
B- Hysterectomy and chemotherapy
C- Hysterectomy and radiotherapy

According to Berek and Hacker’s Gynecologic Oncology


- Cervical cancer has always been staged clinically

By: Wafa AlSalem 143


18 (or 25 yo) female, worraied about cervical cancer, she took her first dose of HPV vaccine 3 months ago. What is
the best thing to do at this visit today?
A. Schedule appointment after 3 months
B. No need to do anything at this visit
C. Give 2nd dose at this visit
D. Repeat 1st dose

According to Centers for Disease Control and Prevention (CDC)


HPV Vaccine Schedule and Dosing
**Who Gets 2 Doses? 9-14 Years old

• A 2-dose schedule is recommended for people who get the first dose before their 15th birthday.
• In a 2-dose series, the second dose should be given 6–12 months after the first dose (0, 6–12 month
schedule).
• The minimum interval is:
- 5 months between the first and second dose.
- If the second dose is administered after a shorter interval, a third dose should be administered a
minimum of 5 months after the first dose and a minimum of 12 weeks after the second dose.
• If the vaccination schedule is interrupted, vaccine doses do not need to be repeated (no maximum interval).
• Immunogenicity studies have shown that 2 doses of HPV vaccine given to 9–14 year-olds at least 6 months
apart provided as good or better protection than 3 doses given to older adolescents or young adults.

**Who Gets 3 Doses? 15-26 Years old

• A 3-dose schedule is recommended for people who get the first dose on or after their 15th birthday, and
for people with certain immunocompromising conditions.
• In a 3-dose series, the second dose should be given 1–2 months after the first dose, and the third dose
should be given 6 months after the first dose (0, 1–2, 6 month schedule).
• The minimum intervals are:
- 4 weeks between the first and second dose.
- 12 weeks between the second and third doses, and 5 months between the first and third doses.
- If a vaccine dose is administered after a shorter interval, it should be re-administered after another
minimum interval has elapsed since the most recent dose.
• If the vaccination schedule is interrupted, vaccine doses do not need to be repeated (no maximum interval).

By: Wafa AlSalem 144


Endometrial Hyperplasia and Carcinoma
Topic Overview:

57 y/o female complaining of abnormal uterine bleeding she has an endometrial polyp, on US endometrial lining was
19mm, what will you offer to this patient AT THIS STAG “next”?
A) Open hysterectomy
B) Laparoscopic hysterectomy
C) Hysteroscopy with polypectomy

57 y/o female complaining of abnormal uterine bleeding she has an endometrial polyp, on US endometrial lining was
19mm, what is the definitive management for this patient?
A) Open hysterectomy
B) Laparoscopic hysterectomy
C) Hysteroscopy with polypectomy

Note to Remember
Abnormal uterine bleeding in elderly is endometrial cancer until proven otherwise ->
**Initially “to diagnose”? D&C biopsy or hysteroscopic sampling (endometrial biopsy).
**Definitive Treatment is by laparoscopic hysterectomy

So, the answer will be different based on if its initially or definitive.

If they asked about definitive or best .. this is my explanation for my answer:

Why did we choose hysterectomy?


Because it’s for patients who do not desire future fertility, or those with pathology suggesting high risk of
concomitant endometrial carcinoma. Risk factors for concomitant endometrial cancer are increased age and
transvaginal ultrasound showing endometrial thickness ≥20 mm

Why not hysteroscopy with polypectomy?

The treatment for postmenopausal women with polyps is Hysterscopic Polypectomy.

But the risk of malignancy in a polyp is highest in postmenopausal women, and those with bleeding compared with
those without bleeding.

I choose hysterectomy based on the advanced age (55 and above), AUB, and the endometrial thickness
(Endometrial carcinoma). Not only the polyp
“See the patient as a whole” that’s my point of view and i could be wrong.

Old lady with uterine fundal mass. (Uterine CA) underwent surgery. What lymph nodes to resect?
a. External iliac lymph node
b. Internal iliac lymph node
c. Deep inguinal lymph node
d. Para-aortic lymph node

By: Wafa AlSalem 145


Highest risk factor of endometrial cancer
A. Age
B. PCOS
C. smoking

According to ACOG:
Prolonged exposure to unopposed estrogen, whether endogenous or exogenous, is associated with most cases of
type I endometrial cancer. Unopposed endogenous estro gen exposure occurs in chronic anovulation (eg, polycystic
ovary syndrome), with estrogen-producing tumors, and with excessive peripheral conversion of androgens to
estrone in adipose tissue.

Urogynecology
Topic Overview:

By: Wafa AlSalem 146

You might also like